Sie sind auf Seite 1von 53

[G.R. NO.

152356 : August 16, 2005] On 27 July 1998, petitioner filed a motion to dismiss the petition
for certification election on the sole ground that herein respondent
SAN MIGUEL CORPORATION (MANDAUE PACKAGING is not listed or included in the roster of legitimate labor
PRODUCTS PLANTS), Petitioners, v. MANDAUE organizations based on the certification issued by the Officer-In-
PACKING PRODUCTS PLANTS-SAN PACKAGING Charge, Regional Director of the DOLE Regional Office No. VII,
PRODUCTS - SAN MIGUEL CORPORATION Atty. Jesus B. Gabor, on 24 July 1998.
MONTHLIES RANK-AND-FILE UNION - FFW (MPPP-
SMPP-SMAMRFU-FFW), Respondent. On 29 July 1998, respondent submitted to the Bureau of Labor
Relations the same documents earlier attached to its petition for
DECISION certification. The accompanying letter, signed by respondent's
president Sagun, stated that such documents were submitted in
compliance with the requirements for the creation of a
TINGA, J.:
local/chapter pursuant to the Labor Code and its Implementing
Rules; and it was hoped that the submissions would facilitate the
The central question in this Petition for Review is on what date listing of respondent under the roster of legitimate labor
did respondent Mandaue Packing Products Plants-San Miguel organizations.5 On 3 August 1998, the Chief of Labor Relations
Packaging Products'San Miguel Corporation Monthlies Rank- Division of DOLE Regional Office No. VII issued a Certificate
And-File Union FFW acquire legal personality in accordance with of Creation of Local/Chapter No. ITD. I-ARFBT-058/98,
the Implementing Rules of the Labor Code. The matter is crucial certifying that from 30 July 1998, respondent has acquired legal
since respondent filed a petition for certification election at a date personality as a labor organization/worker's association, it having
when, it is argued, it had yet to acquire the requisite legal submitted all the required documents.6
personality. The Department of Labor and Employment (DOLE)
and the Court of Appeals both ruled that respondent had acquired
Opting not to file a comment on the Motion to
legal personality on the same day it filed the petition for
Dismiss,7 respondent instead filed a Position Paper wherein it
certification election. The procedure employed by the respondent
asserted that it had complied with all the necessary requirements
did not strictly conform with the relevant provisions of law. But
for the conduct of a certification election, and that the ground
rather than insist on an overly literal reading of the law that
relied upon in the Motion to Dismiss was a mere technicality.8
senselessly suffocates the constitutionally guaranteed right to self-
organization, we uphold the assailed decisions and the liberal
spirit that animates them. In turn, petitioner filed a Comment, wherein it reiterated that
respondent was not a legitimate labor organization at the time of
the filing of the petition. Petitioner also propounded that contrary
Antecedent Facts
to respondent's objectives of establishing an organization
representing rank-and-file employees, two of respondent's
The present petition assailed the Decision dated 7 June 2001 officers, namely Vice-President Emannuel L. Rosell and
rendered by the Court of Appeals Eighth Division1 which in turn Secretary Bathan, were actually supervisory employees. In
affirmed a Decision dated 22 Feburary 1999 by the DOLE support of this allegation, petitioner attached various documents
Undersecretary for Labor Relations, Rosalinda Dimapilis-Baldoz, evidencing the designation of these two officers in supervisory
ordering the immediate conduct of a certification election among roles, as well as their exercise of various supervisory
the petitioner's rank-and-file employees, as prayed for by functions.9 Petitioner cited Article 245 of the Labor Code, which
respondent. The following facts are culled from the records. provides that supervisory employees shall not be eligible for
membership in a labor organization of the rank-and-file
On 15 June 1998, respondent, identifying itself as an affiliate of employees.10
Federation of Free Workers (FFW), filed a petition for
certification election with the DOLE Regional Office No. VII. In On 20 August 1998, petitioner filed a petition to cancel the union
the petition, respondent stated that it sought to be certified and to registration of respondent. However, this petition was denied, and
represent the permanent rank-and-file monthly paid employees of such denial was subsequently affirmed by the Court of Appeals in
the petitioner.2 The following documents were attached to the a decision that has since become final.11
petition: (1) a Charter Certificate issued by FFW on 5 June 1998
certifying that respondent as of that date was duly certified as a
In the meantime, on 15 September 1998, Med-Arbiter Manit
local or chapter of FFW; (2) a copy of the constitution of
issued an Order dismissing respondent's petition for certification
respondent prepared by its Secretary, Noel T. Bathan and attested
election. The sole ground relied upon for the dismissal was the
by its President, Wilfred V. Sagun; (3) a list of respondent's
Med-Arbiter's Opinion that as of the date of filing of the petition
officers and their respective addresses, again prepared by Bathan
on 15 June 1998, respondent did not have the legal personality to
and attested by Sagun; (4) a certification signifying that
file the said petition for certification election.12 No discussion was
respondent had just been organized and no amount had yet been
adduced on petitioner's claims that some of respondent's officers
collected from its members, signed by respondent's treasurer
were actually supervisory employees.
Chita D. Rodriguez and attested by Sagun; and (5) a list of all the
rank-and-file monthly paid employees of the Mandaue Packaging
Products Plants and Mandaue Glass Plant prepared by Bathan and Respondent promptly appealed the 15 September 1998 Order to
attested by Sagun.3 the DOLE. On 22 February 1999, DOLE Undersecretary
Rosalinda Dimapilis-Baldoz rendered a Decision reversing
the Order. Undersecretary Baldoz concluded that respondent
The petition was assigned to Mediator-Arbiter Achilles V. Manit
acquired legal personality as early as 15 June 1998, the date it
of the DOLE Regional Office No. VII, and docketed as Case No.
submitted the required documents, citing Section 3, Rule VI of
R0700-9806-RU-013.4
the New Rules Implementing the Labor Code (Implementing
Rules) which deems that a local/chapter acquires legal personality Article 234 of the Labor Code enumerates the requirements for
from the date of filing of the complete documentary requirements registration of an applicant labor organization, association, or
as mandated in the Implementing Rules. The DOLE also ruled group of unions or workers in order that such entity could acquire
that the contention that two of respondent's officers were actually legal personality and entitlement to the rights and privileges
supervisors can be threshed out in the pre-election conferences granted by law to legitimate labor organizations. These include a
where the list of qualified voters is to be determined. The registration fee of fifty pesos (P50.00); a list of the names of the
dispositive portion of the DOLE Decision stated: members and officers, and copies of the constitution and by-laws
of the applicant union.17
WHEREFORE, the appeal is GRANTED. The order dated 15
September 1999 of the Med-Arbiter is REVERSED and SET However, the Labor Code itself does not lay down the procedure
ASIDE. Accordingly, let the records of the case be remanded to for the registration of a local or chapter of a labor organization.
the office of origin for the immediate conduct of certification Such has been traditionally provided instead in the Implementing
election, subject to the usual pre-election conference, among the Rules, particularly in Book V thereof. However, in the last decade
monthly-paid rank-and-file employees of the Mandaue Packaging or so, significant amendments have been introduced to Book V,
Products Plant San Miguel Corporation, with the following first by Department Order No. 9 which took effect on 21 June
choices: 1997, and again by Department Order No. 40 dated 17 February
2003. The differences in the procedures laid down in these
1. MANDAUE PACKAGING PRODUCT PLANT SAN various versions are significant. However, since the instant
MIGUEL PACKAGING PRODUCTS SAN MIGUEL petition for certification was filed in 1998, the Implementing
CORPORATION MONTHLIES RANK AND FILE UNION Rules, as amended by Department Order No. 9, should govern the
FFW (MPPP-SMPP-SMCMRFUFFW), resolution of this petition.18

2. NO UNION. Preliminarily, we should note that a less stringent procedure


obtains in the registration of a local or chapter than that of a labor
organization. Undoubtedly, the intent of the law in imposing
Pursuant to Rule XI, Section 11.1 of the New Implementing
lesser requirements in the case of a branch or local of a registered
Rules, the company is hereby directed to submit to the office of
federation or national union is to encourage the affiliation of a
origin the certified list of current employees in the bargaining
local union with a federation or national union in order to increase
unit, along with the payrolls covering the members of the
the local union's bargaining powers respecting terms and
bargaining unit for the last three months prior to the issuance of
conditions of labor.19 This policy has remained consistent despite
this decision.
the succeeding amendments to Book V of the Omnibus
Implementing Rules, as contained in Department Orders Nos. 9
SO DECIDED.13 and 40.

These two conclusions of the DOLE were affirmed in the The case of Progressive Development Corp. v. Secretary of
assailed Decision of the Court of Appeals. It is now our task to Labor,20 applying Section 3, Rule II, Book V of the Implementing
review whether these conclusions are warranted under law and Rules, in force before 1997, ruled that "a local or chapter
jurisprudence. First, we shall discuss the aspect of respondent's therefore becomes a legitimate labor organization only upon
legal personality in filing the petition for certification election. submission of the following to the BLR: (1) a charter certificate,
within thirty (30) days from its issuance by the labor federation or
First Issue: On the Acquisition of national union; and (2) The constitution and by-laws, a statement
of the set of officers, and the books of accounts all of which are
Legal Personality by Respondent certified under oath by the secretary or treasurer, as the case may
be, of such local or chapter, and attested to by its president."21 The
submission by the local/chapter of duly certified books of
Statutory Provisions for Registration Of accounts as a prerequisite for registration of the local/chapter was
dropped in Department Order No. 9,22 a development noted by the
Local/Chapter of Federation or National Union Court in Pagpalain Haulers v. Hon. Trajano,23 wherein it was
held that the previous doctrines requiring the submission of books
Before we proceed to evaluate the particular facts of this case, it of accounts as a prerequisite for the registration of a local/chapter
would be useful to review the statutory paradigm that governs the "are already passé and therefore, no longer applicable."24
establishment and acquisition of legal personality by a
local/chapter of a labor organization. The applicable rules have Department Order No. 40, now in effect, has eased the
undergone significant amendments in the last decade, thus a requirements by which a local/chapter may acquire legal
recapitulation of the framework is in order. personality. Interestingly, Department Order No. 40 no longer
uses the term "local/chapter," utilizing instead "chartered local,"
The Labor Code defines a labor organization as any union or which is defined as a "labor organization in the private sector
association of employees which exists in whole or in part for the operating at the enterprise level that acquired legal personality
purpose of collective bargaining or of dealing with employers through the issuance of a charter certificate by a duly registered
concerning terms and conditions of employment,14 and a federation or national union, and reported to the Regional
"legitimate labor organization" as any labor organization duly Office."25 Clearly under the present rules, the first step to be
registered with the DOLE, including any branch or local undertaken in the creation of a chartered local is the issuance of a
thereof.15 Only legitimate labor organizations may file a petition charter certificate by the duly registered federation or national
for certification election.16 union. Said federation or national union is then obligated to report
to the Regional Office the creation of such chartered local, the legal personality of the federation or national union, which in
attaching thereto the charter certificate it had earlier issued.26 turn, had already undergone evaluation and approval from the
Bureau of Legal Relations or Regional Office. In fact, a
But as stated earlier, it is Department Order No. 9 that governs in federation or national union is required, upon registration, to
this case. Section 1, Rule VI thereof prescribes the documentary establish proof of affiliation of at least ten (10) locals or chapters
requirements for the creation of a local/chapter. It states: which are duly recognized as the collective bargaining agent in
the establishment or industry in which they operate; and the
names and addresses of the companies where the locals or
Section 1. Chartering and creation of a local chapter - A duly
chapters operate and the list of all the members in each of the
registered federation or national union may directly create a
companies.31 Once the national union or federation acquires legal
local/chapter by submitting to the Regional Office or to the
personality upon the issuance of its certificate or registration,32 its
Bureau two (2) copies of the following:
legal personality cannot be subject to collateral attack.33

a) A charter certificate issued by the federation or national union


The fact that the local/chapter acquires legal personality from the
indicating the creation or establishment of the local/chapter;
moment the complete documentary requirements are submitted
seems to imply that the duty of the Bureau or Regional Office to
(b) The names of the local/chapter's officers, their addresses, and register the local/chapter is merely ministerial. However,
the principal office of the local/chapter; in Progressive Development Corporation v. Laguesma,34 the
Court, in ruling against a petition for certification filed by a
(c) The local/chapter's constitution and by-laws; provided that chapter, held that the mere submission of the documentary
where the local/chapter's constitution and by-laws is the same as requirements does not render ministerial the function of the
that of the federation or national union, this fact shall be indicated Bureau of Labor Relations in according due recognition to the
accordingly. labor organization.35 Still, that case was decided before the
enactment of Department Order No. 9, including the aforestated
All the foregoing supporting requirements shall be certified under Section 3. Should we consider the said 1997 amendments as
oath by the Secretary or Treasurer of the local/chapter and having obviated our characterization in Progressive of the
attested by its President. Bureau's duty as non-ministerial?chanroblesvirtualawlibrary

In contrast, an independent union seeking registration is further Notwithstanding the amendments, it still is good policy to
required under Dept. Order No. 90 to submit the number and maintain that per Department Order No. 9, the duty of the Bureau
names of the members, and annual financial reports.27 of Labor Relations to recognize the local/chapter upon the
submission of the documentary requirements is not ministerial,
insofar as the Bureau is obliged to adjudge the authenticity of the
Section 3, Rule VI of Department Order No. 9 provides when the documents required to be submitted. For example, the Bureau is
local/chapter acquires legal personality. not mandated to accept just any purported charter certificate
matter how spurious it is in appearance. It is empowered to
Section 3. Acquisition of legal personality by local chapter. 'A ascertain whether the submitted charter certificate is genuine, and
local/chapter constituted in accordance with Section 1 of this Rule if finding that said certificate is fake, deny recognition to the
shall acquire legal personality from the date of filing of the local/chapter.
complete documents enumerated therein. Upon compliance with
all the documentary requirements, the Regional Office or Bureau However, in ascertaining whether or not to recognize and register
shall issue in favor of the local/chapter a certificate indicating that the local/chapter, the Bureau or Regional Office should not look
it is included in the roster of legitimate labor organizations. beyond the authenticity and due execution of the documentary
requirements for the creation of the local/chapter as enumerated
It is evident based on this rule that the local/chapter acquires legal under Section 1, Rule VI, Book V of Department Order No. 9.
personality from the date of the filing of the complete Since the proper submission of these documentary requirements is
documentary requirements, and not from the issuance of a all that is necessary to recognize a local/chapter, it is beyond the
certification to such effect by the Regional Office or Bureau. On province of the Bureau or Regional Offices to resort to other
the other hand, a labor organization is deemed to have acquired grounds as basis for denying legal recognition of the
legal personality only on the date of issuance of its certificate of local/chapter. For example, Department Order No. 9 does not
registration,28 which takes place only after the Bureau of Labor require the local/chapter to submit the names of its members as a
Relations or its Regional Offices has undertaken an evaluation condition precedent to its registration.36 It therefore would be
process lasting up until thirty (30) days, within which period it improper to deny legal recognition to a local/chapter owing to
approves or denies the application.29 In contrast, no such period of questions pertaining to its individual members since the
evaluation is provided in Department Order No. 9 for the local/chapter is not even obliged to submit the names of its
application of a local/chapter, and more importantly, under it such individual members prior to registration.
local/chapter is deemed to acquire legal personality "from the date
of filing" of the documents enumerated under Section 1, Rule VI, Certainly, when a local/chapter applies for registration, matters
Book V. raised against the personality of the federation or national union
itself should not be acted upon by the Bureau or Regional Office,
Apart from promoting a policy of affiliation of local unions with owing to the preclusion of collateral attack. Instead, the proper
national unions,30 there is a practical reason for sanctioning a less matter for evaluation by the Bureau or Regional Office should be
onerous procedure for the registration of a local/chapter, as limited to whether the local/chapter is indeed a duly created
compared to the national union. The local/chapter relies in part on affiliate of the national union or federation.
Parenthetically, under the present Implementing Rules as certificate, along with the names of the local/chapter's officers,
amended by Department Order No. 40, it appears that the constitution and by-laws to the Regional Office or Bureau. It is
local/chapter (or now, "chartered local") acquires legal the submission of these documents, certified under oath by the
personality upon the issuance of the charter certificate by the duly Secretary or Treasurer of the local/chapter and attested by the
registered federation or national union.37 This might signify that President, which vests legal personality in the local/chapter,
the creation of the chartered local is within the sole discretion of which is then free to file on its own a petition for certification
the federation or national union and thus beyond the review or election.
interference of the Bureau of Labor Relations or its Regional
Offices. However, Department Order No. 40 also requires that the In this case, the federation in question, the FFW, did not submit
federation or national union report the creation of the chartered any of these documentary requirements to the Regional Office or
local to the Regional Office. Bureau. It did however issue a charter certificate to the putative
local/chapter (herein respondent). Respondent then submitted the
Acquisition by Respondent of Legal Personality charter certificate along with the other documentary requirements
to the Regional Office, but not for the specific purpose of creating
We now proceed to determine if and when the respondent the local/chapter, but for filing the petition for certification
acquired legal personality under the procedure laid down by the election.
rules then in effect, Department Order No. 9, that is.
It could be properly said that at the exact moment respondent was
At the onset, the arguments raised by petitioner on this point are filing the petition for certification, it did not yet possess any legal
plainly erroneous. Petitioner cites the case of Toyota Motor personality, since the requisites for acquisition of legal
Philippines v. Toyota Motor Philippines Corporation Labor personality under Section 3, Rule VI of Department Order No. 9
Union,38 and the purported holding therein that "[if] it is true that had not yet been complied with. It could also be discerned that the
at the time of the filing of the petition, the said registration intention of the Labor Code and its Implementing Rules that only
certificate has not been approved yet, then, petitioner lacks the those labor organizations that have acquired legal personality are
legal personality to file the petition."39 However, an examination capacitated to file petitions for certification elections. Such is the
of the case actually reveals that the cited portion was lifted from general rule.
one of the antecedent rulings of the Med-Arbiter in that case
which had not even been affirmed or reinstated by the Court on Yet there are peculiar circumstances in this case that allow the
review.40 Moreover, such pronouncement made prior to the Court to rule that respondent acquired the requisite legal
enactment of Department Order No. 9 squarely contradicts personality at the same time it filed the petition for certification
Section 3, Rule VI thereof, which provides that legal personality election. In doing so, the Court acknowledges that the strict letter
of the local/chapter is vested upon the submission of the complete of the procedural rule was not complied with. However, labor
documentary requirements. laws are generally construed liberally in favor of labor, especially
if doing so affirms the constitutionally guaranteed right to self-
It is also worth noting that petitioner union in Toyota was an organization.
independent labor union, and not a local/chapter, and under
Department Order No. 9, independent labor unions, unlike True enough, there was no attempt made by the national
local/chapters, acquire legal personality only upon issuance of the federation, or the local/chapter for that matter, to submit the
certificate of registration by the Bureau or Regional Office. Still, enumerated documentary requirements to the Regional Office or
petitioner cites in its favor Section 5, Rule V of Dept. Order No. Bureau for the specific purpose of creating the local/chapter.
9, which states that "the labor organization or workers' association However, these same documents were submitted by the
shall be deemed registered and vested with legal personality on local/chapter to the Regional Office as attachments to its petition
the date of issuance of its certificate of registration." Again, the for certification election. Under Section 3, Rule VI of Department
citation is obviously misplaced, as respondent herein is a Order No. 9, it is the submission of these same documents to the
local/chapter, the acquisition of its legal personality being Regional Office or Bureau that operates to vest legal personality
governed instead by Section 3, Rule VI. on the local/chapter.

It is thus very clear that the issuance of the certificate of Thus, in order to ascertain when respondent acquired legal
registration by the Bureau or Regional Office is not the operative personality, we only need to determine on what date the Regional
act that vests legal personality upon a local/chapter under Office or Bureau received the complete documentary
Department Order No. 9. Such legal personality is acquired from requirements enumerated under Section 1, Rule VI of Department
the filing of the complete documentary requirements enumerated Order No. 9. There is no doubt that on 15 June 1998, or the date
in Section 1, Rule VI. Admittedly, the manner by which respondent filed its petition for certification election, attached
respondent was deemed to have acquired legal personality by the thereto were respondent's constitution, the names and addresses of
DOLE and the Court of Appeals was not in strict conformity with its officers, and the charter certificate issued by the national union
the provisions of Department Order No. 9. Nonetheless, are the FFW. The first two of these documents were duly certified under
deviations significant enough for the Court to achieve a different oath by respondent's secretary Bathan and attested to by president
conclusion from that made by the DOLE and the Court of Sagun.41
Appeals?chanroblesvirtualawlibrary
It may be noted though that respondent never submitted a separate
In regular order, it is the federation or national union, already in by-laws, nor does it appear that respondent ever intended to
possession of legal personality, which initiates the creation of the prepare a set thereof. Section 1(c), Rule VI, Book V of
local/chapter. It issues a charter certificate indicating the creation Department Order No. 9 provides that the submission of both a
or establishment of the local/chapter. It then submits this charter constitution and a set of by-laws is required, or at least an
indication that the local/chapter is adopting the constitution and affiliation with the larger unit or source its legal personality
by-laws of the federation or national union. A literal reading of therefrom.
the provision might indicate that the failure to submit a specific
set of by-laws is fatal to the recognition of the local/chapter. A In the ordinary course, it should have been FFW, and not
more critical analysis of this requirement though is in order, respondent, which should have submitted the subject documents
especially as it should apply to this petition. to the Regional Office. Nonetheless, there is no good reason to
deny legal personality or defer its conferral to the local/chapter if
By-laws has traditionally been defined as regulations, ordinances, it is evident at the onset that the federation or national union itself
rules or laws adopted by an association or corporation or the like has already through its own means established the local/chapter.
for its internal governance, including rules for routine matters In this case, such is evidenced by the Charter Certificate dated 9
such as calling meetings and the like.42 The importance of by-laws June 1998, issued by FFW, and attached to the petition for
to a labor organization cannot be gainsaid. Without such certification election. The Charter Certificate expressly states that
provisions governing the internal governance of the organization, respondent has been issued the said certificate "to operate as a
such as rules on meetings and quorum requirements, there would local or chapter of the [FFW]". The Charter Certificate expressly
be no apparent basis on how the union could operate. Without a acknowledges FFW's intent to establish respondent as of 9 June
set of by-laws which provides how the local/chapter arrives at its 1998.44 This being the case, we consider it permissible for
decisions or otherwise wields its attributes of legal personality, respondent to have submitted the required documents itself to the
then every action of the local/chapter may be put into legal Regional Office, and proper that respondent's legal personality be
controversy. deemed existent as of 15 June 1998, the date the complete
documents were submitted.
However, if those key by-law provisions on matters such as
quorum requirements, meetings, or on the internal governance of Second Issue: On the Alleged Presence
the local/chapter are themselves already provided for in the
constitution, then it would be feasible to overlook the requirement Of Supervisory Employees as
for by-laws. Indeed in such an event, to insist on the submission
of a separate document denominated as "By-Laws" would be an
Officers of the Respondent
undue technicality, as well as a redundancy.

The second issue hinges on a point of some controversy and


An examination of respondent's constitution reveals it sufficiently
frequent discussion in recent years. Petitioner claims error in the
comprehensive in establishing the necessary rules for its
common pronouncement in the assailed decisions that the matter
operation. Article IV establishes the requisites for membership in
concerning the two officers who are allegedly supervisory
the local/chapter. Articles V and VI name the various officers and
employees may be threshed out during pre-election conferences.
what their respective functions are. The procedure for election of
Petitioner cites the cases of Toyota Motors and Progressive
these officers, including the necessary vote requirements, is
Development Corporation-Pizza Hut v. Ledesma45 wherein the
provided for in Article IX, while Article XV delineates the
Court ruled that the question of prohibited membership of both
procedure for the impeachment of these officers. Article VII
supervisory and rank-and-file employees in the same union must
establishes the standing committees of the local/chapter and how
be inquired into anterior to the granting of an order allowing a
their members are appointed. Article VIII lays down the rules for
certification election; and that a union composed of both of these
meetings of the union, including the notice and quorum
kinds of employees does not possess the requisite personality to
requirements thereof. Article X enumerates with particularity the
file for recognition as a legitimate labor organization. It should be
rules for union dues, special assessments, fines, and other
noted though that in the more recent case of Tagaytay Highlands
payments. Article XII provides the general rule for quorum in
International Golf Club v. Tagaytay Highlands Employees
meetings of the Board of Directors and of the members of the
Union,46 the Court, notwithstanding Toyota and Progressive,
local/chapter, and cites the applicability of the Robert's Rules of
ruled that after a certificate of registration is issued to a union, its
Order43 in its meetings. And finally, Article XVI governs and
legal personality cannot be subject to collateral attack, but
institutes the requisites for the amendment of the constitution.
questioned only in an independent petition for cancellation.47
Indeed, it is difficult to see in this case what a set of by-laws
There is no need to apply any of the above cases at present
separate from the constitution for respondent could provide that is
because the question raised by petitioner on this point is already
not already provided for by the Constitution. These premises
settled law, as a result of the denial of the independent petition for
considered, there is clearly no need for a separate set of by-laws
cancellation filed by petitioner against respondent on 20 August
to be submitted by respondent.
1998. The ground relied upon therein was the alleged fraud,
misrepresentation and false statement in describing itself as a
The Court likewise sees no impediment in deeming respondent as union of rank and file employees when in fact, two of its officers,
having acquired legal personality as of 15 June 1998, the fact that Emmanuel Rosell and Noel Bathan, were occupying supervisory
it was the local/chapter itself, and not the FFW, which submitted positions.48 Said petition was denied by the Regional Director,
the documents required under Section 1, Rule VI of Department this action was affirmed by the DOLE, the Court of Appeals, and
Order No. 9. The evident rationale why the rule states that it is the the Supreme Court.49 The denial made by the Court of Appeals
federation or national union that submits said documents to the and the Supreme Court may have been based on procedural
Bureau or Regional Office is that the creation of the local/chapter grounds,50 but the prior decisions of the Regional Director and the
is the sole prerogative of the federation or national union, and not DOLE ruled squarely on the same issue now raised by the
of any other entity. Certainly, a putative local/chapter cannot, petitioner. We quote from the Resolution of the DOLE dated 29
without the imprimatur of the federation or national union, claim December 1998:
. . . . [The] substantive issue that is now before us is whether or provision of appellee union's constitution and by-laws. What is
not the inclusion of the two alleged supervisory employees in important is that there is an unmistakeable intent of the members
appellee union's membership amounts to fraud, misrepresentation, of appellee union to exercise their right to organize. We cannot
or false statement within the meaning of Article 239(a) and (c) of impose rigorous restraints on such right if we are to give meaning
the Labor Code. to the protection to labor and social justice clauses of the
Constitution.51
We rule in the negative.
The above-cited pronouncement by Bureau of Labor Relations
Under the law, a managerial employee is "one who is vested with Director Benedicto Ernesto R. Bitonio, Jr. in BLR-A-C-41-11-11-
powers or prerogatives to lay down and execute management 98 was affirmed by the Court of Appeals and the Supreme Court.
policies and/or to hire, transfer, suspend, layoff, recall, discharge, Hence, its pronouncement affirming, notwithstanding the
assign or discipline employees." A supervisory employee is "one questions on the employment status of Rossell and Bathan, the
who, in the interest of the employer, effectively recommends legitimacy of the respondent, stands as a final ruling beyond the
managerial actions if the exercise of such recommendatory ambit of review, thus warranting the Court's respect. There may
authority is not merely routinary or clerical in nature but requires be a difference between this case, which involves a petition for
the use of independent judgment. '" Finally, "all employees not certification election, and the other case, which concerns a
falling within the definition of managerial or supervisory petition for cancellation. However, petitioner opposes the petition
employee are considered rank-and-file employees". It is also well- for certification election on the ground of the illegitimacy of
settled that the actual functions of an employee, not merely his respondent, owing to the alleged supervisory nature of the duties
job title, are determinative in classifying such employee as of Rossell and Bathan. That matter has already been settled in the
managerial, supervisory or rank and file. final disposition of the petition for cancellation, and thus cannot
be unsettled by reason of this present petition.
In the case of Emmanuel Rossell, appellant's evidence shows that
he undertakes the filling out of evaluation reports on the Effect of Respondent's Manifestation
performance of mechanics, which in turn are used as basis for
reclassification. Given a ready and standard form to accomplish, Of Subsequent Developments
coupled with the nature of the evaluation, it would appear that his
functions are more routinary than recommendatory and hardly A final note. In its Memorandum, petitioner alleges that the
leave room for independent judgment. In the case of Noel Bathan, bargaining unit that respondent sought to represent is no longer
appellant's evidence does not show his job title although it shows the same because of the dynamic nature of petitioner's business, a
that his recommendations on disciplinary actions appear to have lot of changes having occurred in the work environment, and that
carried some weight on higher management. On this limited four of respondent's officers are no longer connected with
point, he may qualify as a supervisory employee within the petitioner.52 Assuming that these manifestations are true, they
meaning of the law. This may, however, be outweighed by his have no effect on the Court's ruling that a certification election
other functions which are not specified in the evidence. should be immediately conducted with respondent as one of the
available choices. Petitioner's bare manifestations adduce no
Assuming that Bathan is a supervisory employee, this does not reason why the certification election should not be conducted
prove the existence of fraud, false statement or misrepresentation. forthwith. If there are matters that have arisen since the filing of
Because good faith is presumed in all representations, an essential the petition that serve to delay or cancel the election, these can be
element of fraud, false statement and misrepresentation in order threshed out during the pre-election conferences. Neither is the
for these to be actionable is intent to mislead by the party making fact that some of respondent's officers have since resigned from
the representation. In this case, there is no proof to show that petitioner of any moment. The local/chapter retains a separate
Bathan, or appellee union for that matter, intended to mislead legal personality from that of its officers or members that remains
anyone. If this was appellee union's intention, it would have viable notwithstanding any turnover in its officers or members.
refrained from using a more precise description of the
organization instead of declaring that the organization is WHEREFORE, the Petition is DENIED. Costs against
composed of 'rank and file monthlies'. Hence, the charge of fraud, petitioner.
false statement or misrepresentation cannot be sustained.
SO ORDERED.
Appellant's reliance on the Toyota case must be tempered by the
peculiar circumstances of the case. Even assuming that Bathan, or
Rossel for that matter, are supervisory employees,
the Toyota case cannot certainly be given an interpretation that
emasculates the right to self-organization and the promotion of SAMAHANG MANGGAGAWA SA CHARTER
free trade unionism. We take administrative notice of the realities CHEMICAL SOLIDARITY OF UNIONS IN THE
in union organizing, during which the organizers must take their PHILIPPINES FOR EMPOWERMENT AND REFORMS
chances, oftentimes unaware of the fine distinctions between (SMCC-SUPER), ZACARRIAS JERRY VICTORIO -
managerial, supervisory and rank and file employees. The UNION PRESIDENT, PETITIONER,VS. CHARTER
grounds for cancellation of union registration are not meant to be CHEMICAL AND COATING CORPORATION,
applied automatically, but indeed with utmost discretion. Where a RESPONDENT.
remedy short of cancellation is available, that remedy should be
preferred. In this case, no party will be prejudiced if Bathan were DECISION
to be excluded from membership in the union. The vacancy he
will thus create can then be easily filled up through the succession DEL CASTILLO, J.:
be verified and that there was no independent evidence presented
The right to file a petition for certification election is accorded to to establish respondent company's claim that some members of
a labor organization provided that it complies with the petitioner union were holding supervisory positions, the DOLE
requirements of law for proper registration. The inclusion of sustained the dismissal of the petition for certification after it took
supervisory employees in a labor organization seeking to judicial notice that another union, i.e., Pinag-isang
represent the bargaining unit of rank-and-file employees does not Lakas Manggagawa sa Charter Chemical and Coating
divest it of its status as a legitimate labor organization. We apply Corporation, previously filed a petition for certification election
these principles to this case. on January 16, 1998. The Decision granting the said petition
became final and executory on September 16, 1998 and was
This Petition for Review on Certiorari seeks to reverse and set remanded for immediate implementation. Under Section 7, Rule
aside the Court of Appeal's March 15, 2005 Decision[1] in CA- XI of D.O. No. 9, series of 1997, a motion for intervention
G.R. SP No. 58203, which annulled and set aside the January 13, involving a certification election in an unorganized establishment
2000 Decision[2] of the Department of Labor and Employment should be filed prior to the finality of the decision calling for a
(DOLE) in OS-A-6-53-99 (NCR-OD-M-9902-019) and the certification election. Considering that petitioner union filed its
September 16, 2005 Resolution[3] denying petitioner union's petition only on February 14, 1999, the same was filed out of
motion for reconsideration. time.

Factual Antecedents On motion for reconsideration, however, the DOLE reversed its
earlier ruling. In its January 13, 2000 Decision, the DOLE found
On February 19, 1999, Samahang Manggagawa sa Charter that a review of the records indicates that no certification election
Chemical Solidarity of Unions in the Philippines for was previously conducted in respondent company. On the
Empowerment and Reforms (petitioner union) filed a petition for contrary, the prior certification election filed by Pinag-isang
certification election among the regular rank-and-file employees Lakas Manggagawa  sa Charter Chemical and Coating
of Charter Chemical and Coating Corporation (respondent Corporation was, likewise, denied by the Med-Arbiter and, on
company) with the Mediation Arbitration Unit of the DOLE, appeal, was dismissed by the DOLE for being filed out of time.
National Capital Region. Hence, there was no obstacle to the grant of petitioner union's
petition for certification election, viz:
On April 14, 1999, respondent company filed an Answer with
Motion to Dismiss[4] on the ground that petitioner union is not a WHEREFORE, the motion for reconsideration is
legitimate labor organization because of (1) failure to comply hereby GRANTED and the decision of this Office dated 16 July
with the documentation requirements set by law, and (2) the 1999 is MODIFIED to allow the certification election among the
inclusion of supervisory employees within petitioner union.[5] regular rank-and-file employees of Charter Chemical and Coating
Corporation with the following choices:
Med-Arbiter's Ruling
1. Samahang Manggagawa sa Charter Chemical-Solidarity of
On April 30, 1999, Med-Arbiter Tomas F. Falconitin issued a Unions in the Philippines for Empowerment and Reform (SMCC-
Decision[6] dismissing the petition for certification election.  The SUPER); and
Med-Arbiter ruled that petitioner union is not a legitimate labor
organization because the Charter Certificate, "Sama-samang 2. No Union.
Pahayag ng Pagsapi at Authorization," and "Listahan ng mga
Dumalo sa Pangkalahatang Pulong at mga Sumang-ayon at Let the records of this case be remanded to the Regional Office of
Nagratipika sa Saligang Batas" were not executed under oath and origin for the immediate conduct of a certification election,
certified by the union secretary and attested to by the union subject to the usual pre-election conference.
president as required by Section 235 of the Labor Code[7] in
relation to Section 1, Rule VI of Department Order (D.O.) No. 9, SO DECIDED.[9]
series of 1997. The union registration was, thus, fatally defective.
Court of Appeal's Ruling
The Med-Arbiter further held that the list of membership of
petitioner union consisted of 12 batchman, mill operator and   On March 15, 2005, the CA promulgated the assailed
leadman who performed supervisory functions. Under Article 245 Decision, viz:
of the Labor Code, said supervisory employees are prohibited
from joining petitioner union which seeks to represent the rank- WHEREFORE, the petition is hereby GRANTED. The assailed
and-file employees of respondent company. Decision and Resolution dated January 13, 2000 and February 17,
2000 are hereby [ANNULLED] and SET ASIDE.
As a result, not being a legitimate labor organization, petitioner
union has no right to file a petition for certification election for SO ORDERED.[10]
the purpose of collective bargaining.
In nullifying the decision of the DOLE, the appellate court gave
Department of Labor and Employment's Ruling credence to the findings of the Med-Arbiter that petitioner union
failed to comply with the documentation requirements under the
On July 16, 1999, the DOLE initially issued a Decision[8] in favor Labor Code. It, likewise, upheld the Med-Arbiter's finding that
of respondent company dismissing petitioner union's appeal on petitioner union consisted of both rank-and-file and supervisory
the ground that the latter's petition for certification election was employees.  Moreover, the CA held that the issues as to the
filed out of time. Although the DOLE ruled, contrary to the legitimacy of petitioner union may be attacked collaterally in a
findings of the Med-Arbiter, that the charter certificate need not petition for certification election and the infirmity in the
membership of petitioner union cannot be remedied through the collaterally attacked but may be questioned only in an
exclusion-inclusion proceedings in a pre-election conference independent petition for cancellation pursuant to Section 5, Rule
pursuant to the ruling in Toyota Motor Philippines v. Toyota V, Book IV of the Rules to Implement the Labor Code and the
Motor Philippines Corporation Labor Union.[11] Thus, doctrine enunciated in Tagaytay Highlands International Golf
considering that petitioner union is not a legitimate labor Club Incoprorated v. Tagaytay Highlands Empoyees Union-
organization, it has no legal right to file a petition for certification PTGWO.[13]
election.
Respondent Company's Arguments
Issues
Respondent company asserts that it cannot be precluded from
I challenging the July 16, 1999 Decision of the DOLE. The said
decision did not attain finality because the DOLE subsequently
Whether x x x the Honorable Court of Appeals committed grave reversed its earlier ruling and, from this decision, respondent
abuse of discretion tantamount to lack of jurisdiction in granting company timely filed its motion for reconsideration.
the respondent [company's] petition for certiorari (CA G.R. No.
SP No. 58203) in spite of the fact that the issues subject of the On the issue of lack of verification of the charter certificate,
respondent company['s] petition was already settled with finality respondent company notes that Article 235 of the Labor Code and
and barred from being re-litigated. Section 1, Rule VI of the Implementing Rules of Book V, as
amended by D.O. No. 9, series of 1997, expressly requires that
II the charter certificate be certified under oath.

Whether x x x the Honorable Court of Appeals committed grave It also contends that petitioner union is not a legitimate labor
abuse of discretion tantamount to lack of jurisdiction in holding organization because its composition is a mixture of supervisory
that the alleged mixture of rank-and-file and supervisory and rank-and-file employees in violation of Article 245 of the
employee[s] of petitioner [union's] membership is [a] ground for Labor Code. Respondent company maintains that the ruling
the cancellation of petitioner [union's] legal personality and in Toyota Motor Philippines vs. Toyota Motor Philippines Labor
dismissal of [the] petition for certification election. Union[14] continues to be good case law. Thus, the illegal
composition of petitioner union nullifies its legal personality to
III file the subject petition for certification election and its legal
personality may be collaterally attacked in the proceedings for a
Whether x x x the Honorable Court of Appeals committed grave petition for certification election as was done here.
abuse of discretion tantamount to lack of jurisdiction in holding
that the alleged failure to certify under oath the local charter Our Ruling
certificate issued by its mother federation and list of the union
membership attending the organizational meeting [is a ground] The petition is meritorious.
for the cancellation of petitioner [union's] legal personality as a
labor organization and for the dismissal of the petition for The issue as to the legal personality of
certification election.[12] petitioner union is not barred by the
July 16, 1999 Decision of the DOLE.
Petitioner Union's Arguments
A review of the records indicates that the issue as to petitioner
Petitioner union claims that the litigation of the issue as to its union's legal personality has been timely and consistently raised
legal personality to file the subject petition for certification by respondent company before the Med-Arbiter, DOLE, CA and
election is barred by the July 16, 1999 Decision of the DOLE. In now this Court. In its July 16, 1999 Decision, the DOLE found
this decision, the DOLE ruled that petitioner union complied with that petitioner union complied with the documentation
all the documentation requirements and that there was no requirements of the Labor Code and that the evidence was
independent evidence presented to prove an illegal mixture of insufficient to establish that there was an illegal mixture of
supervisory and rank-and-file employees in petitioner union. supervisory and rank-and-file employees in its membership.
After the promulgation of this Decision, respondent company did Nonetheless, the petition for certification election was dismissed
not move for reconsideration, thus, this issue must be deemed on the ground that another union had previously filed a petition
settled. for certification election  seeking  to  represent the  same 
bargaining unit  in respondent company.
Petitioner union further argues that the lack of verification of its
charter certificate and the alleged illegal composition of its Upon motion for reconsideration by petitioner union on January
membership are not grounds for the dismissal of a petition for 13, 2000, the DOLE reversed its previous ruling. It upheld the
certification election under Section 11, Rule XI of D.O. No. 9, right of petitioner union to file the subject petition for certification
series of 1997, as amended, nor are they grounds for the election because its previous decision was based on a mistaken
cancellation of a union's registration under Section 3, Rule VIII of appreciation of facts.[15] From this adverse decision, respondent
said issuance. It contends that what is required to be certified company timely moved for reconsideration by reiterating its
under oath by the local union's secretary or treasurer and attested previous arguments before the Med-Arbiter that petitioner union
to by the local union's president are limited to the union's has no legal personality to file the subject petition for certification
constitution and by-laws, statement of the set of officers, and the election.
books of accounts.
The July 16, 1999 Decision of the DOLE, therefore, never
Finally, the legal personality of petitioner union cannot be attained finality because the parties timely moved for
reconsideration. The issue then as to the legal personality of Miguel Corporation (Mandaue Packaging Products Plants) v.
petitioner union to file the certification election was properly Mandaue Packing Products Plants-San Miguel Corporation
raised before the DOLE, the appellate court and now this Court. Monthlies Rank-and-File Union-FFW (MPPP-SMPP-
SMAMRFU-FFW),[22] which was decided under the auspices of
The charter certificate need not be D.O. No. 9, Series of 1997, we ruled -
certified under oath by the local union's
secretary or treasurer and attested to In San Miguel Foods-Cebu B-Meg Feed Plant v. Hon. Laguesma,
by its president. 331 Phil. 356 (1996), the Court ruled that it was not
necessary for the charter certificate to be certified and attested by
Preliminarily, we must note that Congress enacted Republic Act the local/chapter officers. Id. While this ruling was based on the
(R.A.) No. 9481[16] which took effect on June 14, 2007.[17] This interpretation of the previous Implementing Rules provisions
law introduced substantial amendments to the Labor Code. which were supplanted by the 1997 amendments, we believe
However, since the operative facts in this case occurred in 1999, that the same doctrine obtains in this case. Considering that the
we shall decide the issues under the pertinent legal provisions charter certificate is prepared and issued by the national union and
then in force (i.e., R.A. No. 6715,[18] amending Book V of the not the local/chapter, it does not make sense to have the
Labor Code, and the rules and regulations[19] implementing R.A. local/chapter's officers x x x certify or attest to a document
No. 6715, as amended by D.O. No. 9,[20] which they had no hand in the preparation of.[23] (Emphasis
supplied)
series of 1997) pursuant to our ruling in Republic v. Kawashima
Textile Mfg., Philippines, Inc.[21] In accordance with this ruling, petitioner union's charter
certificate need not be executed under oath. Consequently, it
In the main, the CA ruled that petitioner union failed to comply validly acquired the status of a legitimate labor organization upon
with the requisite documents for registration under Article 235 of submission of (1) its charter certificate,[24] (2) the names of its
the Labor Code and its implementing rules. It agreed with the officers, their addresses, and its principal office,[25] and (3) its
Med-Arbiter that the Charter Certificate, Sama-samang Pahayag constitution and by-laws[26]-- the last two requirements having
ng Pagsapi at Authorization, and Listahan ng mga Dumalo sa been executed under oath by the proper union officials as borne
Pangkalahatang Pulong at mga Sumang-ayon at Nagratipika sa out by the records.
Saligang Batas were not executed under oath. Thus, petitioner
union cannot be accorded the status of a legitimate labor The mixture of rank-and-file and supervisory
organization. employees in petitioner union does not
nullify its legal personality as a legitimate
We disagree. labor organization.

The then prevailing Section 1, Rule VI of the Implementing Rules The CA found that petitioner union has for its membership both
of Book V, as amended by D.O. No. 9, series of 1997, provides: rank-and-file and supervisory employees. However, petitioner
union sought to represent the bargaining unit consisting of rank-
Section 1. Chartering and creation of a local chapter -- A duly and-file employees. Under Article 245[27] of the Labor Code,
registered federation or national union may directly create a supervisory employees are not eligible for membership in a labor
local/chapter by submitting to the Regional Office or to the organization of rank-and-file employees. Thus, the appellate court
Bureau two (2) copies of the following: ruled that petitioner union cannot be considered a legitimate labor
organization pursuant to Toyota Motor Philippines v. Toyota
(a)  A charter certificate issued by the federation or national union Motor Philippines Corporation Labor
indicating the creation or establishment of the local/chapter; Union[28] (hereinafter Toyota).

(b)  The names of the local/chapter's officers, their addresses, and Preliminarily, we note that petitioner union questions the factual
the principal office of the local/chapter; and findings of the Med-Arbiter, as upheld by the appellate court, that
12 of its members, consisting of batchman, mill operator and
(c)  The local/chapter's constitution and by-laws provided that leadman, are supervisory employees. However, petitioner union
where the local/chapter's constitution and by-laws [are] the same failed to present any rebuttal evidence in the proceedings below
as [those] of the federation or national union, this fact shall be after respondent company submitted in evidence the job
indicated accordingly. descriptions[29] of the aforesaid employees. The job descriptions
indicate that the aforesaid employees exercise recommendatory
All the foregoing supporting requirements shall be certified under managerial actions which are not merely routinary but require the
oath by the Secretary or the Treasurer of the local/chapter and use of independent judgment, hence, falling within the definition
attested to by its President. of supervisory employees under Article 212(m)[30] of the Labor
Code. For this reason, we are constrained to agree with the Med-
As readily seen, the Sama-samang Pahayag ng Pagsapi Arbiter, as upheld by the appellate court, that petitioner union
at Authorization and Listahan ng mga Dumalo sa consisted of both rank-and-file and supervisory employees.
Pangkalahatang Pulong at mga Sumang-ayon at Nagratipika sa
Saligang Batas are not among the documents that need to be Nonetheless, the inclusion of the aforesaid supervisory employees
submitted to the Regional Office or Bureau of Labor Relations in in petitioner union does not divest it of its status as a legitimate
order to register a labor organization. As to the charter certificate, labor organization. The appellate court's reliance on Toyota is
the above-quoted rule indicates that it should be executed under misplaced in view of this Court's subsequent ruling in Republic v.
oath. Petitioner union concedes and the records confirm that its Kawashima Textile Mfg., Philippines, Inc.
[31]
charter certificate was not executed under oath. However, in San  (hereinafter Kawashima).  In Kawashima, we explained at
length how and why the Toyota doctrine no longer holds sway inquire into the composition of any labor organization
under the altered state of the law and rules applicable to this whenever the status of the labor organization is challenged on
case, viz: the basis of Article 245 of the Labor Code.

R.A. No. 6715 omitted specifying the exact effect any violation
of the prohibition [on the co-mingling of supervisory and xxxx
rank-and-file employees] would bring about on the legitimacy
of a labor organization. In the case at bar, as respondent union's membership list contains
the names of at least twenty-seven (27) supervisory employees in
It was the Rules and Regulations Implementing R.A. No. 6715 Level Five positions, the union could not, prior to purging itself
(1989 Amended Omnibus Rules) which supplied the deficiency of its supervisory employee members, attain the status of a
by introducing the following amendment to Rule II (Registration legitimate labor organization. Not being one, it cannot possess the
of Unions): requisite personality to file a petition for certification election."
(Emphasis supplied)
"Sec. 1. Who may join unions. - x x x Supervisory employees
and security guards shall not be eligible for membership in a In Dunlop, in which the labor organization that filed a petition for
labor organization of the rank-and-file employees but may certification election was one for supervisory employees, but in
join, assist or form separate labor organizations of their own; which the membership included rank-and-file employees, the
Provided, that those supervisory employees who are included in Court reiterated that such labor organization had no legal right to
an existing rank-and-file bargaining unit, upon the effectivity of file a certification election to represent a bargaining unit
Republic Act No. 6715, shall remain in that unit x x x. (Emphasis composed of supervisors for as long as it counted rank-and-file
supplied) employees among its members.

It should be emphasized that the petitions for certification election


and Rule V (Representation Cases and Internal-Union Conflicts) involved in Toyota and Dunlop were filed on November 26, 1992
of the Omnibus Rules, viz: and September 15, 1995, respectively; hence, the 1989 Rules was
applied in both cases.
"Sec. 1. Where to file. - A petition for certification election may
be filed with the Regional Office which has jurisdiction over the But then, on June 21, 1997, the 1989 Amended Omnibus Rules
principal office of the employer. The petition shall be in writing was further amended by Department Order No. 9, series of 1997
and under oath. (1997 Amended Omnibus Rules). Specifically, the requirement
under Sec. 2(c) of the 1989 Amended Omnibus Rules - that the
Sec. 2. Who may file. - Any legitimate labor organization or the petition for certification election indicate that the bargaining unit
employer, when requested to bargain collectively, may file the of rank-and-file employees has not been mingled with supervisory
petition. employees - was removed. Instead, what the 1997 Amended
Omnibus Rules requires is a plain description of the bargaining
The petition, when filed by a legitimate labor organization, shall unit, thus:
contain, among others:
Rule XI
xxxx Certification Elections

(c) description of the bargaining unit which shall be the xxxx


employer unit unless circumstances otherwise require; and
provided further, that the appropriate bargaining unit of the Sec. 4. Forms and contents of petition. - The petition shall be in
rank-and-file employees shall not include supervisory writing and under oath and shall contain, among others, the
employees and/or security guards. (Emphasis supplied) following: x x x (c) The description of the bargaining unit.

By that provision, any questioned mingling will prevent an In Pagpalain Haulers, Inc. v. Trajano, the Court had occasion to
otherwise legitimate and duly registered labor organization from uphold the validity of the 1997 Amended Omnibus Rules,
exercising its right to file a petition for certification election. although the specific provision involved therein was only Sec. 1,
Rule VI, to wit:
Thus, when the issue of the effect of mingling was brought to the
fore in Toyota, the Court, citing Article 245 of the Labor Code, as "Section. 1. Chartering and creation of a local/chapter.- A duly
amended by R.A. No. 6715, held: registered federation or national union may directly create a
local/chapter by submitting to the Regional Office or to the
"Clearly, based on this provision, a labor organization composed Bureau two (2) copies of the following: a) a charter certificate
of both rank-and-file and supervisory employees is no labor issued by the federation or national union indicating the creation
organization at all. It cannot, for any guise or purpose, be a or establishment of the local/chapter; (b) the names of the
legitimate labor organization. Not being one, an organization local/chapter's officers, their addresses, and the principal office of
which carries a mixture of rank-and-file and supervisory the local/chapter; and (c) the local/ chapter's constitution and by-
employees cannot possess any of the rights of a legitimate laws; provided that where the local/chapter's constitution and by-
labor organization, including the right to file a petition for laws is the same as that of the federation or national union, this
certification election for the purpose of collective fact shall be indicated accordingly.
bargaining. It becomes necessary, therefore, anterior to the
granting of an order allowing a certification election, to All the foregoing supporting requirements shall be certified under
oath by the Secretary or the Treasurer of the local/chapter and be collaterally attacked in the certification election proceedings.
attested to by its President." As we explained in Kawashima:

which does not require that, for its creation and registration, a Except when it is requested to bargain collectively, an employer
local or chapter submit a list of its members. is a mere bystander to any petition for certification election; such
proceeding is non-adversarial and merely investigative, for the
Then came Tagaytay Highlands Int'l. Golf Club, Inc. v. Tagaytay purpose thereof is to determine which organization will represent
Highlands Employees Union-PGTWO in which the core issue was the employees in their collective bargaining with the employer.
whether mingling affects the legitimacy of a labor organization The choice of their representative is the exclusive concern of the
and its right to file a petition for certification election. This time, employees; the employer cannot have any partisan interest
given the altered legal milieu, the Court abandoned the view therein; it cannot interfere with, much less oppose, the process by
in Toyota and Dunlop and reverted to its pronouncement filing a motion to dismiss or an appeal from it; not even a mere
in Lopez that while there is a prohibition against the mingling of allegation that some employees participating in a petition for
supervisory and rank-and-file employees in one labor certification election are actually managerial employees will lend
organization, the Labor Code does not provide for the effects an employer legal personality to block the certification election.
thereof. Thus, the Court held that after a labor organization has The employer's only right in the proceeding is to be notified or
been registered, it may exercise all the rights and privileges of a informed thereof.
legitimate labor organization. Any mingling between supervisory
and rank-and-file employees in its membership cannot affect its The amendments to the Labor Code and its implementing rules
legitimacy for that is not among the grounds for cancellation of its have buttressed that policy even more.[33]
registration, unless such mingling was brought about by
misrepresentation, false statement or fraud under Article 239 of WHEREFORE, the petition is GRANTED. The March 15, 2005
the Labor Code. Decision and September 16, 2005 Resolution of the Court of
Appeals in CA-G.R. SP No. 58203 are REVERSED and SET
In San Miguel Corp. (Mandaue Packaging Products Plants) v. ASIDE. The January 13, 2000 Decision of the Department of
Mandaue Packing Products Plants-San Miguel Packaging Labor and Employment in OS-A-6-53-99 (NCR-OD-M-9902-
Products-San Miguel Corp. Monthlies Rank-and-File Union- 019) is REINSTATED.
FFW, the Court explained that since the 1997 Amended Omnibus
Rules does not require a local or chapter to provide a list of its No pronouncement as to costs.
members, it would be improper for the DOLE to deny recognition
to said local or chapter on account of any question pertaining to SO ORDERED.
its individual members.

More to the point is Air Philippines Corporation v. Bureau of


Labor Relations, which involved a petition for cancellation of
union registration filed by the employer in 1999 against a rank-
G.R. No. 183317               December 21, 2009
and-file labor organization on the ground of mixed
membership: the Court therein reiterated its ruling in Tagaytay
Highlands that the inclusion in a union of disqualified employees MARIWASA SIAM CERAMICS, INC., Petitioner,
is not among the grounds for cancellation, unless such inclusion is vs.
due to misrepresentation, false statement or fraud under the THE SECRETARY OF THE DEPARTMENT OF LABOR
circumstances enumerated in Sections (a) and (c) of Article 239 AND EMPLOYMENT, CHIEF OF THE BUREAU OF
of the Labor Code. LABOR RELATIONS, DEPARTMENT OF LABOR AND
EMPLOYMENT, REGIONAL DIRECTOR OF DOLE
All said, while the latest issuance is R.A. No. 9481, the 1997 REGIONAL OFFICE NUMBER IV-A & SAMAHAN NG
Amended Omnibus Rules, as interpreted by the Court MGA MANGGAGAWA SA MARIWASA SIAM
in Tagaytay Highlands, San Miguel and Air Philippines, had CERAMICS, INC. (SMMSC-INDEPENDENT), Respondents.
already set the tone for it. Toyota and Dunlop no longer hold
sway in the present altered state of the law and the rules. DECISION
[32]
 [Underline supplied]
NACHURA, J.:
The applicable law and rules in the instant case are the same as
those in Kawashima because the present petition for certification
election was filed in 1999 when D.O. No. 9, series of 1997, was This is a petition for review on certiorari1 under Rule 45 of the
still in effect. Hence, Kawashima applies with equal force here. Rules of Court, seeking to annul the Decision2 dated December
As a result, petitioner union was not divested of its status as a 20, 2007 and the Resolution3 dated June 6, 2008 of the Court of
legitimate labor organization even if some of its members were Appeals in CA-G.R. SP No. 98332.
supervisory employees; it had the right to file the subject petition
for certification election. The antecedent facts are as follows—

The legal personality of petitioner union On May 4, 2005, respondent Samahan Ng Mga Manggagawa Sa
cannot be collaterally attacked by respondent Mariwasa Siam Ceramics, Inc. (SMMSC-Independent) was
company in the certification election proceedings. issued a Certificate of Registration4 as a legitimate labor
organization by the Department of Labor and Employment
Petitioner union correctly argues that its legal personality cannot (DOLE), Region IV-A.
On June 14, 2005, petitioner Mariwasa Siam Ceramics, Inc. filed total number of union members of 102 employees who executed
a Petition for Cancellation of Union Registration against affidavits recanting their union membership.
respondent, claiming that the latter violated Article 2345 of the
Labor Code for not complying with the 20% requirement, and It is, thus, imperative that we peruse the affidavits appearing to
that it committed massive fraud and misrepresentation in violation have been executed by these affiants.
of Article 2396 of the same code. The case was docketed as Case
No. RO400-0506-AU-004.
The affidavits uniformly state—
On August 26, 2005, the Regional Director of DOLE IV-A issued
Ako, _____________, Pilipino, may sapat na gulang, regular na
an Order granting the petition, revoking the registration of
empleyado bilang Rank & File sa Mariwasa Siam Ceramics, Inc.,
respondent, and delisting it from the roster of active labor unions.
Bo. San Antonio, Sto. Tomas, Batangas, matapos na
makapanumpa ng naaayon sa batas ay malaya at kusang loob na
Aggrieved, respondent appealed to the Bureau of Labor Relations nagsasaad ng mga sumusunod:
(BLR).
1. Ako ay napilitan at nilinlang sa pagsapi sa Samahan
In a Decision7 dated June 14, 2006, the BLR granted respondent’s ng mga Manggagawa sa Mariwasa Siam Ceramics, Inc.
appeal and disposed as follows— o SMMSC-Independent sa kabila ng aking pag-
aalinlangan[;]
WHEREFORE, premises considered, the appeal by Samahan ng
Manggagawa sa Mariwasa Siam Ceramics, Inc. (SMMSC- 2. Aking lubos na pinagsisihan ang aking pagpirma sa
Independent) is hereby GRANTED, and the Decision dated 26 sipi ng samahan, at handa ako[ng] tumalikod sa
August 2005 by DOLE-Region-IV-A Director Maximo B. Lim is anumang kasulatan na aking nalagdaan sa kadahilanan
hereby REVERSED and SET ASIDE. Samahan ng Manggagawa na hindi angkop sa aking pananaw ang mga mungkahi o
sa Mariwasa Siam Ceramics, Inc. (SMMSC-Independent), under adhikain ng samahan.
Registration Certificate No. RO400-200505-UR-002, remains in
the roster of legitimate labor organizations.
SA KATUNAYAN NANG LAHAT, ako ay lumagda ng aking
pangalan ngayong ika-____ ng ______, 2005 dito sa Lalawigan
SO DECIDED.8 ng Batangas, Bayan ng Sto. Tomas.

Petitioner filed a Motion for Reconsideration but the BLR denied ____________________
it in a Resolution9 dated February 2, 2007. Nagsasalaysay

Petitioner sought recourse with the Court of Appeals (CA) Evidently, these affidavits were written and prepared in advance,
through a Petition for Certiorari; but the CA denied the petition and the pro forma affidavits were ready to be filled out with the
for lack of merit. employees’ names and signatures.

Petitioner’s motion for reconsideration of the CA Decision was The first common allegation in the affidavits is a declaration that,
likewise denied, hence, this petition based on the following in spite of his hesitation, the affiant was forced and deceived into
grounds— joining the respondent union. It is worthy to note, however, that
the affidavit does not mention the identity of the people who
Review of the Factual Findings of the Bureau of Labor Relations, allegedly forced and deceived the affiant into joining the union,
adopted and confirmed by the Honorable Court of Appeals is much less the circumstances that constituted such force and
warranted[;] deceit. Indeed, not only was this allegation couched in very
general terms and sweeping in nature, but more importantly, it
The Honorable Court of Appeals seriously erred in ruling that the was not supported by any evidence whatsoever.
affidavits of recantation cannot be given credence[;]
The second allegation ostensibly bares the affiant’s regret for
The Honorable Court of Appeals seriously erred in ruling that joining respondent union and expresses the desire to abandon or
private respondent union complied with the 20% membership renege from whatever agreement he may have signed regarding
requirement[; and] his membership with respondent.

The Honorable Court of Appeals seriously erred when it ruled Simply put, through these affidavits, it is made to appear that the
that private respondent union did not commit misrepresentation, affiants recanted their support of respondent’s application for
fraud or false statement.10 registration.

The petition should be denied. In appreciating affidavits of recantation such as these, our ruling
in La Suerte Cigar and Cigarette Factory v. Director of the Bureau
of Labor Relations11 is enlightening, viz.—
The petitioner insists that respondent failed to comply with the
20% union membership requirement for its registration as a
legitimate labor organization because of the disaffiliation from the On the second issue—whether or not the withdrawal of 31 union
members from NATU affected the petition for certification
election insofar as the 30% requirement is concerned, We reserve
the Order of the respondent Director of the Bureau of Labor solely on the recantation. It is imperative that a determination be
Relations, it appearing undisputably that the 31 union members first made as to which between the original and the new
had withdrawn their support to the petition before the filing of statements should be given weight or accorded belief, applying
said petition. It would be otherwise if the withdrawal was made the general rules on evidence. In this case, inasmuch as they
after the filing of the petition for it would then be presumed that remain bare allegations, the purported recantations should not be
the withdrawal was not free and voluntary. The presumption upheld.13
would arise that the withdrawal was procured through duress,
coercion or for valuable consideration. In other words, the Nevertheless, even assuming the veracity of the affidavits of
distinction must be that withdrawals made before the filing of the recantation, the legitimacy of respondent as a labor organization
petition are presumed voluntary unless there is convincing proof must be affirmed. While it is true that the withdrawal of support
to the contrary, whereas withdrawals made after the filing of the may be considered as a resignation from the union, the fact
petition are deemed involuntary. remains that at the time of the union’s application for registration,
the affiants were members of respondent and they comprised
The reason for such distinction is that if the withdrawal or more than the required 20% membership for purposes of
retraction is made before the filing of the petition, the names of registration as a labor union. Article 234 of the Labor Code
employees supporting the petition are supposed to be held secret merely requires a 20% minimum membership during the
to the opposite party. Logically, any such withdrawal or retraction application for union registration. It does not mandate that a union
shows voluntariness in the absence of proof to the contrary. must maintain the 20% minimum membership requirement all
Moreover, it becomes apparent that such employees had not given throughout its existence.141avvphi1
consent to the filing of the petition, hence the subscription
requirement has not been met. Respondent asserts that it had a total of 173 union members at the
time it applied for registration. Two names were repeated in
When the withdrawal or retraction is made after the petition is respondent’s list and had to be deducted, but the total would still
filed, the employees who are supporting the petition become be 171 union members. Further, out of the four names alleged to
known to the opposite party since their names are attached to the be no longer connected with petitioner, only two names should be
petition at the time of filing. Therefore, it would not be deleted from the list since Diana Motilla and T.W. Amutan
unexpected that the opposite party would use foul means for the resigned from petitioner only on May 10, 2005 and May 17, 2005,
subject employees to withdraw their support.12 respectively, or after respondent’s registration had already been
granted. Thus, the total union membership at the time of
In the instant case, the affidavits of recantation were executed registration was 169. Since the total number of rank-and-file
after the identities of the union members became public, i.e., after employees at that time was 528, 169 employees would be
the union filed a petition for certification election on May 23, equivalent to 32% of the total rank-and-file workers complement,
2005, since the names of the members were attached to the still very much above the minimum required by law.
petition. The purported withdrawal of support for the registration
of the union was made after the documents were submitted to the For the purpose of de-certifying a union such as respondent, it
DOLE, Region IV-A. The logical conclusion, therefore, following must be shown that there was misrepresentation, false statement
jurisprudence, is that the employees were not totally free from the or fraud in connection with the adoption or ratification of the
employer’s pressure, and so the voluntariness of the employees’ constitution and by-laws or amendments thereto; the minutes of
execution of the affidavits becomes suspect. ratification; or, in connection with the election of officers, the
minutes of the election of officers, the list of voters, or failure to
It is likewise notable that the first batch of 25 pro forma affidavits submit these documents together with the list of the newly
shows that the affidavits were executed by the individual affiants elected-appointed officers and their postal addresses to the BLR.15
on different dates from May 26, 2005 until June 3, 2005, but they
were all sworn before a notary public on June 8, 2005. The bare fact that two signatures appeared twice on the list of
those who participated in the organizational meeting would not, to
There was also a second set of standardized affidavits executed on our mind, provide a valid reason to cancel respondent’s certificate
different dates from May 26, 2005 until July 6, 2005. While these of registration. The cancellation of a union’s registration
77 affidavits were notarized on different dates, 56 of these were doubtless has an impairing dimension on the right of labor to self-
notarized on June 8, 2005, the very same date when the first set of organization. For fraud and misrepresentation to be grounds for
25 was notarized. cancellation of union registration under the Labor Code, the
nature of the fraud and misrepresentation must be grave and
compelling enough to vitiate the consent of a majority of union
Considering that the first set of 25 affidavits was submitted to the
members.
DOLE on June 14, 2005, it is surprising why petitioner was able
to submit the second set of affidavits only on July 12, 2005.
In this case, we agree with the BLR and the CA that respondent
could not have possibly committed misrepresentation, fraud, or
Accordingly, we cannot give full credence to these affidavits,
false statements. The alleged failure of respondent to indicate
which were executed under suspicious circumstances, and which
with mathematical precision the total number of employees in the
contain allegations unsupported by evidence. At best, these
bargaining unit is of no moment, especially as it was able to
affidavits are self-serving. They possess no probative value.
comply with the 20% minimum membership requirement. Even if
the total number of rank-and-file employees of petitioner is 528,
A retraction does not necessarily negate an earlier declaration. For while respondent declared that it should only be 455, it still
this reason, retractions are looked upon with disfavor and do not cannot be denied that the latter would have more than complied
automatically exclude the original statement or declaration based with the registration requirement.
WHEREFORE, the petition is DENIED. The assailed December Going into specifics, Eagle Ridge alleged that the EREU declared
20, 2007 Decision and the June 6, 2008 Resolution of the Court of in its application for registration having 30 members, when the
Appeals are AFFIRMED. Costs against petitioner. minutes of its December 6, 2005 organizational meeting showed
it only had 26 members. The misrepresentation was exacerbated
SO ORDERED. by the discrepancy between the certification issued by the Union
secretary and president that 25 members actually ratified the
constitution and by-laws on December 6, 2005 and the fact that
26 members affixed their signatures on the documents, making
one signature a forgery.

Finally, Eagle Ridge contended that five employees who attended


G.R. No. 178989               March 18, 2010 the organizational meeting had manifested the desire to withdraw
from the union. The five executed individual affidavits
EAGLE RIDGE GOLF & COUNTRY CLUB, Petitioner, or Sinumpaang Salaysay13 on February 15, 2006, attesting that
vs. they arrived late at said meeting which they claimed to be
COURT OF APPEALS and EAGLE RIDGE EMPLOYEES drinking spree; that they did not know that the documents they
UNION (EREU), Respondents. signed on that occasion pertained to the organization of a union;
and that they now wanted to be excluded from the Union. The
DECISION withdrawal of the five, Eagle Ridge maintained, effectively
reduced the union membership to 20 or 21, either of which is
below the mandatory minimum 20% membership requirement
VELASCO, JR., J.:
under Art. 234(c) of the Labor Code. Reckoned from 112 rank-
and-file employees of Eagle Ridge, the required number would be
In this petition for certiorari under Rule 65, Eagle Ridge Golf & 22 or 23 employees.
Country Club (Eagle Ridge) assails and seeks to nullify the
Resolutions of the Court of Appeals (CA) dated April 27,
As a counterpoint, EREU, in its Comment,14 argued in gist:
20071 and June 6, 2007,2 issued in CA-G.R. SP No. 98624,
denying a similar recourse petitioner earlier interposed to set
aside the December 21, 2006 Decision3 of the Bureau of Labor 1) the petition for cancellation was procedurally
Relations (BLR), as reiterated in a Resolution4 of March 7, 2007. deficient as it does not contain a certification against
forum shopping and that the same was verified by one
not duly authorized by Eagle Ridge’s board;
Petitioner Eagle Ridge is a corporation engaged in the business of
maintaining golf courses. It had, at the end of CY 2005, around
112 rank-and-file employees. The instant case is an off-shot of the 2) the alleged discrepancies are not real for before filing
desire of a number of these employees to organize themselves as of its application on December 19, 2005, four additional
a legitimate labor union and their employer’s opposition to their employees joined the union on December 8, 2005, thus
aspiration. raising the union membership to 30 members as of
December 19, 2005;
The Facts
3) the understatement by one member who ratified the
constitution and by-laws was a typographical error,
On December 6, 2005, at least 20% of Eagle Ridge’s rank-and-
which does not make it either grave or malicious
file employees—the percentage threshold required under Article
warranting the cancellation of the union’s registration;
234(c) of the Labor Code for union registration—had a meeting
where they organized themselves into an independent labor union,
named "Eagle Ridge Employees Union" (EREU or 4) the retraction of 5 union members should not be
Union),5 elected a set of officers,6 and ratified7 their constitution given any credence for the reasons that: (a) the sworn
and by-laws.8 statements of the five retracting union members sans
other affirmative evidence presented hardly qualify as
clear and credible evidence considering the joint
On December 19, 2005, EREU formally applied for
affidavits of the other members attesting to the orderly
registration9 and filed BLR Reg. Form No. I-LO, s. 199810 before
conduct of the organizational meeting; (b) the retracting
the Department of Labor and Employment (DOLE) Regional
members did not deny signing the union documents; (c)
Office IV (RO IV). In time, DOLE RO IV granted the application
following, Belyca Corporation v. Ferrer-
and issued EREU Registration Certificate (Reg. Cert.) No.
Calleja15 and Oriental Tin Can Labor Union v.
RO400-200512-UR-003.
Secretary of Labor and Employment,16 it can be
presumed that "duress, coercion or valuable
The EREU then filed a petition for certification election in Eagle consideration" was brought to bear on the retracting
Ridge Golf & Country Club, docketed as Case No. RO400-0601- members; and (d) citing La Suerte Cigar and Cigarette
RU-002. Eagle Ridge opposed this petition,11 followed by its Factory v. Director of Bureau of Labor
filing of a petition for the cancellation12 of Reg. Cert. No. RO400- Relations,17 Belyca Corporation and Oriental Tin Can
200512-UR-003. Docketed as RO400-0602-AU-003, Eagle Labor Union, where the Court ruled that "once the
Ridge’s petition ascribed misrepresentation, false statement, or required percentage requirement has been reached, the
fraud to EREU in connection with the adoption of its constitution employees’ withdrawal from union membership taking
and by-laws, the numerical composition of the Union, and the place after the filing of the petition for certification
election of its officers. election will not affect the petition," it asserted the
applicability of said ruling as the petition for 2) It was erroneous for both the Regional Director and
certification election was filed on January 10, 2006 or the BLR OIC Director to give credence to the retraction
long before February 15, 2006 when the affidavits of statements of union members which were not presented
retraction were executed by the five union members, for reaffirmation during any of the hearings of the case,
thus contending that the retractions do not affect nor be contrary to the requirement for the admission of such
deemed compelling enough to cancel its certificate of evidence under Sec. 11, Rule XI of DO 40-03.
registration.
In a Decision dated December 21, 2006, the BLR, now headed by
The Union presented the duly accomplished union membership Director Rebecca C. Chato, set aside the July 28, 2006 order of
forms18 dated December 8, 2005 of four additional members. And the BLR OIC Director, disposing as follows:
to rebut the allegations in the affidavits of retraction of the five
union members, it presented the Sama-Samang Sinumpaang WHEREFORE, the motion for reconsideration is hereby
Salaysay19 dated March 20, 2006 of eight union members; GRANTED and our Resolution dated 28 July 2006 is hereby
another Sama-Samang Sinumpaang Salaysay,20 also bearing date VACATED. Accordingly, the Eagle Ridge Employees Union
March 20, 2006, of four other union members; and the Sworn (EREU) shall remain in the roster of legitimate organizations.
Statement21 dated March 16, 2006 of the Union’s legal counsel,
Atty. Domingo T. Añonuevo. These affidavits attested to the
In finding for the Union, the BLR Director eschewed procedural
orderly and proper proceedings of the organizational meeting on
technicalities. Nonetheless, she found as without basis allegations
December 6, 2005.
of misrepresentation or fraud as ground for cancellation of
EREU’s registration.
In its Reply,22 Eagle Ridge reiterated the grounds it raised in its
petition for cancellation and asserted further that the four
In turn aggrieved, Eagle Ridge sought but was denied
additional members were fraudulently admitted into the Union.
reconsideration per the BLR’s Resolution dated March 7, 2007.
As Eagle Ridge claimed, the applications of the four neither
complied with the requirements under Section 2, Art. IV of the
union’s constitution and by-laws nor were they shown to have Eagle Ridge thereupon went to the CA on a petition for certiorari.
been duly received, issued receipts for admission fees, processed
with recommendation for approval, and approved by the union The Ruling of the CA
president.
On April 27, 2007, the appellate court, in a terse two-page
Moreover, Eagle Ridge presented another Sinumpaang Resolution,27 dismissed Eagle Ridge’s petition for being deficient,
Salaysay23 of retraction dated March 15, 2006 of another union as:
member. The membership of EREU had thus been further
reduced to only 19 or 20. This same member was listed in the 1. the questioned [BLR] Decision dated December 21,
first Sama-Samang Sinumpaang Salaysay24 presented by the 2006 and the Resolution dated March 7, 2007
Union but did not sign it. Resolution [appended to the petition] are mere machine
copies; and
The Ruling of the DOLE Regional Director
2. the verification and certification of non-forum
After due proceedings, the DOLE Regional Director, Region IV- shopping was subscribed to by Luna C. Piezas on her
A, focusing on the question of misrepresentation, issued on April representation as the legal counsel of the petitioner, but
28, 2006 an Order25 finding for Eagle Ridge, its petition to cancel sans [the requisite] Secretary’s Certificate or Board
Reg. Cert. No. RO400-200512-UR-003 being granted and EREU Resolution authorizing her to execute and sign the same.
being delisted from the roster of legitimate labor organizations.
The CA later denied, in its second assailed resolution, Eagle
Aggrieved, the Union appealed to the BLR, the recourse docketed Ridge’s motion for reconsideration, albeit the latter had submitted
as BLR A-C-30-5-31-06 (Case No. RO400-0602-AU-003). a certificate to show that its legal counsel has been authorized, per
a board resolution, to represent the corporation.
The Ruling of the BLR
The Issues
Initially, the BLR, then headed by an Officer-in-Charge (OIC),
affirmed26 the appealed order of the DOLE Regional Director. Eagle Ridge is now before us via this petition for certiorari on the
submissions that:
Undeterred by successive set backs, EREU interposed a motion
for reconsideration, contending that: I.

1) Contrary to the ruling of the BLR OIC Director, a [THE CA] COMMITTED SERIOUS ERROR AND GRAVE
certificate of non-forum shopping is mandatory ABUSE OF DISCRETION AMOUNTING TO LACK OR
requirement, under Department Order No. (DO) 40-03 EXCESS OF JURISDICTION IN DISMISSING THE
and the Rules of Court, non-compliance with which is a COMPANY’S PETITION FOR CERTIORARI AND DENYING
ground to dismiss a petition for cancellation of a ITS MOTION FOR RECONSIDERATION CONSIDERING
certificate of registration; THAT THE COMPANY’S PREVIOUS COUNSEL WAS
AUTHORIZED TO REPRESENT THE COMPANY IN THE Eagle Ridge tried to address its faux pas by submitting its board
PETITION FOR CERTIORARI FILED BEFORE THE [CA]; secretary’s Certificate33 dated May 15, 2007, attesting to the
issuance on May 10, 2007 of Board Resolution No. ERGCCI
II. 07/III-01 that authorized its counsel of record, Atty. Luna C.
Piezas, to represent it before the appellate court.
IN ORDER NOT TO FURTHER PREJUDICE THE
COMPANY, IT IS RESPECTFULLY SUBMITTED THAT The CA, however, rejected Eagle Ridge’s virtual plea for the
THIS HONORABLE COURT COULD TAKE COGNIZANCE relaxation of the rules on the signing of the verification and
OF THE MERITS OF THIS CASE AND RESOLVE THAT certification against forum shopping, observing that the board
BASED ON THE EVIDENCE ON RECORD, THERE WAS resolution adverted to was approved after Atty. Piezas has signed
FRAUD, MISREPRESENTATION AND/OR FALSE and filed for Eagle Ridge the petition for certiorari.
STATEMENT WHICH WARRANT THE CANCELLATION
OF CERTIFICATE OF REGISTRATION OF EREU.28 The appellate court’s assailed action is in no way tainted with
grave abuse of discretion, as Eagle Ridge would have this Court
The Court’s Ruling believed. Indeed, a certification of non-forum shopping signed by
counsel without the proper authorization is defective and
constitutes a valid cause for dismissal of the petition.34
We dismiss the petition.
The submission of the board secretary’s certificate through a
Procedural Issue: Lack of Authority
motion for reconsideration of the CA’s decision dismissing the
petition for certiorari may be considered a substantial compliance
Certiorari is an extraordinary, prerogative remedy and is never with the Rules of Court.35 Yet, this rule presupposes that the
issued as a matter of right.29 Accordingly, the party who seeks to authorizing board resolution, the approval of which is certified to
avail of it must strictly observe the rules laid down by law.30 by the secretary’s certification, was passed within the
reglementary period for filing the petition. This particular
Petitions for certiorari under Rule 65 of the Rules of Court require situation does not, however, obtain under the premises. The
a "sworn certification of non-forum shopping as provided in the records yield the following material dates and incidents: Eagle
third paragraph of Section 3, Rule 46."31 Sec. 3, paragraphs 4 and Ridge received the May 7, 2007 resolution of the BLR Director
6 of Rule 46 pertinently provides: on March 9, 2007, thus giving it 60 days or up to May 8, 2007 to
file a petition for certiorari, as it in fact filed its petition on April
SEC. 3. Contents and filing of petition; effect of non-compliance 18, 2007 before the CA. The authorization for its counsel,
with requirements. — x x x x however, was only issued in a meeting of its board on May 10,
2007 or a couple of days beyond the 60-day reglementary period
referred to in filing a certiorari action. Thus, there was no
xxxx substantial compliance with the Rules.

xxxx As with most rules of procedure, however, exceptions are


invariably recognized and the relaxation of procedural rules on
The petitioner shall also submit together with the petition review has been effected to obviate jeopardizing substantial
a sworn certification that he has not theretofore commenced justice.36 This liberality stresses the importance of review in our
any action involving the same issues in the Supreme Court, the judicial grievance structure to accord every party litigant the
Court of Appeals x x x, or any other tribunal or agency; if there is amplest opportunity for the proper and just disposition of his
such other action or proceeding, he must state the status of the cause, freed from the constraints of technicalities.37 But
same x x x. concomitant to a liberal interpretation of the rules of procedure
should be an effort on the part of the party invoking liberality to
xxxx adequately explain his failure to abide by the rules.381avvphi1

The failure of the petitioner to comply with any of the To us, Eagle Ridge has not satisfactorily explained its failure to
foregoing requirements shall be sufficient ground for the comply. It may be true, as Eagle Ridge urges, that its counsel’s
dismissal of the petition. (Emphasis supplied.) authority to represent the corporation was never questioned before
the DOLE regional office and agency. But EREU’s misstep could
hardly lend Eagle Ridge comfort. And obviously, Eagle Ridge
Evidently, the Rules requires the petitioner, not his counsel, to
and its counsel erred in equating the latter’s representation as
sign under oath the requisite certification against non-forum
legal counsel with the authority to sign the verification and the
shopping. Such certification is a peculiar personal representation
certificate of non-forum shopping in the former’s behalf. We note
on the part of the principal party, an assurance to the court that
that the authority to represent a client before a court or quasi-
there are no other pending cases involving basically the same
judicial agency does not require an authorizing board resolution,
parties, issues, and cause of action.32
as the counsel-client relationship is presumed by the counsel’s
representation by the filing of a pleading on behalf of the client.
In the instant case, the sworn verification and certification of non- In filing a pleading, the counsel affixes his signature on it, but it is
forum shopping in the petition for certiorari of Eagle Ridge filed the client who must sign the verification and the certification
before the CA carried the signature of its counsel without the against forum shopping, save when a board resolution authorizes
requisite authority. the former to sign so.
It is entirely a different matter for the counsel to sign the entitled to the rights and privileges granted by law to legitimate
verification and the certificate of non-forum shopping. The labor organizations upon issuance of the certificate of registration
attestation or certification in either verification or certification of based on the following requirements:
non-forum shopping requires the act of the principal party. As
earlier indicated, Sec. 3 of Rule 46 exacts this requirement; so (a) Fifty pesos (P50.00) registration fee;
does the first paragraph of Sec. 5 of Rule 7 pertinently reading:
(b) The names of its officers, their addresses, the
SEC. 5. Certification against forum shopping. — The plaintiff or principal address of the labor organization, the minutes
principal party shall certify under oath in the complaint or other of the organizational meetings and the list of workers
initiatory pleading asserting a claim for relief, or in a sworn who participated in such meetings;
certification annexed thereto and simultaneously filed therewith:
(a) that he has not theretofore commenced any action or filed any
(c) The names of all its members comprising at least
claim involving the same issues in any court, tribunal or quasi-
twenty percent (20%) of all the employees in the
judicial agency and, to the best of his knowledge, no such other
bargaining unit where it seeks to operate;
action or claim is pending therein; (b) if there is such other
pending action or claim, a complete statement of the present
status thereof; and (c) if he should thereafter learn that the same xxxx
or similar action or claim has been filed or is pending, he shall
report that fact within five (5) days therefrom to the court wherein (e) Four copies (4) of the constitution and by-laws of the
his aforesaid complaint or initiatory pleading has been filed. applicant union, minutes of its adoption or
(Emphasis added.) ratification and the list of the members who
participated in it.41
It is, thus, clear that the counsel is not the proper person to sign
the certification against forum shopping. If, for any reason, the xxxx
principal party cannot sign the petition, the one signing on his
behalf must have been duly authorized.39 ART. 239. GROUNDS FOR CANCELLATION OF UNION
REGISTRATION. –– The following shall constitute grounds for
In addition, Eagle Ridge maintains that the submitted board cancellation of union registration:
resolution, albeit passed after the filing of the petition was filed,
should be treated as a ratificatory medium of the counsel’s act of (a) Misrepresentation, false statements or fraud in
signing the sworn certification of non-forum shopping. connection with the adoption or ratification of the
constitution and by-laws or amendments thereto,
We are not inclined to grant the desired liberality owing to Eagle the minutes of ratification, and the list of members
Ridge’s failure to sufficiently explain its failure to follow the who took part in the ratification;
clear rules.
xxxx
If for the foregoing considerations alone, the Court could very
well dismiss the instant petition. Nevertheless, the Court will (c) Misrepresentation, false statements or fraud in
explore the merits of the instant case to obviate the inequity that connection with the election of officers, minutes of
might result from the outright denial of the petition. the election of officers, the list of voters, or failure to
submit these documents together with the list of the
Substantive Issue: No Fraud in the Application newly elected/appointed officers and their postal
addresses within thirty (30) days from
Eagle Ridge cites the grounds provided under Art. 239(a) and (c) election.42 (Emphasis supplied.)
of the Labor Code for its petition for cancellation of the EREU’s
registration. On the other hand, the Union asserts bona A scrutiny of the records fails to show any misrepresentation,
fide compliance with the registration requirements under Art. 234 false statement, or fraud committed by EREU to merit
of the Code, explaining the seeming discrepancies between the cancellation of its registration.
number of employees who participated in the organizational
meeting and the total number of union members at the time it First. The Union submitted the required documents
filed its registration, as well as the typographical error in its attesting to the facts of the organizational meeting on
certification which understated by one the number of union December 6, 2005, the election of its officers, and the
members who ratified the union’s constitution and by-laws. adoption of the Union’s constitution and by-laws. It
submitted before the DOLE Regional Office with its
Before their amendment by Republic Act No. 948140 on June 15, Application for Registration and the duly filled out BLR
2007, the then governing Art. 234 (on the requirements of Reg. Form No. I-LO, s. 1998, the following documents,
registration of a labor union) and Art. 239 (on the grounds for to wit:
cancellation of union registration) of the Labor Code respectively
provided as follows: (a) the minutes of its organizational
meeting43 held on December 6, 2005 showing
ART. 234. REQUIREMENTS OF REGISTRATION. –– 26 founding members who elected its union
Any applicant labor organization, association or group of officers by secret ballot;
unions or workers shall acquire legal personality and shall be
(b) the list of rank-and-file employees44 of members comprising at least 20% of the bargaining unit.
Eagle Ridge who attended the organizational The fact that EREU had 30 members when it applied for
meeting and the election of officers with their registration on December 19, 2005 while only 26
individual signatures; actually participated in the organizational meeting is
borne by the records.
(c) the list of rank-and-file employees45 who
ratified the union’s constitution and by-laws Fourth. In its futile attempt to clutch at straws, Eagle
showing the very same list as those who Ridge assails the inclusion of the additional four
attended the organizational meeting and the members allegedly for not complying with what it
election of officers with their individual termed as "the sine qua non requirements" for union
signatures except the addition of four member applications under the Union’s constitution and
employees without their signatures, i.e., Cherry by-laws, specifically Sec. 2 of Art. IV. We are not
Labajo, Grace Pollo, Annalyn Poniente and persuaded. Any seeming infirmity in the application and
Rowel Dolendo; admission of union membership, most especially in
cases of independent labor unions, must be viewed in
(d) the union’s constitution and by-laws46 as favor of valid membership.
approved on December 6, 2005;
The right of employees to self-organization and
(e) the list of officers47 and their addresses; membership in a union must not be trammeled by undue
difficulties. In this case, when the Union said that the
four employee-applicants had been admitted as union
(f) the list of union members48 showing a total
members, it is enough to establish the fact of admission
of 30 members; and
of the four that they had duly signified such desire by
accomplishing the membership form. The fact, as
(g) the Sworn Statement49 of the union’s pointed out by Eagle Ridge, that the Union, owing to its
elected president and secretary. All the scant membership, had not yet fully organized its
foregoing documents except the sworn different committees evidently shows the direct and
statement of the president and the secretary valid acceptance of the four employee applicants rather
were accompanied by Certifications50 by the than deter their admission—as erroneously asserted by
union secretary duly attested to by the union Eagle Ridge.
president.
Fifth. The difference between the number of 26
Second. The members of the EREU totaled 30 members, who ratified the Union’s constitution and by-
employees when it applied on December 19, 2005 for laws, and the 25 members shown in the certification of
registration. The Union thereby complied with the the Union secretary as having ratified it, is, as shown by
mandatory minimum 20% membership requirement the factual antecedents, a typographical error. It was an
under Art. 234(c). Of note is the undisputed number of insignificant mistake committed without malice or
112 rank-and-file employees in Eagle Ridge, as shown prevarication. The list of those who attended the
in the Sworn Statement of the Union president and organizational meeting shows 26 members, as evidenced
secretary and confirmed by Eagle Ridge in its petition by the signatures beside their handwritten names. Thus,
for cancellation. the certification’s understatement by one member, while
not factual, was clearly an error, but neither a
Third. The Union has sufficiently explained the misleading one nor a misrepresentation of what had
discrepancy between the number of those who attended actually happened.
the organizational meeting showing 26 employees and
the list of union members showing 30. The difference is Sixth. In the more meaty issue of the affidavits of
due to the additional four members admitted two days retraction executed by six union members, we hold that
after the organizational meeting as attested to by their the probative value of these affidavits cannot overcome
duly accomplished Union Membership forms. those of the supporting affidavits of 12 union members
Consequently, the total number of union members, as of and their counsel as to the proceedings and the conduct
December 8, 2005, was 30, which was truthfully of the organizational meeting on December 6, 2005. The
indicated in its application for registration on December DOLE Regional Director and the BLR OIC Director
19, 2005. obviously erred in giving credence to the affidavits of
retraction, but not according the same treatment to the
As aptly found by the BLR Director, the Union already supporting affidavits.
had 30 members when it applied for registration, for the
admission of new members is neither prohibited by law The six affiants of the affidavits of retraction were not
nor was it concealed in its application for registration. presented in a hearing before the Hearing Officer
Eagle Ridge’s contention is flawed when it equated the (DOLE Regional Director), as required under the Rules
requirements under Art. 234(b) and (c) of the Labor Implementing Book V of the Labor Code covering
Code. Par. (b) clearly required the submission of the Labor Relations. Said Rules is embodied in Department
minutes of the organizational meetings and the list of Order No. (DO) 40-03 which was issued on February
workers who participated in the meetings, while par. (c) 17, 2003 and took effect on March 15, 2003 to replace
merely required the list of names of all the union
DO 9 of 1997. Sec. 11, Rule XI of DO 40-03 least 22 employees (112 x 205 = 22.4). When the EREU
specifically requires: filed its application for registration on December 19,
2005, there were clearly 30 union members. Thus, when
Section 11. Affirmation of testimonial evidence. the certificate of registration was granted, there is no
– Any affidavit submitted by a party to prove his/her dispute that the Union complied with the mandatory
claims or defenses shall be re-affirmed by the 20% membership requirement.
presentation of the affiant before the Med-Arbiter
or Hearing Officer, as the case may be. Any affidavit Besides, it cannot be argued that the six affidavits of
submitted without the re-affirmation of the retraction retroact to the time of the application of
affiant during a scheduled hearing shall not be registration or even way back to the organizational
admitted in evidence, except when the party against meeting. Prior to their withdrawal, the six employees in
whom the affidavit is being offered admits all question were bona fide union members. More so, they
allegations therein and waives the examination of the never disputed affixing their signatures beside their
affiant. handwritten names during the organizational meetings.
While they alleged that they did not know what they
It is settled that affidavits partake the nature of hearsay were signing, it bears stressing that their affidavits of
evidence, since they are not generally prepared by the retraction were not re-affirmed during the hearings of
affiant but by another who uses his own language in the instant case rendering them of little, if any,
writing the affiant’s statement, which may thus be either evidentiary value.
omitted or misunderstood by the one writing them.51 The
above rule affirms the general requirement in adversarial With the withdrawal of six union members, there is still
proceedings for the examination of the affiant by the compliance with the mandatory membership
party against whom the affidavit is offered. In the requirement under Art. 234(c), for the remaining 24
instant case, it is required for affiants to re-affirm the union members constitute more than the 20%
contents of their affidavits during the hearing of the membership requirement of 22 employees.
instant case for them to be examined by the opposing
party, i.e., the Union. Eagle Ridge further argues that the list of union
members includes a supervisory employee. This is a
For their non-presentation and consonant to the above- factual issue which had not been raised at the first
quoted rule, the six affidavits of retraction are instance before the DOLE Regional Director and cannot
inadmissible as evidence against the Union in the instant be appreciated in this proceeding. To be sure, Eagle
case. Moreover, the affidavit and joint-affidavits Ridge knows well who among its personnel belongs or
presented by the Union before the DOLE Regional does not belong to the supervisory group. Obviously, its
Director were duly re-affirmed in the hearing of March attempt to raise the issue referred to is no more than an
20, 2006 by the affiants. Thus, a reversible error was afterthought and ought to be rejected.
committed by the DOLE Regional Director and the BLR
OIC Director in giving credence to the inadmissible Eighth. Finally, it may not be amiss to note, given the
affidavits of retraction presented by Eagle Ridge while factual antecedents of the instant case, that Eagle Ridge
not giving credence to the duly re-affirmed affidavits has apparently resorted to filing the instant case for
presented by the Union. cancellation of the Union’s certificate of registration to
bar the holding of a certification election. This can be
Evidently, the allegations in the six affidavits of gleaned from the fact that the grounds it raised in its
retraction have no probative value and at the very least opposition to the petition for certification election are
cannot outweigh the rebutting attestations of the duly re- basically the same grounds it resorted to in the instant
affirmed affidavits presented by the Union. case for cancellation of EREU’s certificate of
registration. This amounts to a clear circumvention of
Seventh. The fact that six union members, indeed, the law and cannot be countenanced.
expressed the desire to withdraw their membership
through their affidavits of retraction will not cause the For clarity, we reiterate the following undisputed
cancellation of registration on the ground of violation of antecedent facts:
Art. 234(c) of the Labor Code requiring the mandatory
minimum 20% membership of rank-and-file employees (1) On December 6, 2005, the Union was
in the employees’ union. organized, with 26 employees of Eagle Ridge
attending;
The six retracting union members clearly severed and
withdrew their union membership. The query is whether (2) On December 19, 2005, the Union filed its
such separation from the Union can detrimentally affect formal application for registration indicating a
the registration of the Union. total of 30 union members with the inclusion
of four additional members on December 8,
We answer in the negative. 2005 (Reg. Cert. No. RO400-200512-UR-003
was eventually issued by the DOLE RO IV-A);
Twenty percent (20%) of 112 rank-and-file employees
in Eagle Ridge would require a union membership of at
(3) On January 10, 2006, the Union filed labor union made before the filing of the petition for
before the DOLE RO IV-A its petition for certification election is presumed voluntary, while withdrawal
certification election in Eagle Ridge; after the filing of such petition is considered to be involuntary
and does not affect the same. Now then, if a withdrawal from
(4) On February 13, 2006, Eagle Ridge filed its union membership done after a petition for certification
Position Paper opposing the petition for election has been filed does not vitiate such petition, is it not
certification election on essentially the same but logical to assume that such withdrawal cannot work to
grounds it raised in the instant case; and nullify the registration of the union? Upon this light, the Court
is inclined to agree with the CA that the BLR did not abuse its
discretion nor gravely err when it concluded that the affidavits of
(5) On February 24, 2006, Eagle Ridge filed
retraction of the 82 members had no evidentiary
the instant case for cancellation of the Union’s
weight.59 (Emphasis supplied.)
certificate of registration on essentially the
same grounds it raised in its opposition to the
Union’s petition for certification election. WHEREFORE, premises considered, we DISMISS the instant
petition for lack of merit.
Evidently, as the Union persuasively argues, the withdrawal of six
member-employees from the Union will affect neither the Costs against petitioner.
Union’s registration nor its petition for certification election, as
their affidavits of retraction were executed after the Union’s SO ORDERED.
petition for certification election had been filed. The initial five
affidavits of retraction were executed on February 15, 2006; the
sixth, on March 15, 2006. Indisputably, all six were executed way
after the filing of the petition for certification election on January
10, 2006.
G.R. No. 178296               January 12, 2011
In Eastland Manufacturing Company, Inc. v. Noriel,52 the Court
emphasized, and reiterated its earlier rulings,53 that "even if there
were less than 30% [the required percentage of minimum THE HERITAGE HOTEL MANILA, acting through its
membership then] of the employees asking for a certification owner, GRAND PLAZA HOTEL
election, that of itself would not be a bar to respondent Director CORPORATION, Petitioner,
ordering such an election provided, of course, there is no grave vs.
abuse of discretion."54 Citing Philippine Association of Free NATIONAL UNION OF WORKERS IN THE HOTEL,
Labor Unions v. Bureau of Labor Relations,55 the Court RESTAURANT AND ALLIED INDUSTRIES-HERITAGE
emphasized that a certification election is the most appropriate HOTEL MANILA SUPERVISORS CHAPTER
procedure for the desired goal of ascertaining which of the (NUWHRAIN-HHMSC), Respondent.
competing organizations should represent the employees for the
purpose of collective bargaining.56 DECISION

Indeed, where the company seeks the cancellation of a union’s NACHURA, J.:


registration during the pendency of a petition for certification
election, the same grounds invoked to cancel should not be used Before the Court is a petition for review on certiorari of the
to bar the certification election. A certification election is the Decision1 of the Court of Appeals (CA) dated May 30, 2005 and
most expeditious and fairest mode of ascertaining the will of a Resolution dated June 4, 2007. The assailed Decision affirmed the
collective bargaining unit as to its choice of its exclusive dismissal of a petition for cancellation of union registration filed
representative.57 It is the fairest and most effective way of by petitioner, Grand Plaza Hotel Corporation, owner of Heritage
determining which labor organization can truly represent the Hotel Manila, against respondent, National Union of Workers in
working force. It is a fundamental postulate that the will of the the Hotel, Restaurant and Allied Industries-Heritage Hotel Manila
majority, if given expression in an honest election with freedom Supervisors Chapter (NUWHRAIN-HHMSC), a labor
on the part of the voters to make their choice, is controlling.58 organization of the supervisory employees of Heritage Hotel
Manila.
The Court ends this disposition by reproducing the following apt
excepts from its holding in S.S. Ventures International, Inc. v. The case stemmed from the following antecedents:
S.S. Ventures Labor Union (SSVLU) on the effect of the
withdrawal from union membership right before or after the filing
On October 11, 1995, respondent filed with the Department of
of a petition for certification election:
Labor and Employment-National Capital Region (DOLE-NCR) a
petition for certification election.2 The Med-Arbiter granted the
We are not persuaded. As aptly noted by both the BLR and CA, petition on February 14, 1996 and ordered the holding of a
these mostly undated written statements submitted by Ventures on certification election.3 On appeal, the DOLE Secretary, in a
March 20, 2001, or seven months after it filed its petition for Resolution dated August 15, 1996, affirmed the Med-Arbiter’s
cancellation of registration, partake of the nature of withdrawal of order and remanded the case to the Med-Arbiter for the holding of
union membership executed after the Union’s filing of a petition a preelection conference on February 26, 1997. Petitioner filed a
for certification election on March 21, 2000. We have in motion for reconsideration, but it was denied on September 23,
precedent cases said that the employees’ withdrawal from a 1996.
The preelection conference was not held as initially scheduled; it certification election had already been held, and the members had
was held a year later, or on February 20, 1998. Petitioner moved manifested their will to be represented by respondent.
to archive or to dismiss the petition due to alleged repeated non-
appearance of respondent. The latter agreed to suspend Citing National Union of Bank Employees v. Minister of Labor,
proceedings until further notice. The preelection conference et al.9 and Samahan ng Manggagawa sa Pacific Plastic v. Hon.
resumed on January 29, 2000. Laguesma,10 the Med-Arbiter held that the pendency of a petition
for cancellation of registration is not a bar to the holding of a
Subsequently, petitioner discovered that respondent had failed to certification election. Thus, in an Order11 dated January 26, 2001,
submit to the Bureau of Labor Relations (BLR) its annual the Med-Arbiter dismissed petitioner’s protest, and certified
financial report for several years and the list of its members since respondent as the sole and exclusive bargaining agent of all
it filed its registration papers in 1995. Consequently, on May 19, supervisory employees.
2000, petitioner filed a Petition for Cancellation of Registration of
respondent, on the ground of the non-submission of the said Petitioner subsequently appealed the said Order to the DOLE
documents. Petitioner prayed that respondent’s Certificate of Secretary.12 The appeal was later dismissed by DOLE Secretary
Creation of Local/Chapter be cancelled and its name be deleted Patricia A. Sto. Tomas (DOLE Secretary Sto. Tomas) in the
from the list of legitimate labor organizations. It further requested Resolution of August 21, 2002.13 Petitioner moved for
the suspension of the certification election proceedings.4 reconsideration, but the motion was also denied.14

On June 1, 2000, petitioner reiterated its request by filing a In the meantime, Regional Director Alex E. Maraan (Regional
Motion to Dismiss or Suspend the [Certification Election] Director Maraan) of DOLE-NCR finally resolved the petition for
Proceedings,5 arguing that the dismissal or suspension of the cancellation of registration. While finding that respondent had
proceedings is warranted, considering that the legitimacy of indeed failed to file financial reports and the list of its members
respondent is seriously being challenged in the petition for for several years, he, nonetheless, denied the petition,
cancellation of registration. Petitioner maintained that the ratiocinating that freedom of association and the employees’ right
resolution of the issue of whether respondent is a legitimate labor to self-organization are more substantive considerations. He took
organization is crucial to the issue of whether it may exercise into account the fact that respondent won the certification election
rights of a legitimate labor organization, which include the right and that it had already been certified as the exclusive bargaining
to be certified as the bargaining agent of the covered employees. agent of the supervisory employees. In view of the foregoing,
Regional Director Maraan—while emphasizing that the non-
Nevertheless, the certification election pushed through on June compliance with the law is not viewed with favor—considered
23, 2000. Respondent emerged as the winner.6 the belated submission of the annual financial reports and the list
of members as sufficient compliance thereof and considered them
On June 28, 2000, petitioner filed a Protest with Motion to Defer as having been submitted on time. The dispositive portion of the
Certification of Election Results and Winner,7 stating that the decision15 dated December 29, 2001 reads:
certification election held on June 23, 2000 was an exercise in
futility because, once respondent’s registration is cancelled, it WHEREFORE, premises considered, the instant petition to delist
would no longer be entitled to be certified as the exclusive the National Union of Workers in the Hotel, Restaurant and
bargaining agent of the supervisory employees. Petitioner also Allied Industries-Heritage Hotel Manila Supervisors Chapter
claimed that some of respondent’s members were not qualified to from the roll of legitimate labor organizations is hereby DENIED.
join the union because they were either confidential employees or
managerial employees. It then prayed that the certification of the SO ORDERED.16
election results and winner be deferred until the petition for
cancellation shall have been resolved, and that respondent’s
Aggrieved, petitioner appealed the decision to the BLR.17 BLR
members who held confidential or managerial positions be
Director Hans Leo Cacdac inhibited himself from the case
excluded from the supervisors’ bargaining unit.
because he had been a former counsel of respondent.
Meanwhile, respondent filed its Answer8 to the petition for the
In view of Director Cacdac’s inhibition, DOLE Secretary Sto.
cancellation of its registration. It averred that the petition was
Tomas took cognizance of the appeal. In a resolution18 dated
filed primarily to delay the conduct of the certification election,
February 21, 2003, she dismissed the appeal, holding that the
the respondent’s certification as the exclusive bargaining
constitutionally guaranteed freedom of association and right of
representative of the supervisory employees, and the
workers to self-organization outweighed respondent’s
commencement of bargaining negotiations. Respondent prayed
noncompliance with the statutory requirements to maintain its
for the dismissal of the petition for the following reasons: (a)
status as a legitimate labor organization.
petitioner is estopped from questioning respondent’s status as a
legitimate labor organization as it had already recognized
respondent as such during the preelection conferences; (b) Petitioner filed a motion for reconsideration,19 but the motion was
petitioner is not the party-in-interest, as the union members are likewise denied in a resolution20 dated May 30, 2003. DOLE
the ones who would be disadvantaged by the non-submission of Secretary Sto. Tomas admitted that it was the BLR which had
financial reports; (c) it has already complied with the reportorial jurisdiction over the appeal, but she pointed out that the BLR
requirements, having submitted its financial statements for 1996, Director had voluntarily inhibited himself from the case because
1997, 1998, and 1999, its updated list of officers, and its list of he used to appear as counsel for respondent. In order to maintain
members for the years 1995, 1996, 1997, 1998, and 1999; (d) the the integrity of the decision and of the BLR, she therefore
petition is already moot and academic, considering that the accepted the motion to inhibit and took cognizance of the appeal.
Petitioner filed a petition for certiorari with the CA, raising the II.
issue of whether the DOLE Secretary acted with grave abuse of
discretion in taking cognizance of the appeal and affirming the The Court of Appeals gravely erred in affirming the dismissal of
dismissal of its petition for cancellation of respondent’s the Cancellation Petition despite the mandatory and unequivocal
registration. provisions of the Labor Code and its Implementing Rules.24

In a Decision dated May 30, 2005, the CA denied the petition. The petition has no merit.
The CA opined that the DOLE Secretary may legally assume
jurisdiction over an appeal from the decision of the Regional
Jurisdiction to review the decision of the Regional Director lies
Director in the event that the Director of the BLR inhibits himself
with the BLR. This is clearly provided in the Implementing Rules
from the case. According to the CA, in the absence of the BLR
of the Labor Code and enunciated by the Court in Abbott. But as
Director, there is no person more competent to resolve the appeal
pointed out by the CA, the present case involves a peculiar
than the DOLE Secretary. The CA brushed aside the allegation of
circumstance that was not present or covered by the ruling in
bias and partiality on the part of the DOLE Secretary, considering
Abbott. In this case, the BLR Director inhibited himself from the
that such allegation was not supported by any evidence.
case because he was a former counsel of respondent. Who, then,
shall resolve the case in his place?
The CA also found that the DOLE Secretary did not commit
grave abuse of discretion when she affirmed the dismissal of the
In Abbott, the appeal from the Regional Director’s decision was
petition for cancellation of respondent’s registration as a labor
directly filed with the Office of the DOLE Secretary, and we
organization. Echoing the DOLE Secretary, the CA held that the
ruled that the latter has no appellate jurisdiction. In the instant
requirements of registration of labor organizations are an exercise
case, the appeal was filed by petitioner with the BLR, which,
of the overriding police power of the State, designed for the
undisputedly, acquired jurisdiction over the case. Once
protection of workers against potential abuse by the union that
jurisdiction is acquired by the court, it remains with it until the
recruits them. These requirements, the CA opined, should not be
full termination of the case.25
exploited to work against the workers’ constitutionally protected
right to self-organization.
Thus, jurisdiction remained with the BLR despite the BLR
Director’s inhibition. When the DOLE Secretary resolved the
Petitioner filed a motion for reconsideration, invoking this
appeal, she merely stepped into the shoes of the BLR Director and
Court’s ruling in Abbott Labs. Phils., Inc. v. Abbott Labs.
performed a function that the latter could not himself perform.
Employees Union,21 which categorically declared that the DOLE
She did so pursuant to her power of supervision and control over
Secretary has no authority to review the decision of the Regional
the BLR.26
Director in a petition for cancellation of union registration, and
Section 4,22 Rule VIII, Book V of the Omnibus Rules
Implementing the Labor Code. Expounding on the extent of the power of control, the Court, in
Araneta, et al. v. Hon. M. Gatmaitan, et al.,27 pronounced that, if a
certain power or authority is vested by law upon the Department
In its Resolution23 dated June 4, 2007, the CA denied petitioner’s
Secretary, then such power or authority may be exercised directly
motion, stating that the BLR Director’s inhibition from the case
by the President, who exercises supervision and control over the
was a peculiarity not present in the Abbott case, and that such
departments. This principle was incorporated in the
inhibition justified the assumption of jurisdiction by the DOLE
Administrative Code of 1987, which defines "supervision and
Secretary.
control" as including the authority to act directly whenever a
specific function is entrusted by law or regulation to a
In this petition, petitioner argues that: subordinate.28 Applying the foregoing to the present case, it is
clear that the DOLE Secretary, as the person exercising the power
I. of supervision and control over the BLR, has the authority to
directly exercise the quasi-judicial function entrusted by law to
The Court of Appeals seriously erred in ruling that the Labor the BLR Director.
Secretary properly assumed jurisdiction over Petitioner’s appeal
of the Regional Director’s Decision in the Cancellation Petition x It is true that the power of control and supervision does not give
x x. the Department Secretary unbridled authority to take over the
functions of his or her subordinate. Such authority is subject to
A. Jurisdiction is conferred only by law. The Labor certain guidelines which are stated in Book IV, Chapter 8, Section
Secretary had no jurisdiction to review the decision of 39(1)(a) of the Administrative Code of 1987.29 However, in the
the Regional Director in a petition for cancellation. Such present case, the DOLE Secretary’s act of taking over the
jurisdiction is conferred by law to the BLR. function of the BLR Director was warranted and necessitated by
the latter’s inhibition from the case and the objective to "maintain
the integrity of the decision, as well as the Bureau itself."30
B. The unilateral inhibition by the BLR Director cannot
justify the Labor Secretary’s exercise of jurisdiction
over the Appeal. Petitioner insists that the BLR Director’s subordinates should
have resolved the appeal, citing the provision under the
Administrative Code of 1987 which states, "in case of the absence
C. The Labor Secretary’s assumption of jurisdiction
or disability of the head of a bureau or office, his duties shall be
over the Appeal without notice violated Petitioner’s
performed by the assistant head."31 The provision clearly does not
right to due process.
apply considering that the BLR Director was neither absent nor
suffering from any disability; he remained as head of the BLR. These provisions give the Regional Director ample discretion in
Thus, to dispel any suspicion of bias, the DOLE Secretary opted dealing with a petition for cancellation of a union’s registration,
to resolve the appeal herself. particularly, determining whether the union still meets the
requirements prescribed by law. It is sufficient to give the
Petitioner was not denied the right to due process when it was not Regional Director license to treat the late filing of required
notified in advance of the BLR Director’s inhibition and the documents as sufficient compliance with the requirements of the
DOLE Secretary’s assumption of the case. Well-settled is the rule law. After all, the law requires the labor organization to submit
that the essence of due process is simply an opportunity to be the annual financial report and list of members in order to verify
heard, or, as applied to administrative proceedings, an opportunity if it is still viable and financially sustainable as an organization so
to explain one’s side or an opportunity to seek a reconsideration as to protect the employer and employees from fraudulent or fly-
of the action or ruling complained of.32 Petitioner had the by-night unions. With the submission of the required documents
opportunity to question the BLR Director’s inhibition and the by respondent, the purpose of the law has been achieved, though
DOLE Secretary’s taking cognizance of the case when it filed a belatedly.
motion for reconsideration of the latter’s decision. It would be
well to state that a critical component of due process is a hearing We cannot ascribe abuse of discretion to the Regional Director
before an impartial and disinterested tribunal, for all the elements and the DOLE Secretary in denying the petition for cancellation
of due process, like notice and hearing, would be meaningless if of respondent’s registration. The union members and, in fact, all
the ultimate decision would come from a partial and biased the employees belonging to the appropriate bargaining unit should
judge.33 It was precisely to ensure a fair trial that moved the BLR not be deprived of a bargaining agent, merely because of the
Director to inhibit himself from the case and the DOLE Secretary negligence of the union officers who were responsible for the
to take over his function. submission of the documents to the BLR.

Petitioner also insists that respondent’s registration as a legitimate Labor authorities should, indeed, act with circumspection in
labor union should be cancelled. Petitioner posits that once it is treating petitions for cancellation of union registration, lest they
determined that a ground enumerated in Article 239 of the Labor be accused of interfering with union activities. In resolving the
Code is present, cancellation of registration should follow; it petition, consideration must be taken of the fundamental rights
becomes the ministerial duty of the Regional Director to cancel guaranteed by Article XIII, Section 3 of the Constitution, i.e., the
the registration of the labor organization, hence, the use of the rights of all workers to self-organization, collective bargaining
word "shall." Petitioner points out that the Regional Director has and negotiations, and peaceful concerted activities. Labor
admitted in its decision that respondent failed to submit the authorities should bear in mind that registration confers upon a
required documents for a number of years; therefore, cancellation union the status of legitimacy and the concomitant right and
of its registration should have followed as a matter of course. privileges granted by law to a legitimate labor organization,
particularly the right to participate in or ask for certification
We are not persuaded. election in a bargaining unit.36 Thus, the cancellation of a
certificate of registration is the equivalent of snuffing out the life
of a labor organization. For without such registration, it loses - as
Articles 238 and 239 of the Labor Code read:
a rule - its rights under the Labor Code.37

ART. 238. CANCELLATION OF REGISTRATION; APPEAL


It is worth mentioning that the Labor Code’s provisions on
cancellation of union registration and on reportorial requirements
The certificate of registration of any legitimate labor organization, have been recently amended by Republic Act (R.A.) No. 9481,
whether national or local, shall be canceled by the Bureau if it has An Act Strengthening the Workers’ Constitutional Right to Self-
reason to believe, after due hearing, that the said labor Organization, Amending for the Purpose Presidential Decree No.
organization no longer meets one or more of the requirements 442, As Amended, Otherwise Known as the Labor Code of the
herein prescribed.34 Philippines, which lapsed into law on May 25, 2007 and became
effective on June 14, 2007. The amendment sought to strengthen
ART. 239. GROUNDS FOR CANCELLATION OF UNION the workers’ right to self-organization and enhance the
REGISTRATION. Philippines’ compliance with its international obligations as
embodied in the International Labour Organization (ILO)
The following shall constitute grounds for cancellation of union Convention No. 87,38 pertaining to the non-dissolution of
registration: workers’ organizations by administrative authority.39 Thus, R.A.
No. 9481 amended Article 239 to read:
xxxx
ART. 239. Grounds for Cancellation of Union Registration.—The
following may constitute grounds for cancellation of union
(d) Failure to submit the annual financial report to the Bureau registration:
within thirty (30) days after the closing of every fiscal year and
misrepresentation, false entries or fraud in the preparation of the
financial report itself; (a) Misrepresentation, false statement or fraud in
connection with the adoption or ratification of the
constitution and by-laws or amendments thereto, the
xxxx minutes of ratification, and the list of members who
took part in the ratification;
(i) Failure to submit list of individual members to the Bureau
once a year or whenever required by the Bureau.35
(b) Misrepresentation, false statements or fraud in State to exact compliance with reporting requirements. Yet there
connection with the election of officers, minutes of the is more at stake in this case than merely monitoring union
election of officers, and the list of voters; activities and requiring periodic documentation thereof.

(c) Voluntary dissolution by the members. The more substantive considerations involve the constitutionally
guaranteed freedom of association and right of workers to self-
R.A. No. 9481 also inserted in the Labor Code Article 242-A, organization. Also involved is the public policy to promote free
which provides: trade unionism and collective bargaining as instruments of
industrial peace and democracy.1avvphi1 An overly stringent
interpretation of the statute governing cancellation of union
ART. 242-A. Reportorial Requirements.—The following are
registration without regard to surrounding circumstances cannot
documents required to be submitted to the Bureau by the
be allowed. Otherwise, it would lead to an unconstitutional
legitimate labor organization concerned:
application of the statute and emasculation of public policy
objectives. Worse, it can render nugatory the protection to labor
(a) Its constitution and by-laws, or amendments thereto, and social justice clauses that pervades the Constitution and the
the minutes of ratification, and the list of members who Labor Code.
took part in the ratification of the constitution and by-
laws within thirty (30) days from adoption or ratification
Moreover, submission of the required documents is the duty of
of the constitution and by-laws or amendments thereto;
the officers of the union. It would be unreasonable for this Office
to order the cancellation of the union and penalize the entire
(b) Its list of officers, minutes of the election of officers, union membership on the basis of the negligence of its officers.
and list of voters within thirty (30) days from election; In National Union of Bank Employees vs. Minister of Labor, L-
53406, 14 December 1981, 110 SCRA 296, the Supreme Court
(c) Its annual financial report within thirty (30) days ruled:
after the close of every fiscal year; and
As aptly ruled by respondent Bureau of Labor Relations Director
(d) Its list of members at least once a year or whenever Noriel: "The rights of workers to self-organization finds general
required by the Bureau. and specific constitutional guarantees. x x x Such constitutional
guarantees should not be lightly taken much less nullified. A
Failure to comply with the above requirements shall not be a healthy respect for the freedom of association demands that acts
ground for cancellation of union registration but shall subject the imputable to officers or members be not easily visited with capital
erring officers or members to suspension, expulsion from punishments against the association itself."
membership, or any appropriate penalty.
At any rate, we note that on 19 May 2000, appellee had submitted
ILO Convention No. 87, which we have ratified in 1953, provides its financial statement for the years 1996-1999. With this
that "workers’ and employers’ organizations shall not be liable to submission, appellee has substantially complied with its duty to
be dissolved or suspended by administrative authority." The ILO submit its financial report for the said period. To rule differently
has expressed the opinion that the cancellation of union would be to preclude the union, after having failed to meet its
registration by the registrar of labor unions, which in our case is periodic obligations promptly, from taking appropriate measures
the BLR, is tantamount to dissolution of the organization by to correct its omissions. For the record, we do not view with favor
administrative authority when such measure would give rise to appellee’s late submission. Punctuality on the part of the union
the loss of legal personality of the union or loss of advantages and its officers could have prevented this petition.41
necessary for it to carry out its activities, which is true in our
jurisdiction. Although the ILO has allowed such measure to be WHEREFORE, premises considered, the Court of Appeals
taken, provided that judicial safeguards are in place, i.e., the right Decision dated May 30, 2005 and Resolution dated June 4, 2007
to appeal to a judicial body, it has nonetheless reminded its are AFFIRMED.
members that dissolution of a union, and cancellation of
registration for that matter, involve serious consequences for SO ORDERED.
occupational representation. It has, therefore, deemed it preferable
if such actions were to be taken only as a last resort and after
exhausting other possibilities with less serious effects on the
organization.40
[G.R. Nos. 100342-44. October 29, 1999.]
The aforesaid amendments and the ILO’s opinion on this matter RURAL BANK OF ALAMINOS EMPLOYEES UNION
serve to fortify our ruling in this case. We therefore quote with (RBAEU) and ISMAEL TAMAYO, SR., Petitioner, v.
approval the DOLE Secretary’s rationale for denying the petition, NATIONAL LABOR RELATIONS COMMISSION, Third
thus: Division, Exec. Labor Arbiter JOSE B. BOLISAY and
RURAL BANK OF ALAMINOS, Inc., Respondents.
It is undisputed that appellee failed to submit its annual financial
reports and list of individual members in accordance with Article DECISION
239 of the Labor Code. However, the existence of this ground
should not necessarily lead to the cancellation of union
registration. Article 239 recognizes the regulatory authority of the PURISIMA, J.:
NO. SUB-RAB-01-04-7-0059-89.

Before The Court is a Petition for Certiorari under Rule 65 of the The Union, assailing the alleged constructive dismissal of its
Revised Rules of Court to nullify and set aside the Resolution of members brought about or resulting from the strike, lodged
the National Labor Relations Commission, dated January 31, against RBAI a complaint for unfair labor practice with prayer for
1991, and the subsequent Resolution of March 26, 1991 denying moral and exemplary damages. This complaint has been docketed
petitioner’s motion for reconsideration, for being tainted with as NLRC CASE NO. SUB-RAB-01-06-7-0097-89.
grave abuse of discretion amounting to lack or excess of
jurisdiction. The assailed Resolutions set aside the consolidated The identity of the parties led the Labor Arbiter, Ricardo N.
decision of Labor Arbiter Ricardo N. Olairez and remanded the Olairez, to consolidate the three (3) aforementioned cases.
cases to the Regional Arbitration Branch of origin for further
proceedings.chanrobles virtual lawlibrary On December 14, 1989, Labor Arbiter Ricardo N. Olairez
rendered a consolidated decision.chanrobles virtual lawlibrary
The Petition stems from three cases originally instituted before
Sub-Regional Arbitration Branch No. 1 of the National Labor With respect to Ismael P. Tamayo’s complaint for illegal
Relations Commission in Dagupan City. The first case, NLRC dismissal against RBAI, the Labor Arbiter held that complainant,
Case No. 01-03-7-0049-89, was commenced by the herein whose nature of work, that of internal auditor, was usually
petitioner, Ismael Tamayo, Sr., against Rural Bank of Alaminos, necessary and desirable in the business engaged in by the
Inc. (RBAI) for illegal dismissal and damages. The second case, respondent, was a regular employee whose summary discharge
docketed as NLRC Case No. 01-04-7-0059-89, was filed by the from the service effective January 1, 1989, was illegal.
herein private respondent, Rural Bank of Alaminos, Inc., against
the Rural Bank of Alaminos Employees Union for unfair labor The dismissal being illegal, complainant should have ordinarily
practice, declaration of illegality of strike and damages. While the been reinstated to his former position. However, the Labor
third case, docketed as NLRC Case No. 01-06-0097-89, was filed Arbiter, finding the complainant to have reached the retirable age,
by the Employees Union against the Bank, charging the latter opted instead to award the latter the following amounts, to wit:
with unfair labor practice and damages. P63,442.56 representing full backwages/benefits; P29,822.80 as
retirement pay; and P9,326.53 as attorney’s fees, or a grand total
As gathered by the respondent National Labor Relations of P102, 591.89 plus one (1) percent interest per month until
Commission, the facts of the controversy litigated upon are as actually paid.
follows:chanrob1es virtual 1aw library
Regarding the bank’s petition, NLRC Case No. SUB-RAB-01-04-
With the appointment of one Benefredo Quinto to the position of 7-0059-89, the Labor Arbiter disposed of the same by holding
internal auditor, which position he had held since January 1, that the strike staged by the Union was legal and not in violation
1976, Ismael P. Tamayo, Sr., who had been with Rural Bank of of any provision of the Labor Code. Hence, the dismissal of the
Alaminos, Inc. (RBAI for brevity) since it started operations in petition.
September of 1956, feeling shortchanged, filed on June 3, 1988 a
complaint against RBAI for illegal dismissal. In the third case, NLRC CASE NO. SUB-RAB-01-06-0097-89,
the Labor Arbiter held the bank accountable for the full
In an effort to buy peace, that is, to settle the case amicably, backwages and other benefits due the Union members who he
RBAI agreed on a compromise agreement dated July 13, 1988 to found to have been constructively dismissed during the strike.
reinstate Ismael P. Tamayo, Sr. to the position of internal
auditor.chanrobles Moreover, the Labor Arbiter, on the Union’s claim for damages
of not less than P200,000.00, ruled that the award of P10,000.00
Claiming that his services were not actually needed, RBAI moral damages and P5,000.00 exemplary damages to each of the
terminated effective January 1, 1989 Ismael P. Tamayo, Sr.’s union member is in order.
services. This led to the filing on March 27, 1989 of a complaint
for illegal dismissal by Ismael P. Tamayo, Sr. against Rural Bank The Labor Arbiter disposed thus:jgc:chanrobles.com.ph
of Alaminos, Inc. docketed as NLRC Case No. SUB-RAB-01-03-
7-0049-89. "WHEREFORE, with all the foregoing considerations, judgment
is hereby rendered as follows:chanrob1es virtual 1aw library
Subsequent to its certification on December 12, 1989 as the sole
bargaining agent of the employees of RBAI, the Rural Bank of 1. In the first case, we find complainant Ismael Tamayo, Sr.
Alaminos Employees Union (hereinafter called the Union) illegally and unjustly dismissed and we hereby order the
submitted sometime in February 1989 proposals with respect to respondent Rural Bank of Alaminos, Inc. to pay him as
salary/wage increases. follows:chanrob1es virtual 1aw library

RBAI’s counter-proposals not (sic) acceptable to it, the Union, P 63,442.56 — Total full backwages/benefits
which had earlier filed a notice of strike on March 3, 1989, went
on strike on April 3, 1989. 29,822.80 — Retirement pay

Its position being that the strike staged by the Union is illegal and ————
in violation of Article 248 (e) of the Labor Code, RBAI instituted
a petition for the declaration of the strike as illegal and for actual P 93,265.36
damages it incurred by way of loss of earnings to the tune of
P30,000.00 per day. This petition was docketed as NLRC CASE 9,326.53 — 10% Attorney’s fees
———— Whether or not the respondent NLRC Third Division committed
grave abuse of discretion and exceeded its jurisdiction amounting
P102,591.89 — Total award as of December 31, 1989 to lack of jurisdiction in remanding the case for further
proceedings, in spite of its finding which affirmed the ruling of
plus one (l) percent interest per month until the award is actually the Labor Arbiter that the strike is legal and where the complaint
paid. is for a declaration of illegality of the strike.

2. In the second case, we find the strike legal and the Union B. Case No. 0097-89:chanrob1es virtual 1aw library
having violated no provision of the Labor Code, the complaint of
the Rural Bank of Alaminos, Inc. is hereby dismissed for lack of Whether or not the respondent NLRC Third Division committed
merit.chanroblesvirtuallawlibrary grave abuse of discretion and exceeded its jurisdiction amounting
to lack of jurisdiction in remanding the case for further
3. In the third case, we find the respondent Rural Bank of proceedings, despite the fact that the private respondent failed to
Alaminos, Inc. guilty of unfair labor practice, whose act is appeal the Labor Arbiter’s finding that the respondent is guilty of
tantamount to an illegal lockout amounting to a constructive unfair labor practice thus the said issue not raised on appeal had
dismissal of the Union members, and we hereby order the bank to become final.
pay them their full backwages and other benefits for the nine (9)
months period from April to December 1989 with the Whether or not the respondent NLRC Third Division committed
computations to include the wage increase under R.A. 6727 grave abuse of discretion and exceeded its jurisdiction amounting
effective July 1, 1989. to lack of jurisdiction in remanding the case for further
proceedings for the reason that it could not resolve the issues
The Bank is hereby ordered to re-open and accept/reinstate the squarely because it was at a loss as to the exact number of the
striking union members without loss of seniority rights and the bank’s employees, but which is contrary to the record of the case,
union members are likewise ordered to return to work and may as evidenced by the list of employees in private respondent’s
now claim their respective 13th month pay for 1989. In case they Annex "A" .
are not immediately reinstated, their full backwages shall not
exceed a maximum of three (3) years. C. Case No. 0049-89:chanrob1es virtual 1aw library

The respondent bank is further ordered to pay the Union members Whether or not respondent NLRC Third Division committed
P10,000.00 as moral damages and P5,000.00 as exemplary grave abuse of discretion amounting to lack of jurisdiction where
damages to each of the Union members, plus attorney’s fees and it ruled that private respondent was denied the right to cross-
litigation expenses of ten (10) percent of the total awards. examine petitioner Ismael Tamayo, in spite of the express
mandate of Article 221 of the Labor Code and the 90-day Rule
The order of reinstatement or return to work is immediately under Executive Order No.109 (1-23-89)
executory, hence the Bank is commanded to reopen its banking
business immediately. D. In all three cases:chanrob1es virtual 1aw library

SO ORDERED" Whether or not the right of petitioners to speedy disposition of


labor justice has been violated in the remand of the cases for
Dissatisfied with the disposition of the Labor Arbiter, the Rural further proceedings and consequently, whether a writ of
Bank of Alaminos, Inc. appealed to the National Labor Relations prohibition shall lie.chanroblesvirtual|awlibrary
Commission, which promulgated, on January 31, 1991 its assailed
Resolution setting aside the ruling of the Labor Arbiter and Whether or not respondent NLRC Third Division committed
ordering the remand of all the three cases to grave abuse of discretion amounting to lack of jurisdiction when
wit:jgc:chanrobles.com.ph it denied the motion for reconsideration for being filed out of
time, despite the fact that neither petitioner Ismael Tamayo, Sr.
". . . In the broader interest of justice, We deem it best to remand nor his deceased counsel was furnished with a copy of the
all the afore-numbered cases to Regional Arbitration Branch of resolution, and likewise the copy for the petitioner union was
origin for further proceedings. served to a stranger who is not an employee of the law office of
petitioner’s counsel.
WHEREFORE, premises considered, all the aforenumbered cases
are hereby remanded to the Regional Arbitration Branch of origin Respondent NLRC ordered the remand of all the three cases to
for further proceedings. the Labor Arbiter for further proceedings: opining that with
respect to NLRC Case No. 0049-89, respondent Bank should
SO ORDERED." chanrobles virtual lawlibrary have been given an opportunity to cross-examine the petitioner,
Ismael Tamayo, Sr., as to the veracity of the allegations contained
The reversal by the respondent Commission of the Labor in his unverified position paper, the lack or absence of which
Arbiter’s original Resolution prompted petitioners to bring the amounted to a denial of due process. As regards, Case No. 097-
present petition imputing grave abuse of discretion amounting to 89, the NLRC held that the finding by the Labor Arbiter of an
lack of or excess jurisdiction to the respondent Commission, illegal lock-out was not substantiated by evidence, as it was found
particularly describing the errors under attack:chanrob1es virtual out that no proof was ever adduced by the Union to show that the
1aw library bank refused them employment during the pendency of the strike,
thus necessitating the remand of the case to the Labor Arbiter for
A. Case No. 0059-89:chanrob1es virtual 1aw library reception of evidence. So also, the NLRC ruled that a complete
disposition of the case could not be had since there was nothing in
the record which indicates the number of employees A "lock-out" means the temporary refusal of an employer to
constructively dismissed by the respondent Bank. furnish work as a result of an industrial or labor dispute. 2 As
correctly found by the NLRC, in the case under consideration
Before delving into the merits of the case, it should be evidence of illegal lock-out is wanting such that there can be no
remembered that in the decision in the case of St. Martin Funeral conclusive determination by the NLRC as to the charge.
Homes v. National Labor Relations Commission, G.R. No. Petitioners failed to present sufficient proof to support the
130866, promulgated on September 16, 1998, this Court allegation of illegal lock-out. No evidence was adduced by the
pronounced that petitions for certiorari relating to NLRC Union to show that the Bank really refused them employment
decisions must be filed directly with the Court of Appeals, and during the pendency of the strike. As to the allegation that the
labor cases pending before this Court should be referred to the Bank was interfering with and restraining the employees in the
appellate court for proper disposition. However, in cases where exercise of their right to self-organization, suffice it to state that
the Memoranda of both parties have been filed with this Court filing a petition for cancellation of the Union’s registration is not
prior to the promulgation of the St. Martin decision, the Court per se an act of unfair labor practice. It must be shown by
generally opts to take the case itself for its final disposition. substantial evidence that the filing of the petition for cancellation
of union registration by the employer was aimed to oppress the
With respect to the first assigned error, petitioners contend that it Union. Consequently, the NLRC was right in ordering the remand
was an error for NLRC to remand Case No. 0059-89 to the Labor of Case No. 0097-89 for further proceedings.
Arbiter as the issue of legality of subject strike has been resolved
in favor of the Union by the Arbiter and the Commission. Anent the observation of NLRC that it was at a loss as to the
exact number of employees who were constructively dismissed by
There is merit in petitioners’ contention. NLRC Case No. 0059- the Bank, such claim is belied by the records clearly indicating
89 was filed by the respondent Bank against the Union, for that in its Complaint in NLRC Case. 0059-89, petitioner Union
declaration of illegality of the strike, unfair labor practice and did attach a letter addressed to the respondent Bank containing a
damages. In the proceedings below, the Labor Arbiter found that list of the bank’s employees together with their length of service
the strike conducted by the Union was legal and complied with all and monthly basic salary. 3 Respondent avers that since the said
the requirements of law. Such finding was, in fact, affirmed by list was presented in evidence in Case No. 0059-89, the same
the NLRC in the following Resolution, to wit:chanroblesvirtual| could not be considered as evidence in Case No. 0097-89 because
awlibrary these two cases are separate and distinct from each other.

". . . It appears that the Union filed its notice of strike on March 3, The contention is untenable. It must be recalled that Case No.
1989 and that it commenced its strike thirty (30) days thereafter, 0097-89 was filed in the nature of a countercharge to Case No.
or on April 3, 1989. 0059-89 by the petitioner Union against respondent Bank.
Besides, all the three cases were consolidated before the Labor
Obviously, the Union had duly observed the mandatory cooling- Arbiter because of the identity of the parties. Thus, the list,
off period such that the strike it eventually undertook complied although introduced in Case No. 0059-89, could likewise be
with what is required by the Labor Code. Hence, Our finding that properly considered as evidence in Case No. 0097-89. As held by
the strike is legal."cralaw virtua1aw library this Court, proceedings before a labor arbiter are non-litigious in
nature in which, subject to the requirements of due process, the
It is well-settled that when findings of fact by the labor arbiter are technicalities of law and procedure and the rules obtaining in
sufficiently supported by the evidence on record, the same must courts of law do not strictly apply. 4
be accorded due respect by this Court. 1 More so when such
findings by the Labor Arbiter are affirmed by the NLRC on Petitioners’ stance that the finding of unfair labor practice already
appeal. Since the NLRC found the strike conducted by the Union became final as the issue was not raised on appeal, is untenable.
legal, the Court finds no justifiable reason for the Commission to The first, fourth and fifth issues raised by the Bank in its Appeal
remand Case No. 0059-89 to the Labor Arbiter for further Memorandum filed with the NLRC on December 24, 1989
proceedings. The allegation of unfair labor practice and the claim theorized that:chanrobles law library : red
for damages proceed from and are consequences of the strike, the
findings of which are based on the legality or illegality thereof. "1. The Labor Arbiter gravely abused his discretion in holding
The strike thus being adjudged as legal, the charges of unfair that appellant violated its duty to bargain collectively.
labor practice and damages are thereby negated and bereft of any
basis. Therefore, the NLRC gravely abused its discretion when it x       x       x
ordered the remand of NLRC Case No. 0059-89 to the Labor
Arbiter for further proceedings.
4. The Labor Arbiter gravely abused his discretion in holding that
Anent the second assigned error which pertains to NLRC Case appellant is guilty of illegal lock-out.
No. 0097-89, the case instituted by the Union against the
respondent Bank for unfair labor practice with damages, the Court 5. The Labor Arbiter gravely abused his discretion in holding that
believes, and so holds, that the remand of said case was in appellant illegally dismissed its employees.
order.chanroblesvirtuallawlibrary:red
x       x       x." 5
NLRC Case No. 0097-89 charged RBAI with unfair labor
practice and the Labor Arbiter concluded that the Bank employed Although the issue was not collectively appealed as unfair labor
all available means to further delay the resolution of the dispute, practice, the first, fourth and fifth issues relate to acts which by
thus creating a scenario of an illegal lock-out. themselves constitute unfair labor practice.
SAN MIGUEL CORPORATION EMPLOYEES UNION
As regards the third assigned error, petitioners maintain that the PHILIPPINE TRANSPORT AND GENERAL WORKERS
NLRC acted with grave abuse of discretion in remanding NLRC ORGANIZATION (SMCEU PTGWO), Petitioner, v. SAN
Case No. 0049-89 for further proceedings because the Labor MIGUEL PACKAGING PRODUCTS EMPLOYEES UNION
Arbiter denied respondent Bank’s right to cross-examine PAMBANSANG DIWA NG MANGGAGAWANG PILIPINO
petitioner Ismael Tamayo, Sr. Respondent NLRC, on the other (SMPPEU PDMP), Respondent 1 .
hand, ruled that the Labor Arbiter should have granted respondent
Bank the right to cross-examine the said petitioner on the veracity DECISION
of the allegations in his unverified position paper, and it was
grave abuse of discretion not to allow respondent Bank to cross-
CHICO-NAZARIO, J.:
examine petitioner Tamayo.

In a long line of cases, this Court has held that the holding of a In this Petition for Review on Certiorari under Rule 45 of the
trial is discretionary on the part of the Labor Arbiter, and it cannot Revised Rules of Court, petitioner SAN MIGUEL
be demanded as a matter of right by the parties. 6 The absence of CORPORATION EMPLOYEES UNION-PHILIPPINE
a formal hearing or trial before the Labor Arbiter is no cause for a TRANSPORT AND GENERAL WORKERS ORGANIZATION
party to impute grave abuse of discretion. 7 The submission of (SMCEU-PTGWO) prays that this Court reverse and set aside the
position papers and memoranda in labor cases satisfies the (a) Decision2 dated 9 March 2005 of the Court of Appeals in CA-
requirements of due process, and a decision rendered on the basis G.R. SP No. 66200, affirming the Decision3 dated 19 February
of the position papers which were found to be sufficient, meets 2001 of the Bureau of Labor Relations (BLR) of the Department
the requirements of a fair and open hearing. 8 Thus, in the case of Labor and Employment (DOLE) which upheld the Certificate
under scrutiny, the Labor Arbiter did not act with grave abuse his of Registration of respondent SAN MIGUEL PACKAGING
discretion in not conducting a formal hearing or trial and in PRODUCTS EMPLOYEES UNION PAMBANSANG DIWA
basing his decision solely on the position papers submitted by the NG MANGGAGAWANG PILIPINO (SMPPEU PDMP); and (b)
parties. The fact that the position paper submitted by petitioner the Resolution4 dated 16 January 2006 of the Court of Appeals in
Tamayo was not verified is of no moment. Succinct and clear is the same case, denying petitioner's Motion for Reconsideration of
the ruling of this Court that the lack of a verification of a position the aforementioned Decision.
paper is only a formal and not a jurisdictional defect. 9 It is not
fatal and could be easily corrected by requiring an oath. 10 The following are the antecedent facts:

Petitioner Tamayo faults respondent NLRC for denying his Petitioner is the incumbent bargaining agent for the bargaining
motion for reconsideration for the reason that it was filed out of unit comprised of the regular monthly-paid rank and file
time. He contends that neither he nor his deceased former employees of the three divisions of San Miguel Corporation
counsel, Atty. de Vera was furnished a copy of the NLRC’s (SMC), namely, the San Miguel Corporate Staff Unit (SMCSU),
resolution.chanroblesvirtuallawlibrary San Miguel Brewing Philippines (SMBP), and the San Miguel
Packaging Products (SMPP), in all offices and plants of SMC,
Petitioner’s allegation is not meritorious. It is axiomatic that including the Metal Closure and Lithography Plant in Laguna. It
notice to counsel is notice to parties and when a party is had been the certified bargaining agent for 20 years - from 1987
represented by counsel, notices should be made upon the counsel to 1997.
of record at his given address, to which notices of all kinds
emanating from the court should be sent. 11 In the appeal before
the respondent Commission, it was Atty. Teofilo Humilde who Respondent is registered as a chapter of Pambansang Diwa ng
entered appearance in behalf of the Union and petitioner Tamayo. Manggagawang Pilipino (PDMP). PDMP issued Charter
It was thus reasonable for the NLRC to send a copy of the NLRC Certificate No. 112 to respondent on 15 June 1999.5 In
Resolution to the said lawyer. Since the said resolution was compliance with registration requirements, respondent submitted
received by counsel on February 26, 1991 and the motion for the requisite documents to the BLR for the purpose of acquiring
reconsideration was filed only on March 13, 1991, the denial by legal personality.6 Upon submission of its charter certificate and
the NLRC was in order, the ten-day period for filing a motion for other documents, respondent was issued Certificate of Creation of
reconsideration having lapsed. Local or Chapter PDMP-01 by the BLR on 6 July
1999.7 Thereafter, respondent filed with the Med-Arbiter of the
WHEREFORE, the petition is partly GRANTED in that the Order DOLE Regional Officer in the National Capital Region (DOLE-
of the NLRC remanding NLRC Cases No. 0049-89 and No. 0059- NCR), three separate petitions for certification election to
89 to the Labor Arbiter is SET ASIDE but, the Order remanding represent SMPP, SMCSU, and SMBP.8 All three petitions were
NLRC Case No. 0097-89 to said Labor Arbiter for further dismissed, on the ground that the separate petitions fragmented a
proceedings, is UPHELD. No pronouncement as to costs. single bargaining unit.9

SO ORDERED. On 17 August 1999, petitioner filed with the DOLE-NCR a


petition seeking the cancellation of respondent's registration and
its dropping from the rolls of legitimate labor organizations. In its
petition, petitioner accused respondent of committing fraud and
falsification, and non-compliance with registration requirements
in obtaining its certificate of registration. It raised allegations that
respondent violated Articles 239(a), (b) and (c)10 and 234(c)11 of
[G.R. NO. 171153 : September 12, 2007]
the Labor Code. Moreover, petitioner claimed that PDMP is not a
legitimate labor organization, but a trade union center, hence, it
cannot directly create a local or chapter. The petition was in the roster of legitimate labor organizations. The [herein
docketed as Case No. NCR-OD-9908-007-IRD.12 respondent] is an affiliate of a registered federation PDMP,
having been issued a charter certificate. Under the rules we have
On 14 July 2000, DOLE-NCR Regional Director Maximo B. Lim reviewed, there is no need for SMPPEU to show a membership of
issued an Order dismissing the allegations of fraud and 20% of the employees of the bargaining unit in order to be
misrepresentation, and irregularity in the submission of recognized as a legitimate labor union.
documents by respondent. Regional Director Lim further ruled
that respondent is allowed to directly create a local or chapter. xxx
However, he found that respondent did not comply with the 20%
membership requirement and, thus, ordered the cancellation of its In view of the foregoing, the assailed decision and resolution of
certificate of registration and removal from the rolls of legitimate the BLR are AFFIRMED, and the petition is DISMISSED.16
labor organizations.13 Respondent appealed to the BLR. In a
Decision dated 19 February 2001, it declared:
Subsequently, in a Resolution dated 16 January 2006, the Court of
Appeals denied petitioner's Motion for Reconsideration of the
As a chartered local union, appellant is not required to submit the aforementioned Decision.
number of employees and names of all its members comprising at
least 20% of the employees in the bargaining unit where it seeks
Hence, this Petition for Certiorari under Rule 45 of the Revised
to operate. Thus, the revocation of its registration based on non-
Rules of Court where petitioner raises the sole issue of:
compliance with the 20% membership requirement does not have
any basis in the rules.
WHETHER OR NOT THE HONORABLE COURT OF
APPEALS COMMITTED REVERSIBLE ERROR IN RULING
Further, although PDMP is considered as a trade union center, it
THAT PRIVATE RESPONDENT IS NOT REQUIRED TO
is a holder of Registration Certificate No. FED-11558-LC issued
SUBMIT THE NUMBER OF EMPLOYEES AND NAMES OF
by the BLR on 14 February 1991, which bestowed upon it the
ALL ITS MEMBERS COMPRISING AT LEAST 20% OF THE
status of a legitimate labor organization with all the rights and
EMPLOYEES IN THE BARGAINING UNIT WHERE IT
privileges to act as representative of its members for purposes of
SEEKS TO OPERATE.
collective bargaining agreement. On this basis, PDMP can
charter or create a local, in accordance with the provisions of
Department Order No. 9. The present petition questions the legal personality of respondent
as a legitimate labor organization.
WHEREFORE, the appeal is hereby GRANTED. Accordingly,
the decision of the Regional Director dated July 14, 2000, Petitioner posits that respondent is required to submit a list of
canceling the registration of appellant San Miguel Packaging members comprising at least 20% of the employees in the
Products Employees Union-Pambansang Diwa ng bargaining unit before it may acquire legitimacy, citing Article
Manggagawang Pilipino (SMPPEU-PDMP) is REVERSED and 234(c) of the Labor Code which stipulates that any applicant labor
SET ASIDE. Appellant shall hereby remain in the roster of organization, association or group of unions or workers shall
legitimate labor organizations.14 acquire legal personality and shall be entitled to the rights and
privileges granted by law to legitimate labor organizations upon
issuance of the certificate of registration based on the following
While the BLR agreed with the findings of the DOLE Regional
requirements:
Director dismissing the allegations of fraud and
misrepresentation, and in upholding that PDMP can directly
create a local or a chapter, it reversed the Regional Director's A. Fifty pesos (P50.00) registration fee;
ruling that the 20% membership is a requirement for respondent
to attain legal personality as a labor organization. Petitioner b. The names of its officers, their addresses, the principal address
thereafter filed a Motion for Reconsideration with the BLR. In a of the labor organization, the minutes of the organizational
Resolution rendered on 19 June 2001 in BLR-A-C-64-05-9-00 meetings and the list of the workers who participated in such
(NCR-OD-9908-007-IRD), the BLR denied the Motion for meetings;
Reconsideration and affirmed its Decision dated 19 February
2001.15 c. The names of all its members comprising at least twenty
percent (20%) of all the employees in the bargaining unit where it
Invoking the power of the appellate court to review decisions of seeks to operate;
quasi-judicial agencies, petitioner filed with the Court of Appeals
a Petition for Certiorari under Rule 65 of the 1997 Rules of Civil d. If the applicant union has been in existence for one or more
Procedure docketed as CA-G.R. SP No. 66200. The Court of years, copies of its annual financial reports; andcralawlibrary
Appeals, in a Decision dated 9 March 2005, dismissed the petition
and affirmed the Decision of the BLR, ruling as follows:
e. Four (4) copies of the constitution and by-laws of the applicant
union, minutes of its adoption or ratification and the list of the
In Department Order No. 9, a registered federation or national members who participated in it.17
union may directly create a local by submitting to the BLR copies
of the charter certificate, the local's constitution and by-laws, the
principal office address of the local, and the names of its officers Petitioner also insists that the 20% requirement for registration of
and their addresses. Upon complying with the documentary respondent must be based not on the number of employees of a
requirements, the local shall be issued a certificate and included single division, but in all three divisions of the company in all the
offices and plants of SMC since they are all part of one
bargaining unit. Petitioner refers to Section 1, Article 1 of the since respondent's petition for certification election was filed with
Collective Bargaining Agreement (CBA),18 quoted hereunder: the BLR in 1999; and that of petitioner on 17 August 1999.26

ARTICLE 1 The applicable Implementing Rules enunciates a two-fold


procedure for the creation of a chapter or a local. The first
SCOPE involves the affiliation of an independent union with a federation
or national union or industry union. The second, finding
application in the instant petition, involves the direct creation of a
Section 1. Appropriate Bargaining Unit. The appropriate
local or a chapter through the process of chartering.27
bargaining unit covered by this Agreement consists of all regular
rank and file employees paid on the basis of fixed salary per
month and employed by the COMPANY in its Corporate Staff A duly registered federation or national union may directly create
Units (CSU), San Miguel Brewing Products (SMBP) and San a local or chapter by submitting to the DOLE Regional Office or
Miguel Packaging Products (SMPP) and in different operations to the BLR two copies of the following:
existing in the City of Manila and suburbs, including Metal
Closure and Lithography Plant located at Canlubang, Laguna (a) A charter certificate issued by the federation or national union
subject to the provisions of Article XV of this Agreement indicating the creation or establishment of the local/chapter;
provided however, that if during the term of this Agreement, a
plant within the territory covered by this Agreement is transferred (b) The names of the local/chapter's officers, their addresses, and
outside but within a radius of fifty (50) kilometers from the Rizal the principal office of the local/chapter; andcralawlibrary
Monument, Rizal Park, Metro Manila, the employees in the
transferred plant shall remain in the bargaining unit covered by
(c) The local/chapter's constitution and by-laws; Provided, That
this Agreement. (Emphasis supplied.)
where the local/chapter's constitution and by-laws is the same as
that of the federation or national union, this fact shall be indicated
Petitioner thus maintains that respondent, in any case, failed to accordingly.
meet this 20% membership requirement since it based its
membership on the number of employees of a single division
All the foregoing supporting requirements shall be certified under
only, namely, the SMPP.
oath by the Secretary or the Treasurer of the local/chapter and
attested to by its President.28
There is merit in petitioner's contentions.
The Implementing Rules stipulate that a local or chapter may be
A legitimate labor organization19 is defined as "any labor directly created by a federation or national union. A duly
organization duly registered with the Department of Labor and constituted local or chapter created in accordance with the
Employment, and includes any branch or local thereof."20 The foregoing shall acquire legal personality from the date of filing of
mandate of the Labor Code is to ensure strict compliance with the the complete documents with the BLR.29 The issuance of the
requirements on registration because a legitimate labor certificate of registration by the BLR or the DOLE Regional
organization is entitled to specific rights under the Labor Office is not the operative act that vests legal personality upon a
Code,21 and are involved in activities directly affecting matters of local or a chapter under Department Order No. 9. Such legal
public interest. Registration requirements are intended to afford a personality is acquired from the filing of the complete
measure of protection to unsuspecting employees who may be documentary requirements enumerated in Section 1, Rule VI.30
lured into joining unscrupulous or fly-by-night unions whose sole
purpose is to control union funds or use the labor organization for
Petitioner insists that Section 3 of the Implementing Rules, as
illegitimate ends.22 Legitimate labor organizations have exclusive
amended by Department Order No. 9, violated Article 234 of the
rights under the law which cannot be exercised by non-legitimate
Labor Code when it provided for less stringent requirements for
unions, one of which is the right to be certified as the exclusive
the creation of a chapter or local. This Court disagrees.
representative23 of all the employees in an appropriate collective
bargaining unit for purposes of collective bargaining.24 The
acquisition of rights by any union or labor organization, Article 234 of the Labor Code provides that an independent labor
particularly the right to file a petition for certification election, organization acquires legitimacy only upon its registration with
first and foremost, depends on whether or not the labor the BLR:
organization has attained the status of a legitimate labor
organization.25 Any applicant labor organization, association or group of unions
or workers shall acquire legal personality and shall be entitled to
A perusal of the records reveals that respondent is registered with the rights and privileges granted by law to legitimate labor
the BLR as a "local" or "chapter" of PDMP and was issued organizations upon issuance of the certificate of registration based
Charter Certificate No. 112 on 15 June 1999. Hence, respondent on the following requirements:
was directly chartered by PDMP.
(a) Fifty pesos (P50.00) registration fee;
The procedure for registration of a local or chapter of a labor
organization is provided in Book V of the Implementing Rules of (b) The names of its officers, their addresses, the principal address
the Labor Code, as amended by Department Order No. 9 which of the labor organization, the minutes of the organizational
took effect on 21 June 1997, and again by Department Order No. meetings and the list of the workers who participated in such
40 dated 17 February 2003. The Implementing Rules as amended meetings;
by D.O. No. 9 should govern the resolution of the petition at bar
(c) The names of all its members comprising at least twenty Still, petitioner postulates that respondent was not validly and
percent (20%) of all the employees in the bargaining unit where legitimately created, for PDMP cannot create a local or chapter as
it seeks to operate; it is not a legitimate labor organization, it being a trade union
center.
(d) If the applicant union has been in existence for one or more
years, copies of its annual financial reports; andcralawlibrary Petitioner's argument creates a predicament as it hinges on the
legitimacy of PDMP as a labor organization. Firstly, this line of
(e) Four (4) copies of the constitution and by-laws of the reasoning attempts to predicate that a trade union center is not a
applicant union, minutes of its adoption or ratification, and the list legitimate labor organization. In the process, the legitimacy of
of the members who participated in it. (Italics supplied.) PDMP is being impugned, albeit indirectly. Secondly, the same
contention premises that a trade union center cannot directly
create a local or chapter through the process of chartering.
It is emphasized that the foregoing pertains to the registration of
an independent labor organization, association or group of unions
or workers. Anent the foregoing, as has been held in a long line of cases, the
legal personality of a legitimate labor organization, such as
PDMP, cannot be subject to a collateral attack. The law is very
However, the creation of a branch, local or chapter is treated
clear on this matter. Article 212 (h) of the Labor Code, as
differently. This Court, in the landmark case of Progressive
amended, defines a legitimate labor organization37 as "any labor
Development Corporation v. Secretary, Department of Labor and
organization duly registered with the DOLE, and includes any
Employment,31 declared that when an unregistered union becomes
branch or local thereof."38 On the other hand, a trade union center
a branch, local or chapter, some of the aforementioned
is any group of registered national unions or federations
requirements for registration are no longer necessary or
organized for the mutual aid and protection of its members; for
compulsory. Whereas an applicant for registration of an
assisting such members in collective bargaining; or for
independent union is mandated to submit, among other things, the
participating in the formulation of social and employment
number of employees and names of all its members comprising at
policies, standards, and programs, and is duly registered with the
least 20% of the employees in the bargaining unit where it seeks
DOLE in accordance with Rule III, Section 2 of the Implementing
to operate, as provided under Article 234 of the Labor Code and
Rules.39
Section 2 of Rule III, Book V of the Implementing Rules, the
same is no longer required of a branch, local or chapter.32 The
intent of the law in imposing less requirements in the case of a The Implementing Rules stipulate that a labor organization shall
branch or local of a registered federation or national union is to be deemed registered and vested with legal personality on the date
encourage the affiliation of a local union with a federation or of issuance of its certificate of registration. Once a certificate of
national union in order to increase the local union's bargaining registration is issued to a union, its legal personality cannot be
powers respecting terms and conditions of labor.33 subject to collateral attack.40 It may be questioned only in an
independent petition for cancellation in accordance with Section 5
of Rule V, Book V of the Implementing Rules. The
Subsequently, in Pagpalain Haulers, Inc. v. Trajano34 where the
aforementioned provision is enunciated in the following:
validity of Department Order No. 9 was directly put in issue, this
Court was unequivocal in finding that there is no inconsistency
between the Labor Code and Department Order No. 9. Sec. 5. Effect of registration. The labor organization or workers'
association shall be deemed registered and vested with legal
personality on the date of issuance of its certificate of registration.
As to petitioner's claims that respondent obtained its Certificate of
Such legal personality cannot thereafter be subject to collateral
Registration through fraud and misrepresentation, this Court finds
attack, but may be questioned only in an independent petition for
that the imputations are not impressed with merit. In the instant
cancellation in accordance with these Rules.
case, proof to declare that respondent committed fraud and
misrepresentation remains wanting. This Court had, indeed, on
several occasions, pronounced that registration based on false and PDMP was registered as a trade union center and issued
fraudulent statements and documents confer no legitimacy upon a Registration Certificate No. FED-11558-LC by the BLR on 14
labor organization irregularly recognized, which, at best, holds on February 1991. Until the certificate of registration of PDMP is
to a mere scrap of paper. Under such circumstances, the labor cancelled, its legal personality as a legitimate labor organization
organization, not being a legitimate labor organization, acquires subsists. Once a union acquires legitimate status as a labor
no rights.35 organization, it continues to be recognized as such until its
certificate of registration is cancelled or revoked in an
independent action for cancellation.41 It bears to emphasize that
This Court emphasizes, however, that a direct challenge to the
what is being directly challenged is the personality of respondent
legitimacy of a labor organization based on fraud and
as a legitimate labor organization and not that of PDMP. This
misrepresentation in securing its certificate of registration is a
being a collateral attack, this Court is without jurisdiction to
serious allegation which deserves careful scrutiny. Allegations
entertain questions indirectly impugning the legitimacy of PDMP.
thereof should be compounded with supporting circumstances and
evidence. The records of the case are devoid of such evidence.
Furthermore, this Court is not a trier of facts, and this doctrine Corollarily, PDMP is granted all the rights and privileges
applies with greater force in labor cases. Findings of fact of appurtenant to a legitimate labor organization,42 and continues to
administrative agencies and quasi-judicial bodies, such as the be recognized as such until its certificate of registration is
BLR, which have acquired expertise because their jurisdiction is successfully impugned and thereafter cancelled or revoked in an
confined to specific matters, are generally accorded not only great independent action for cancellation.
respect but even finality.36
We now proceed to the contention that PDMP cannot directly composed of a group of registered national unions or
create a local or a chapter, it being a trade union center. federations.49

This Court reverses the finding of the appellate court and BLR on The Implementing Rules, as amended by Department Order No.
this ground, and rules that PDMP cannot directly create a local or 9, provide that "a duly registered federation or national union"
chapter. may directly create a local or chapter. The provision reads:

After an exhaustive study of the governing labor law provisions, Section 1. Chartering and creation of a local/chapter. - A duly
both statutory and regulatory,43 we find no legal justification to registered federation or national union may directly create a
support the conclusion that a trade union center is allowed to local/chapter by submitting to the Regional Office or to the
directly create a local or chapter through chartering. Apropos, we Bureau two (2) copies of the following:
take this occasion to reiterate the first and fundamental duty of
this Court, which is to apply the law. The solemn power and duty (a) A charter certificate issued by the federation or national union
of the Court to interpret and apply the law does not include the indicating the creation or establishment of the local/chapter;
power to correct by reading into the law what is not written
therein.44
(b) The names of the local/chapter's officers, their addresses, and
the principal office of the local/chapter; andcralawlibrary
Presidential Decree No. 442, better known as the Labor Code,
was enacted in 1972. Being a legislation on social justice,45 the
(c) The local/chapter's constitution and by-laws; provided that
provisions of the Labor Code and the Implementing Rules have
where the local/chapter's constitution and by-laws is the same as
been subject to several amendments, and they continue to evolve,
that of the federation or national union, this fact shall be indicated
considering that labor plays a major role as a socio-economic
accordingly.
force. The Labor Code was first amended by Republic Act No.
6715, and recently, by Republic Act No. 9481. Incidentally, the
term trade union center was never mentioned under Presidential All the foregoing supporting requirements shall be certified under
Decree No. 442, even as it was amended by Republic Act No. oath by the Secretary or the Treasurer of the local/chapter and
6715. The term trade union center was first adopted in the attested to by its President.50
Implementing Rules, under Department Order No. 9.
Department Order No. 9 mentions two labor organizations either
Culling from its definition as provided by Department Order No. of which is allowed to directly create a local or chapter through
9, a trade union center is any group of registered national unions chartering - a duly registered federation or a national union.
or federations organized for the mutual aid and protection of its Department Order No. 9 defines a "chartered local" as a labor
members; for assisting such members in collective bargaining; or organization in the private sector operating at the enterprise level
for participating in the formulation of social and employment that acquired legal personality through a charter certificate, issued
policies, standards, and programs, and is duly registered with the by a duly registered federation or national union and reported to
DOLE in accordance with Rule III, Section 2 of the Implementing the Regional Office in accordance with Rule III, Section 2-E of
Rules.46 The same rule provides that the application for these Rules.51
registration of an industry or trade union center shall be supported
by the following: Republic Act No. 9481 or "An Act Strengthening the Workers'
Constitutional Right to Self-Organization, Amending for the
(a) The list of its member organizations and their respective Purpose Presidential Decree No. 442, As Amended, Otherwise
presidents and, in the case of an industry union, the industry Known as the Labor Code of the Philippines" lapsed52 into law on
where the union seeks to operate; 25 May 2007 and became effective on 14 June 2007.53 This law
further amends the Labor Code provisions on Labor Relations.
(b) The resolution of membership of each member organization,
approved by the Board of Directors of such union; Pertinent amendments read as follows:

(c) The name and principal address of the applicant, the names of SECTION 1. Article 234 of Presidential Decree No. 442, as
its officers and their addresses, the minutes of its organizational amended, otherwise known as the Labor Code of the Philippines,
meeting/s, and the list of member organizations and their is hereby further amended to read as follows:
representatives who attended such meeting/s; andcralawlibrary
ART. 234. Requirements of Registration. - A federation, national
(d) A copy of its constitution and by-laws and minutes of its union or industry or trade union center or an independent union
ratification by a majority of the presidents of the member shall acquire legal personality and shall be entitled to the rights
organizations, provided that where the ratification was done and privileges granted by law to legitimate labor organizations
simultaneously with the organizational meeting, it shall be upon issuance of the certificate of registration based on the
sufficient that the fact of ratification be included in the minutes of following requirements:
the organizational meeting.47
(a) Fifty pesos (P50.00) registration fee;
Evidently, while a "national union" or "federation" is a labor
organization with at least ten locals or chapters or affiliates, each (b) The names of its officers, their addresses, the principal address
of which must be a duly certified or recognized collective of the labor organization, the minutes of the organizational
bargaining agent;48 a trade union center, on the other hand, is
meetings and the list of the workers who participated in such centers" in identifying the labor organizations allowed to charter a
meetings; chapter or local. Anything that is not included in the enumeration
is excluded therefrom, and a meaning that does not appear nor is
(c) In case the applicant is an independent union, the names of all intended or reflected in the very language of the statute cannot be
its members comprising at least twenty percent (20%) of all the placed therein.57 The rule is restrictive in the sense that it proceeds
employees in the bargaining unit where it seeks to operate; from the premise that the legislating body would not have made
specific enumerations in a statute if it had the intention not to
restrict its meaning and confine its terms to those expressly
(d) If the applicant union has been in existence for one or more
mentioned.58 Expressium facit cessare tacitum.59 What is
years, copies of its annual financial reports; andcralawlibrary
expressed puts an end to what is implied. Casus omissus pro
omisso habendus est. A person, object or thing omitted must have
(e) Four copies of the constitution and by-laws of the applicant been omitted intentionally.
union, minutes of its adoption or ratification, and the list of the
members who participated in it.
Therefore, since under the pertinent status and applicable
implementing rules, the power granted to labor organizations to
SECTION 2. A new provision is hereby inserted into the Labor directly create a chapter or local through chartering is given to a
Code as Article 234-A to read as follows: federation or national union, then a trade union center is without
authority to charter directly.
ART. 234-A. Chartering and Creation of a Local Chapter. - A
duly registered federation or national union may directly create a The ruling of this Court in the instant case is not a departure from
local chapter by issuing a charter certificate indicating the the policy of the law to foster the free and voluntary organization
establishment of the local chapter. The chapter shall acquire legal of a strong and united labor movement,60 and thus assure the
personality only for purposes of filing a petition for certification rights of workers to self-organization.61 The mandate of the Labor
election from the date it was issued a charter certificate. Code in ensuring strict compliance with the procedural
requirements for registration is not without reason. It has been
The chapter shall be entitled to all other rights and privileges of a observed that the formation of a local or chapter becomes a handy
legitimate labor organization only upon the submission of the tool for the circumvention of union registration requirements.
following documents in addition to its charter certificate: Absent the institution of safeguards, it becomes a convenient
device for a small group of employees to foist a not-so-desirable
(a) The names of the chapter's officers, their addresses, and the federation or union on unsuspecting co-workers and pare the need
principal office of the chapter; andcralawlibrary for wholehearted voluntariness, which is basic to free
unionism.62 As a legitimate labor organization is entitled to
specific rights under the Labor Code and involved in activities
(b) The chapter's constitution and by-laws: Provided, That where directly affecting public interest, it is necessary that the law afford
the chapter's constitution and by-laws are the same as that of the utmost protection to the parties affected.63 However, as this Court
federation or the national union, this fact shall be indicated has enunciated in Progressive Development Corporation v.
accordingly. Secretary of Department of Labor and Employment, it is not this
Court's function to augment the requirements prescribed by law.
The additional supporting requirements shall be certified under Our only recourse, as previously discussed, is to exact strict
oath by the secretary or treasurer of the chapter and attested by its compliance with what the law provides as requisites for local or
president. (Emphasis ours.) chapter formation.64

Article 234 now includes the term trade union center, but In sum, although PDMP as a trade union center is a legitimate
interestingly, the provision indicating the procedure for chartering labor organization, it has no power to directly create a local or
or creating a local or chapter, namely Article 234-A, still makes chapter. Thus, SMPPEU-PDMP cannot be created under the more
no mention of a "trade union center." lenient requirements for chartering, but must have complied with
the more stringent rules for creation and registration of an
Also worth emphasizing is that even in the most recent independent union, including the 20% membership requirement.
amendment of the implementing rules,54 there was no mention of
a trade union center as being among the labor organizations WHEREFORE, the instant Petition is GRANTED. The
allowed to charter. Decision dated 09 March 2005 of the Court of Appeals in CA-GR
SP No. 66200 is REVERSED and SET ASIDE. The Certificate
This Court deems it proper to apply the Latin maxim expressio of Registration of San Miguel Packaging Products Employees
unius est exclusio alterius. Under this maxim of statutory Union Pambansang Diwa ng Manggagawang Pilipino
interpretation, the expression of one thing is the exclusion of is ORDERED CANCELLED, and SMPPEU-
another. When certain persons or things are specified in a law, PDMP DROPPED from the rolls of legitimate labor
contract, or will, an intention to exclude all others from its organizations.
operation may be inferred. If a statute specifies one exception to a
general rule or assumes to specify the effects of a certain Costs against petitioner.
provision, other exceptions or effects are excluded.55 Where the
terms are expressly limited to certain matters, it may not, by SO ORDERED.
interpretation or construction, be extended to other
matters.56 Such is the case here. If its intent were otherwise, the
law could have so easily and conveniently included "trade union
Ynares-Santiago, J., Chairperson, Austria-Martinez, Nachura, election among the rank-and-file employees of respondent,
Reyes, JJ., concur. subject to the usual pre-election conference and inclusion-
exclusion proceedings.8

On January 23, 2003 or six days after the issuance of said


decision, respondent filed its comment on the motion for
reconsideration of petitioner, asserting that the order of the med-
[G.R. NO. 167141 : March 13, 2009]
arbiter could only be reviewed by way of appeal and not by a
motion for reconsideration pursuant to Department Order (D.O.)
SAMAHAN NG MGA MANGGAGAWA SA SAMMA- No. 9, series of 1997.9
LAKAS SA INDUSTRIYA NG KAPATIRANG HALIGI NG
ALYANSA (SAMMA-LIKHA), Petitioner, v. SAMMA
On February 6, 2003, respondent filed its motion for
CORPORATION, Respondent.
reconsideration of the January 17, 2003 decision. In a resolution
dated April 3, 2003, Secretary Patricia A. Sto. Tomas denied the
DECISION motion.10

CORONA, J.: Meanwhile, on April 14, 2003, Crispin D. Dannug, Jr., Officer-in-


Charge/Regional Director of DOLE Regional Office IV, issued a
This is a Petition for Review on Certiorari 1 of the August 31, resolution revoking the charter certificate of petitioner as local
2004 decision2 and February 15, 2005 resolution3 of the Court of chapter of LIKHA Federation on the ground of prohibited mixture
Appeals (CA) in CA-G.R. SP No. 77156. of supervisory and rank-and-file employees and non-compliance
with the attestation clause under paragraph 2 of Article 235 of the
Petitioner Samahan ng mga Manggagawa sa Samma' Lakas sa Labor Code.11 On May 6, 2003, petitioner moved for the
Industriya ng Kapatirang Haligi ng Alyansa (SAMMA-LIKHA) reconsideration of this resolution.12
filed a petition for certification election on July 24, 2001 in the
Department of Labor and Employment (DOLE), Regional Office Respondent filed a petition for certiorari 13 in the CA assailing
IV.4 It claimed that: (1) it was a local chapter of the LIKHA the January 17, 2003 decision and April 3, 2003 resolution of the
Federation, a legitimate labor organization registered with the Secretary of Labor. In a decision dated August 31, 2004, the CA
DOLE; (2) it sought to represent all the rank-and-file employees reversed the same.14 It denied reconsideration in a resolution
of respondent Samma Corporation; (3) there was no other dated February 15, 2005. It held that Administrative Circular No.
legitimate labor organization representing these rank-and-file 04-94 which required the filing of a certificate of non-forum
employees; (4) respondent was not a party to any collective shopping applied to petitions for certification election. It also
bargaining agreement and (5) no certification or consent election ruled that the Secretary of Labor erred in granting the appeal
had been conducted within the employer unit for the last 12 despite the lack of proof of service on respondent. Lastly, it found
months prior to the filing of the petition. that petitioner had no legal standing to file the petition for
certification election because its members were a mixture of
Respondent moved for the dismissal of the petition arguing that supervisory and rank-and-file employees.15
(1) LIKHA Federation failed to establish its legal personality; (2)
petitioner failed to prove its existence as a local chapter; (3) it Hence, this petition.
failed to attach the certificate of non-forum shopping and (4) it
had a prohibited mixture of supervisory and rank-and-file The issues for our resolution are the following: (1) whether a
employees.5 certificate for non-forum shopping is required in a petition for
certification election; (2) whether petitioner's motion for
In an order dated November 12, 2002, med-arbiter Arturo V. reconsideration which was treated as an appeal by the Secretary
Cosuco ordered the dismissal of the petition on the following of Labor should not have been given due course for failure to
grounds: (1) lack of legal personality for failure to attach the attach proof of service on respondent and (3) whether petitioner
certificate of registration purporting to show its legal personality; had the legal personality to file the petition for certification
(2) prohibited mixture of rank-and-file and supervisory election.
employees and (3) failure to submit a certificate of non-forum
shopping.6 Requirement of Certificate
Of Non-Forum Shopping
Petitioner moved for reconsideration on November 29, 2001. The Is Not Required in a Petition
Regional Director of DOLE Regional Office IV forwarded the For Certification Election
case to the Secretary of Labor. Meanwhile, on December 14,
2002, respondent filed a petition for cancellation of petitioner's In ruling against petitioner, the CA declared that under
union registration in the DOLE Regional Office IV.7 Administrative Circular No. 04-94,16 a certificate of non-forum
shopping was required in a petition for certification election. The
On January 17, 2003, Acting Secretary Manuel G. Imson, treating circular states:
the motion for reconsideration as an appeal, rendered a decision
reversing the order of the med-arbiter. He ruled that the legal The complaint and other initiatory pleadings referred to and
personality of a union cannot be collaterally attacked but may subject of this Circular are the original civil complaint,
only be questioned in an independent petition for cancellation of counterclaim, cross-claim, third (fourth, etc.) party complaint, or
registration. Thus, he directed the holding of a certification complaint-in-intervention, petition, or application wherein a
party asserts his claim for relief. (Emphasis certification against forum shopping, the pendency of another
supplied)cralawlibrary disciplinary action against the same respondent may still be
ascertained with ease.21 (Emphasis supplied)cralawlibrary
According to the CA, a petition for certification election asserts a
claim, i.e., the conduct of a certification election. As a result, it is The same situation holds true for a petition for certification
covered by the circular.17 election. Under the omnibus rules implementing the Labor Code
as amended by D.O. No. 9,22 it is supposed to be filed in the
We disagree. Regional Office which has jurisdiction over the principal office of
the employer or where the bargaining unit is principally
situated.23 The rules further provide that where two or more
The requirement for a certificate of non-forum shopping refers to
petitions involving the same bargaining unit are filed in one
complaints, counter-claims, cross-claims, petitions or applications
Regional Office, the same shall be automatically
where contending parties litigate their respective positions
consolidated.24 Hence, the filing of multiple suits and the
regarding the claim for relief of the complainant, claimant,
possibility of conflicting decisions will rarely happen in this
petitioner or applicant. A certification proceeding, even though
proceeding and, if it does, will be easy to discover.
initiated by a "petition," is not a litigation but an investigation of a
non-adversarial and fact-finding character.18
Notably, under the Labor Code and the rules pertaining to the
form of the petition for certification election, there is no
Such proceedings are not predicated upon an allegation of
requirement for a certificate of non-forum shopping either in D.O.
misconduct requiring relief, but, rather, are merely of an
No. 9, series of 1997 or in D.O. No. 40-03, series of 2003 which
inquisitorial nature. The Board's functions are not judicial in
replaced the former.25
nature, but are merely of an investigative character. The object of
the proceedings is not the decision of any alleged commission of
wrongs nor asserted deprivation of rights but is merely the Considering the nature of a petition for certification election and
determination of proper bargaining units and the ascertainment of the rules governing it, we therefore hold that the requirement for a
the will and choice of the employees in respect of the selection of certificate of non-forum shopping is inapplicable to such a
a bargaining representative. The determination of the proceedings petition.
does not entail the entry of remedial orders to redress rights, but
culminates solely in an official designation of bargaining units Treatment of Motion for Reconsideration as an Appeal
and an affirmation of the employees' expressed choice of
bargaining agent.19 (Emphasis supplied)cralawlibrary The CA ruled that petitioner's motion for reconsideration, which
was treated as an appeal by the Secretary of Labor, should not
In Pena v. Aparicio,20 we ruled against the necessity of attaching a have been given due course for lack of proof of service in
certification against forum shopping to a disbarment complaint. accordance with the implementing rules as amended by D.O. No.
We looked into the rationale of the requirement and concluded 9:
that the evil sought to be avoided is not present in disbarment
proceedings. Section 12. Appeal; finality of decision. - The decision of the
Med-Arbiter may be appealed to the Secretary for any violation of
'[The] rationale for the requirement of a certification against these Rules. Interloculory orders issued by the Med-Arbiter prior
forum shopping is to apprise the Court of the pendency of another to the grant or denial of the petition, including order granting
action or claim involving the same issues in another court, motions for intervention issued after an order calling for a
tribunal or quasi-judicial agency, and thereby precisely avoid the certification election, shall not be appealable. However, any issue
forum shopping situation. Filing multiple petitions or complaints arising therefrom may be raised in the appeal on the decision
constitutes abuse of court processes, which tends to degrade the granting or denying the petition.
administration of justice, wreaks havoc upon orderly judicial
procedure, and adds to the congestion of the heavily burdened The appeal shall be under oath and shall consist of a
dockets of the courts. Furthermore, the rule proscribing forum memorandum of appeal specifically stating the grounds relied
shopping seeks to promote candor and transparency among upon by the appellant with the supporting arguments and
lawyers and their clients in the pursuit of their cases before the evidence. The appeal shall be deemed not filed unless
courts to promote the orderly administration of justice, prevent accompanied by proof of service thereof to
undue inconvenience upon the other party, and save the precious appellee.26 (Emphasis supplied)cralawlibrary
time of the courts. It also aims to prevent the embarrassing
situation of two or more courts or agencies rendering conflicting
In accepting the appeal, the Secretary of Labor stated:
resolutions or decisions upon the same issue.

[Petitioner's] motion for reconsideration of the Med-Arbiter's


It is in this light that we take a further look at the necessity of
Order dated November 12, 2002 was verified under oath by
attaching a certification against forum shopping to a disbarment
[petitioner's] president Gil Dispabiladeras before Notary Public
complaint. It would seem that the scenario sought to be
Wilfredo A. Ruiz on 29 November 2002, and recorded in the
avoided, i.e., the filing of multiple suits and the possibility of
Notarial Register under Document No. 186, Page No. 38, Book
conflicting decisions, rarely happens in disbarment
V, series of 2002. On page 7 of the said motion also appears the
complaints considering that said proceedings are either "taken by
notation "copy of respondent to be delivered personally with the
the Supreme Court motu proprio, or by the Integrated Bar of the
name and signature of one Rosita Simon, 11/29/02." The
Philippines (IBP) upon the verified complaint of any person."
motion contained the grounds and arguments relied upon by
Thus, if the complainant in a disbarment case fails to attach a
[petitioner] for the reversal of the assailed Order. Hence, the
motion for reconsideration has complied with the formal collateral attack, but may be questioned only in an independent
requisites of an appeal. petition for cancellation in accordance with these Rules.30

The signature of Rosita Simon appearing on the last page of the -0-
motion can be considered as compliance with the required
proof of service upon respondent. Rosita Simon's employment Section 3. Acquisition of legal personality by local chapter. - A
status was a matter that should have been raised earlier by local/chapter constituted in accordance with Section 1 of this Rule
[respondent]. But [respondent] did not question the same and shall acquire legal personality from the date of filing of the
slept on its right to oppose or comment on [petitioner's] motion complete documents enumerated therein. Upon compliance with
for reconsideration. It cannot claim that it was unaware of the all the documentary requirements, the Regional Office or Bureau
filing of the appeal by [petitioner], because a copy of the of Labor Relations shall issue in favor of the local/chapter a
indorsement of the entire records of the petition to the Office of certificate indicating that it is included in the roster of legitimate
the Secretary "in view of the memorandum of appeal filed by Mr. labor organizations.31
Jesus B. Villamor" was served upon the employer and legal
counsels Atty. Ismael De Guzman and Atty. Anatolio Sabillo at
Such legal personality cannot thereafter be subject to collateral
the Samma Corporation Office, Main Avenue, PEZA, Rosario,
attack, but may be questioned only in an independent petition for
Cavite on December 5, 2002.27 (Emphasis supplied)cralawlibrary
cancellation of certificate of registration.32 Unless petitioner's
union registration is cancelled in independent proceedings, it shall
The motion for reconsideration was properly treated as an appeal continue to have all the rights of a legitimate labor organization,
because it substantially complied with the formal requisites of the including the right to petition for certification election.
latter. The lack of proof of service was not fatal as respondent had
actually received a copy of the motion. Consequently, it had the
Furthermore, the grounds for dismissal of a petition for
opportunity to oppose the same. Under these circumstances, we
certification election based on the lack of legal personality of a
find that the demands of substantial justice and due process were
labor organization are the following: (a) petitioner is not listed by
satisfied.
the Regional Office or the Bureau of Labor Relations in its
registry of legitimate labor organizations or (b) its legal
We stress that rules of procedure are interpreted liberally to personality has been revoked or cancelled with finality in
secure a just, speedy and inexpensive disposition of every action. accordance with the rules.33
They should not be applied if their application serves no useful
purpose or hinders the just and speedy disposition of cases.
As mentioned, respondent filed a petition for cancellation of the
Specifically, technical rules and objections should not hamper the
registration of petitioner on December 14, 2002. In a resolution
holding of a certification election wherein employees are to select
dated April 14, 2003, petitioner's charter certificate was revoked
their bargaining representative. A contrary rule will defeat the
by the DOLE. But on May 6, 2003, petitioner moved for the
declared policy of the Stateςηαñrοblεš νιr†υαl lαω lιbrαrÿ
reconsideration of this resolution. Neither of the parties alleged
that this resolution revoking petitioner's charter certificate had
to promote the free and responsible exercise of the right to self- attained finality. However, in this petition, petitioner prayed that
organization through the establishment of a simplified its charter certificate be "reinstated in the roster of active
mechanism for the speedy registration of labor organizations and legitimate labor [organizations]."34 This cannot be granted here.
workers' associations, determination of representation status, To repeat, the proceedings on a petition for cancellation of
and resolution of intra and inter-union disputes.28 xxx (Emphasis registration are independent of those of a petition for certification
supplied)cralawlibrary election. This case originated from the latter. If it is shown that
petitioner's legal personality had already been revoked or
Legal Personality of Petitioner cancelled with finality in accordance with the rules, then it is no
longer a legitimate labor organization with the right to petition for
Petitioner argues that the erroneous inclusion of one supervisory a certification election.
employee in the union of rank-and-file employees was not a
ground to impugn its legitimacy as a legitimate labor organization A Final Note
which had the right to file a petition for certification election.
Respondent, as employer, had been the one opposing the holding
We agree. of a certification election among its rank-and-file employees. This
should not be the case. We have already declared that, in
LIKHA was granted legal personality as a federation under certification elections, the employer is a bystander; it has no right
certificate of registration no. 92-1015-032-11638-FED-LC. or material interest to assail the certification election.35
Subsequently, petitioner as its local chapter was issued its charter
certificate no. 2-01.29 With certificates of registration issued in [This] Court notes that it is petitioner, the employer, which has
their favor, they are clothed with legal personality as legitimate offered the most tenacious resistance to the holding of a
labor organizations: certification election among its monthly-paid rank-and-file
employees. This must not be so, for the choice of a collective
Section 5. Effect of registration. - The labor organization or bargaining agent is the sole concern of the employees. The only
workers' association shall be deemed registered and vested with exception to this rule is where the employer has to file the petition
legal personality on the date of issuance of its certificate of for certification election pursuant to Article 258 of the Labor
registration. Such legal personality cannot thereafter be subject to Code because it was requested to bargain collectively, which
exception finds no application in the case before us. Its role in a
certification election has aptly been described in Trade Unions of only through a separate and independent action for cancellation of
the Philippines and Allied Services (TUPAS) v. Trajano, as that of union registration. Finally, as to the alleged acts of
a mere bystander. It has no legal standing in a certification misrepresentation, KML asserted that LEGEND failed to
election as it cannot oppose the petition or appeal the Med- substantiate its claim.
Arbiter's orders related thereto. . .36
Ruling of the Med-Arbiter
WHEREFORE, the petition is hereby GRANTED. Let the
records of the case be remanded to the office of origin, the On September 20, 2001, the Med-Arbiter[4] rendered
Regional Office IV of the Department of Labor and Employment, judgment[5] dismissing for lack of merit the petition for
for determination of the status of petitioner's legal personality. If certification election.  The Med-Arbiter found that indeed there
petitioner is still a legitimate labor organization, then said office were several supervisory employees in KML's membership. 
shall conduct a certification election subject to the usual pre- Since Article 245 of the Labor Code expressly prohibits
election conference. supervisory employees from joining the union of rank and file
employees, the Med-Arbiter concluded that KML is not a
legitimate labor organization.  KML was also found to have
SO ORDERED.
fraudulently procured its registration certificate by
misrepresenting that 70 employees were among those who
attended its organizational meeting on April 5, 2001 when in fact
they were either at work or elsewhere.
[G.R. No. 169754, February 23 : 2011]
KML thus appealed to the Office of the Secretary of the DOLE.
LEGEND INTERNATIONAL RESORTS LIMITED,
PETITIONER, VS. KILUSANG MANGGAGAWA NG Ruling of the Office of the Secretary of DOLE
LEGENDA (KML- INDEPENDENT), RESPONDENT.
On May 22, 2002, the Office of the Secretary of DOLE rendered
DECISION its Decision[6] granting KML's appeal thereby reversing and
setting aside the Med-Arbiter's Decision.  The Office of the
DEL CASTILLO, J.: Secretary of DOLE held that KML's legitimacy as a union could
not be collaterally attacked, citing Section 5,[7] Rule V of
This Petition for Review on Certiorari assails the September 18, Department Order No. 9, series of 1997.
2003 Decision of the Court of Appeals in CA-G.R. SP No. 72848
which found no grave abuse of discretion on the part of the Office The Office of the Secretary of DOLE also opined that Article 245
of the Secretary of the Department of Labor and Employment of the Labor Code merely provides for the prohibition on
(DOLE) which ruled in favor of Kilusang Manggagawa ng managerial employees to form or join a union and the ineligibility
Legenda (KML).  Also assailed is the September 14, 2005 of supervisors to join the union of the rank and file employees
Resolution denying petitioner's motion for reconsideration. and vice versa. It declared that any violation of the provision of
Article 245 does not ipso facto render the existence of the labor
Factual Antecedents organization illegal. Moreover, it held that Section 11, paragraph
II of Rule XI which provides for the grounds for dismissal of a
On June 6, 2001, KML filed with the Med-Arbitration Unit of the petition for certification election does not include mixed
DOLE, San Fernando, Pampanga, a Petition for Certification membership in one union.
Election[1] docketed as Case No. RO300-0106-RU-001. KML
alleged that it is a legitimate labor organization of the rank and The dispositive portion of the Office of the Secretary of DOLE's
file employees of Legend International Resorts Limited Decision reads:
(LEGEND).  KML claimed that it was issued its Certificate of
Registration No. RO300-0105-UR-002 by the DOLE on May 18, WHEREFORE, the appeal is hereby GRANTED and the order of
2001. the Med-Arbiter dated 20 September 2001 is REVERSED and
SET ASIDE.
LEGEND moved to dismiss[2] the petition alleging that KML is
not a legitimate labor organization because its membership is a Accordingly, let the entire record of the case be remanded to the
mixture of rank and file and supervisory employees in violation of regional office of origin for the immediate conduct of the
Article 245 of the Labor Code. LEGEND also claimed that KML certification election, subject to the usual pre-election conference,
committed acts of fraud and misrepresentation when it made it among the rank and file employees of LEGEND
appear that certain employees attended its general membership INTERNATIONAL RESORTS LIMITED with the following
meeting on April 5, 2001 when in reality some of them were choices:
either at work; have already resigned as of March 2001; or were
abroad. 1. KILUSANG MANGGAGAWA NG LEGENDA (KML-
INDEPENDENT); and
In its Comment,[3] KML argued that even if 41 of its members are
indeed supervisory employees and therefore excluded from its 2. NO UNION.
membership, the certification election could still proceed because
the required number of the total rank and file employees Pursuant to Rule XI, Section II.1 of D.O. No. 9, the employer is
necessary for certification purposes is still sustained. KML also hereby directed to submit to the office of origin, within ten days
claimed that its legitimacy as a labor union could not be from receipt of the decision, the certified list of employees in the
collaterally attacked in the certification election proceedings but bargaining unit for the last three (3) months prior to the issuance
of this decision. and AFFIRMED.  The instant petition is DENIED due course
and, accordingly, DISMISSED for lack of merit.[18]
SO DECIDED.[8]
LEGEND filed a Motion for Reconsideration[19] alleging, among
[9]
LEGEND filed its Motion for Reconsideration  reiterating its others, that it has appealed to the Court of Appeals the March 26,
earlier arguments.  It also alleged that on August 24, 2001, it filed 2002 Decision in Case No. RO300-0108-CP-001 denying its
a Petition[10] for Cancellation of Union Registration of KML petition for cancellation and that it is still pending resolution.
docketed as Case No. RO300-0108-CP-001 which was
granted[11] by the DOLE Regional Office No. III of San Fernando, On September 14, 2005, the appellate court denied LEGEND's
Pampanga in its Decision[12] dated November 7, 2001. motion for reconsideration.

In a Resolution[13] dated August 20, 2002, the Office of the Hence, this Petition for Review on Certiorari raising the lone
Secretary of DOLE denied LEGEND's motion for assignment of error, viz:
reconsideration. It opined that Section 11, paragraph II(a), Rule
XI of Department Order No. 9 requires a final order of WHETHER X X X THE HONORABLE COURT OF APPEALS
cancellation before a petition for certification election may be COMMITTED SERIOUS ERRORS IN THE APPLICATION OF
dismissed on the ground of lack of legal personality.  Besides, it LAW IN DENYING THE PETITIONER'S PETITION FOR
noted that the November 7, 2001 Decision of DOLE Regional CERTIORARI.[20]
Office No. III of San Fernando, Pampanga in Case No. RO300-
0108-CP-001 was reversed by the Bureau of Labor Relations in a Petitioner's Arguments
Decision dated March 26, 2002.
 LEGEND submits that the Court of Appeals grievously erred in
Ruling of the Court of Appeals ruling that the March 26, 2002 Decision denying its Petition for
Cancellation of KML's registration has already become final and
Undeterred, LEGEND filed a Petition for Certiorari[14] with the executory.  It asserts that it has seasonably filed a Petition
Court of Appeals docketed as CA-G.R. SP No. 72848.  LEGEND for Certiorari[21] before the CA docketed as CA-G.R. SP No.
alleged that the Office of the Secretary of DOLE gravely abused 72659 assailing said Decision.  In fact, on June 30, 2005, the
its discretion in reversing and setting aside the Decision of the Court of Appeals granted the petition, reversed the March 26,
Med-Arbiter despite substantial and overwhelming evidence 2002 Decision of the Bureau of Labor Relations and reinstated the
against KML. November 7, 2001 Decision of the DOLE Regional Office III
ordering the cancellation of KML's registration.
For its part, KML alleged that the Decision dated March 26, 2002
of the Bureau of Labor Relations in Case No. RO300-0108-CP- Finally, LEGEND posits that the cancellation of KML's
001 denying LEGEND's petition for cancellation and upholding certificate of registration should retroact to the time of its
KML's legitimacy as a labor organization has already become issuance.[22]  It thus claims that the petition for certification
final and executory, entry of judgment having been made on election and all of KML's activities should be nullified because it
August 21, 2002.[15] has no legal personality to file the same, much less demand
collective bargaining with LEGEND.[23]
The Office of the Secretary of DOLE also filed its
Comment[16] asserting that KML's legitimacy cannot be attacked LEGEND thus prays that the September 20, 2001 Decision of the
collaterally.  Finally, the Office of the Secretary of DOLE Med-Arbiter dismissing KML's petition for certification election
stressed that LEGEND has no legal personality to participate in be reinstated.[24]
the certification election proceedings.
Respondent's Arguments
On September 18, 2003, the Court of Appeals rendered its
Decision[17] finding no grave abuse of discretion on the part of the In its Comment filed before this Court dated March 21, 2006,
Office of the Secretary of DOLE.  The appellate court held that KML insists that the Decision of the Bureau of Labor Relations
the issue on the legitimacy of KML as a labor organization has upholding its legitimacy as a labor organization has already
already been settled with finality in Case No. RO300-0108-CP- attained finality[25] hence there was no more hindrance to the
001.  The March 26, 2002 Decision of the Bureau of Labor holding of a certification election.  Moreover, it claims that the
Relations upholding the legitimacy of KML as a labor instant petition has become moot because the certification
organization had long become final and executory for failure of election sought to be prevented had already been conducted.
LEGEND to appeal the same.  Thus, having already been settled
that KML is a legitimate labor organization, the latter could Our Ruling
properly file a petition for certification election.  There was
nothing left for the Office of the Secretary of DOLE to do but to The petition is partly meritorious.
order the holding of such certification election.
LEGEND has timely appealed the
The dispositive portion of the Decision reads: March 26, 2002 Decision of the Bureau
of Labor Relations to the Court of
WHEREFORE, in view of the foregoing, and finding that no Appeals.
grave abuse of discretion amounting to lack or excess of
jurisdiction has been committed by the Department of Labor and We cannot understand why the Court of Appeals totally
Employment, the assailed May 22, 2002 Decision and August 20, disregarded LEGEND's allegation in its Motion for
2002 Resolution in Case No. RO300-106-RU-001 are UPHELD Reconsideration that the March 26, 2002 Decision of the Bureau
of Labor Relations has not yet attained finality considering that it x x x It is well-settled rule that `a certification proceedings is not
has timely appealed the same to the Court of Appeals and which a litigation in the sense that the term is ordinarily understood, but
at that time is still pending resolution.   The Court of Appeals an investigation of a non-adversarial and fact finding character.'
never bothered to look into this allegation and instead dismissed (Associated Labor Unions (ALU) v. Ferrer-Calleja, 179 SCRA
outright LEGEND's motion for reconsideration.  By doing so, the 127 [1989]; Philippine Telegraph and Telephone Corporation v.
Court of Appeals in effect maintained its earlier ruling that the NLRC, 183 SCRA 451 [1990].  Thus, the technical rules of evidence
March 26, 2002 Decision of the Bureau of Labor Relations do not apply if the decision to grant it proceeds from an
upholding the legitimacy of KML as a labor organization has long examination of the sufficiency of the petition as well as a careful
become final and executory for failure of LEGEND to appeal the look into the arguments contained in the position papers and other
same. documents.

This is inaccurate.  Records show that (in the cancellation of At any rate, the Court applies the established rule correctly
registration case) LEGEND has timely filed on September 6, followed by the public respondent that an order to hold a
2002 a petition for certiorari[26] before the Court of Appeals certification election is proper despite the pendency of the
which was docketed as CA-G.R. SP No. 72659 assailing the petition for cancellation of the registration certificate of the
March 26, 2002 Decision of the Bureau of Labor Relations.   In respondent union.  The rationale for this is that at the time
fact, KML received a copy of said petition on September 10, the respondent union filed its petition, it still had the legal
2002[27] and has filed its Comment thereto on December 2, 2002. personality to perform such act absent an order directing the
[28]
 Thus, we find it quite interesting for KML to claim in its cancellation.[39] (Emphasis supplied.)
Comment (in the certification petition case) before this Court
dated March 21, 2006[29] that the Bureau of Labor Relations' In Capitol Medical Center, Inc. v. Hon. Trajano,[40] we also held
Decision in the petition for cancellation case has already attained that "the pendency of a petition for cancellation of union
finality.   Even in its Memorandum[30] dated March 13, 2007 filed registration does not preclude collective bargaining."[41]  Citing
before us, KML is still insisting that the Bureau of Labor the Secretary of Labor, we held viz:
Relations' Decision has become final and executory.
That there is a pending cancellation proceedings against the
Our perusal of the records shows that on June 30, 2005, the Court respondent Union is not a bar to set in motion the mechanics
of Appeals rendered its Decision[31] in CA-G.R. SP No. 72659 of collective bargaining.  If a certification election may still be
reversing the March 26, 2002 Decision of the Bureau of Labor ordered despite the pendency of a petition to cancel the
Relations and reinstating the November 7, 2001 Decision of the union's registration certificate x x x more so should the
Med-Arbiter which canceled the certificate of registration of collective bargaining process continue despite its pendency. [42]
KML.[32]  On September 30, 2005, KML's motion for (Emphasis supplied.)
reconsideration was denied for lack of merit.[33]  On November
25, 2005, KML filed its Petition for Review In Association of Court of Appeals Employees v. Ferrer-Calleja,
on Certiorari[34] before this Court which was docketed as G.R. [43]
 this Court was tasked to resolve the issue of whether "the
No. 169972.  However, the same was denied in a certification proceedings should be suspended pending [the
Resolution[35] dated February 13, 2006 for having been filed out of petitioner's] petition for the cancellation of union registration of
time.  KML moved for reconsideration but it was denied with the UCECA[44]."[45]  The Court resolved the issue in the negative
finality in a Resolution[36] dated June 7, 2006.  Thereafter, the said holding that "an order to hold a certification election is proper
Decision canceling the certificate of registration of KML as a despite the pendency of the petition for cancellation of the
labor organization became final and executory and entry of registration certificate of the respondent union.  The rationale
judgment was made on July 18, 2006.[37] for this is that at the time the respondent union filed its petition, it
still had the legal personality to perform such act absent an order
The cancellation of KML's certificate of directing a cancellation."[46]  We reiterated this view in Samahan
registration should not retroact to the ng Manggagawa sa Pacific Plastic v. Hon. Laguesma[47] where
time of its issuance. we declared that "a certification election can be conducted
despite pendency of a petition to cancel the union registration
Notwithstanding the finality of the Decision canceling the certificate.  For the fact is that at the time the respondent union
certificate of registration of KML, we cannot subscribe to filed its petition for certification, it still had the legal personality
LEGEND's proposition that the cancellation of KML's certificate to perform such act absent an order directing its cancellation."[48]
of registration should retroact to the time of its issuance. 
LEGEND claims that KML's petition for certification election Based on the foregoing jurisprudence, it is clear that a
filed during the pendency of the petition for cancellation and its certification election may be conducted during the pendency of
demand to enter into collective bargaining agreement with the cancellation proceedings.  This is because at the time the
LEGEND should be dismissed due to KML's lack of legal petition for certification was filed, the petitioning union is
personality. presumed to possess the legal personality to file the same.  There
is therefore no basis for LEGEND's assertion that the cancellation
This issue is not new or novel.  In Pepsi-Cola Products of KML's certificate of registration should retroact to the time of
Philippines, Inc. v. Secretary of Labor,[38] we already ruled that: its issuance or that it effectively nullified all of KML's activities,
including its filing of the petition for certification election and its
Anent the issue of whether or not the Petition to cancel/revoke demand to collectively bargain.
registration is a prejudicial question to the petition for
certification election, the following ruling in the case The legitimacy of the legal personality
of Association of the Court of Appeals Employees (ACAE) v. of KML cannot be collaterally attacked
Hon. Pura Ferrer-Calleja, x x x is in point, to wit: in a petition for certification election.
dated September 18, 2003 in CA-G.R. SP No. 72848 insofar as it
We agree with the ruling of the Office of the Secretary of DOLE affirms the May 22, 2002 Decision and August 20, 2002
that the legitimacy of the legal personality of KML cannot be Resolution of the Office of the Secretary of Department of Labor
collaterally attacked in a petition for certification election and Employment is AFFIRMED.  The Decision of the Court of
proceeding.  This is in consonance with our ruling in Laguna Appeals insofar as it declares that the March 26, 2002 Decision of
Autoparts Manufacturing Corporation v. Office of the Secretary, the Bureau of Labor Relations in Case No. RO300-0108-CP-001
Department of Labor and Employment[49] that "such legal upholding that the legitimacy of KML as a labor organization has
personality may not be subject to a collateral attack but only long become final and executory for failure of LEGEND to
through a separate action instituted particularly for the purpose of appeal the same, is REVERSED and SET ASIDE.
assailing it."[50]  We further held therein that:
SO ORDERED.
This is categorically prescribed by Section 5, Rule V of the
Implementing Rules of Book V, which states as follows:

SEC. 5.[51]  Effect of registration. - The labor organization or


worker's association shall be deemed registered and vested with
legal personality on the date of issuance of its certificate of G.R. No. 87332 August 13, 1990
registration.  Such legal personality cannot thereafter be subject
to collateral attack but may be questioned only in an independent
EDUARDO CRUZ, MOISES MACASO, ROY GALLINERA,
petition for cancellation in accordance with these Rules.
PERCIVAL CRUZ, BUBBY VALERIO, ROY PAMITTAN,
MANOLO ORBETA & MARIANO VILLA, petitioners,
Hence, to raise the issue of the respondent union's legal
vs.
personality is not proper in this case.  The pronouncement of the
THE HON. PURA FERRER-CALLEJA, in her capacity as
Labor Relations Division Chief, that the respondent union
Director of the Bureau of Labor Relations, & ROLANDO G.
acquired a legal personality x x x cannot be challenged in a
OCAMPO, ET AL., * respondents.
petition for certification election.

The discussion of the Secretary of Labor and Employment on this Douglas G. Baarde for petitioners.
point is also enlightening, thus:
Banzuela, Flores, Miralles, Raneses, Taquio, Sy and Associates
. . . Section 5, Rule V of D.O. 9 is instructive on the matter.  It for private respondents.
provides that the legal personality of a union cannot be the subject
of collateral attack in a petition for certification election, but may
be questioned only in an independent petition for cancellation of
union registration.  This has been the rule since NUBE v. Minister
GRIÑO-AQUINO, J.:
of Labor, 110 SCRA 274 (1981).  What applies in this case is the
principle that once a union acquires a legitimate status as a labor
organization, it continues as such until its certificate of The only issue raised by the petition for certiorari in this case is
registration is cancelled or revoked in an independent action for whether the respondent, Pura Ferrer-Calleja, Director of the
cancellation. Bureau of Labor Relations, gravely abused her discretion in
nullifying the resolution of the Board of Administrators of the
Equally important is Section 11, Paragraph II, Rule IX of D.O. 9, Allied Bank Employees Union which extended for one year the
which provides for the dismissal of a petition for certification term of office of the union officers which should have expired on
election based on the lack of legal personality of a labor February 10, 1987 and postponed to February 10, 1988 the
organization only in the following instances: (1) appellant is not holding of the election.
listed by the Regional Office or the BLR in its registry of
legitimate labor organizations; or (2) appellant's legal personality In 1984, the Allied Bank Employees Union (ABEU), which was
has been revoked or cancelled with finality.  Since appellant is then a mere chapter of the National Union of Bank Employees
listed in the registry of legitimate labor organizations, and its (NUBE), elected its officers, whose term of office would expire
legitimacy has not been revoked or cancelled with finality, the on February 10, 1987.
granting of its petition for certification election is proper.[52]
Before the expiration of the old CBA between the ABEU and
"[T]he legal personality of a legitimate labor organization x x x Allied Bank on June 30, 1984, the ABEU negotiated for a new
cannot be subject to a collateral attack.  The law is very clear on CBA. However, because the Union and the Bank could not agree
this matter. x x x The Implementing Rules stipulate that a labor on major economic proposals, a bargaining deadlock ensued.
organization shall be deemed registered and vested with legal
personality on the date of issuance of its certificate of
registration.  Once a certificate of registration is issued to a union, ABEU filed a Notice of Strike on November 26, 1984. Labor
its legal personality cannot be subject to a collateral attack.  In Minister Blas Ople assumed jurisdiction over the labor dispute on
may be questioned only in an independent petition for December 19, 1984.
cancellation in accordance with Section 5 of Rule V, Book V of
the Implementing Rules."[53] In defiance of the Minister's return-to-work order, the Union
declared a strike on January 3, 1985 and established picketlines at
WHEREFORE, in view of the foregoing, the petition the Bank's Head Office and Binondo Branch.
is PARTLY GRANTED.  The Decision of the Court of Appeals
On January 31, 1985, Minister Ople issued an Order resolving the After the hearing of the petition and the submission of the parties'
deadlock issues in the collective bargaining and in effect drew up position papers, the Med-Arbiter, on January 4, 1988, issued an
a new CBA for the parties. The 3-year term of the new CBA order directing the Union to call a general membership meeting
would expire on January 31, 1988. The union asked for where the manner of conducting the election could be discussed
reconsideration, and on February 11, 1985, continued its strike. before fixing the date of the election. The order declared null and
Two hundred seventy (270) striking officers and employees of void the one-year extension of the CBA from February 1, 1988 to
ABEU (among them were the private respondents) were January 31, 1989.
dismissed by the Bank for abandonment of work and commission
of illegal acts. Petitioners filed a motion for reconsideration (which was treated
as an appeal) of the order of the Med-Arbiter. In a resolution
On March 4, 1985, the Bank filed in the Arbitration Branch, dated January 6, 1988, the Interim Board appointed a Comelec
NCR, NLRC, a petition to declare the strike illegal. which, on January 8, 1988, issued a resolution setting the election
of officers on February 10, 1988.
Upon receipt of the Resolution dated March 7, 1985 modifying
Minister Ople's order dated January 31, 1985, the ABEU on On motion of Ocampo, et al, the Bureau of Labor Relations
March 8, 1985 lifted its picketlines and announced its intention to issued on February 9, 1988, a temporary restraining order
return to work. However, the Bank refused to admit the strikers. enjoining petitioners, including the Union's Comelec, from
proceeding with the election on February 10, 1988. However,
On March 19, 1985, a referendum was conducted by the ABEU to despite the restraining order, the Union held the election on
ratify the 1985-1988 CBA incorporating the additional benefits February 10, 1988 as scheduled. Petitioners were declared the
awarded in the March 7, 1985 resolution. A majority voted for winners in the election.
ratification.
On March 2, 1989, the public respondent, Director Pura Calleja of
On July 15, 1985, the Bank filed a motion praying for the the BLR, issued a resolution whose dispositive portion reads as
issuance of an order directing the Union to hold a general follows:
membership meeting for the purpose of designating union
representatives who would sign the CBA inasmuch as the Union's WHEREFORE, premises considered, the
officers had already been dismissed by the Bank. election conducted in Allied Bank Employees
Union on February 10, 1988 is hereby declared
On November 11, 1985, the NUBE issued a special resolution null and void.
creating an ABEU Interim Board tasked to sign the new CBA
with the Bank in lieu of the union officers who had been Another election is hereby ordered conducted
dismissed by the Bank. ** in accordance with the express tenor of the
Med-Arbiter's Order dated 4 January 1988,
In January, 1987, the Interim Board commenced negotiations with that a general membership meeting shall first
the Bank for a one-year extension of the CBA which was expiring be held where the mechanics of the election
on January 31, 1988. A drive for the extension of the CBA began shall be fully threshed out.
in March 1987 for the referendum would take place on June 23,
1987. However, the Interim Board also submitted to the In the meantime, the officers who were elected
referendum the matter of extending for one year the term of office on 10 February 1988 and whose election to
of the Interim Board, in effect, postponing for one year the office we now declare null and void are hereby
election of the regular officers of the Union. The overwhelming temporarily charged with the safekeeping of
majority of the union members voted yes in the referendum. The the union funds subject to accounting before
postponement of the election did not sit well with the ousted the new set of officers which shall be elected
officers led by private respondent Rolando Ocampo. On June 23, in accordance with this Order.
1987, they filed in the Office of Labor Secretary Franklin Drilon a
letter-petition against the postponement of the election of the The respondent Bank is likewise enjoined to
officers of the Union and to nullify the one year extension of the observe absolute neutrality during these
CBA. activities, they being purely an internal affair
of the union. (p. 7, Rollo.)
After the referendum result was announced on July 3, 1987, the
Bank granted a P600 "signing bonus" to all the employees. The issues raised in this petition for certiorari are as follows:
Private respondents and other employees stationed in the Strata
Building in Pasig, Metro Manila, collected the signing bonus but,
a) Whether or not the public respondent erred in declaring null
in protest, they deposited it in the Equitable Banking Corporation,
and void the election held on February 10, 1988; and
payable to Allied Bank through Secretary Drilon.

b) Whether or not the extension of the CBA was valid.


The Office of the Secretary of Labor forwarded the letter-petition
of respondent Rolando Ocampo to the National Capital Region,
DOLE, Manila, which subsequently treated it as a formal petition, There is no merit in the petitioners' contention that the public
docketed as Case No. NCR-OP-M-8-611-87. respondent gravely abused her discretion in annulling the
February 10, 1988 election of officers. The public respondent
correctly noted that in ordering the postponement of the election
for one year (in effect extending their term of office for one year), G.R. No. 85333 February 26, 1990
the ABEU Interim Board "overstepped its bounds" for it was
constituted and authorized only "to sign for and in behalf of the CARMELITO L. PALACOL, ET AL., petitioners,
union the Collective Bargaining Agreement with the Bank and vs.
administer the CBA and the operation of the union." (p. PURA FERRER-CALLEJA, Director of the Bureau of Labor
27, Rollo.) Relations, MANILA CCBPI SALES FORCE UNION, and
COCA-COLA BOTTLERS (PHILIPPINES),
Instead of calling a regular election of officers INC., respondents.
of ABEU on 11 February 1987, as mandated
by the Union's Constitution and by-laws, Wellington B. Lachica for petitioners.
respondents submitted in a "referendum" the
extension of their term of office for yet another
Adolpho M. Guerzon for respondent Union.
year, from 11 February 1987 to 10 February
1988.

From the very inception the referendum


process initiated by the Interim Board was GANCAYCO, J.:
improper. The results therefrom are therefore,
invalid. It may be true, that the task of Can a special assessment be validly deducted by a labor union
administering the operation of the union was from the lump-sum pay of its members, granted under a collective
given to the ABEU-Interim Board at the time it bargaining agreement (CBA), notwithstanding a subsequent
was constituted, to fill in the vacuum in the disauthorization of the same by a majority of the union members?
local union's leadership during that time. This is the main issue for resolution in the instant petition for
Nonetheless said task could not be exercised certiorari.
beyond the regular term of the regular officers.
Stated simply, the exercise of said task is only As gleaned from the records of the case, the pertinent facts are as
coterminous with the term of the regular follows:
officers, in whose shoes, the members of the
ABEU-Interim Board merely stepped into.
On October 12, 1987, the respondent Manila CCBPI Sales Force
Union (hereinafter referred to as the Union), as the collective
When the term of the union's regular officers bargaining agent of all regular salesmen, regular helpers, and
expired on February 11, 1987 the election of relief helpers of the Manila Plant and Metro Manila Sales Office
officers should have been held, in accordance of the respondent Coca-Cola Bottlers (Philippines), Inc.
with the provision of the union constitution (hereinafter referred to as the Company) concluded a new
and by laws. With the expiration of the term of collective bargaining agreement with the latter. 1 Among the
the regular officers, the term of the ABEU- compensation benefits granted to the employees was a general
Interim Board, expired too. In calling the salary increase to be given in lump sum including recomputation
referendum therefore, the ABEU-Interim of actual commissions earned based on the new rates of increase.
Board clearly overstepped its bounds. (pp. 27-
28, Rollo.)
On the same day, the president of the Union submitted to the
Company the ratification by the union members of the new CBA
The second issue regarding the validity of the one-year extension and authorization for the Company to deduct union dues
of the CBA, as observed by the BLR, has become moot and equivalent to P10.00 every payday or P20.00 every month and, in
academic. The public respondent's view that the one-year addition, 10% by way of special assessment, from the CBA lump-
extension was also null and void is not quite correct for the sum pay granted to the union members. The last one among the
extension was approved by the Union in a referendum which was aforementioned is the subject of the instant petition.
properly supervised by the Department of Labor. It was accepted
by the Bank which gave a "signing bonus" to the employees who
voted for it. Since the holding of the referendum was within the As embodied in the Board Resolution of the Union dated
authority of the Interim Board "to administer the CBA and September 29, 1987, the purpose of the special assessment sought
operate the union," and the extension was acceptable to both of to be levied is "to put up a cooperative and credit union; purchase
the parties to the agreement, and did not violate any law, it is vehicles and other items needed for the benefit of the officers and
valid and binding on them. the general membership; and for the payment for services
rendered by union officers, consultants and others." 2 There was
also an additional proviso stating that the "matter of allocation ...
WHEREFORE, the instant petition for certiorari is dismissed for shall be at the discretion of our incumbent Union President."
lack of merit. No costs.
This "Authorization and CBA Ratification" was obtained by the
SO ORDERED. Union through a secret referendum held in separate local
membership meetings on various dates. 3 The total membership of
the Union was about 800. Of this number, 672 members
originally authorized the 10% special assessment, while 173
opposed the same. 4
Subsequently however, one hundred seventy (170) members of may be checked off from
the Union submitted documents to the Company stating that any amount due to an
although they have ratified the new CBA, they are withdrawing or employee without an
disauthorizing the deduction of any amount from their CBA lump individual written
sum. Later, 185 other union members submitted similar authorization duly signed
documents expressing the same intent. These members, by the employee. The
numbering 355 in all (170 + 185), added to the original oppositors authorization should
of 173, turned the tide in favor of disauthorization for the special specifically state the
assessment, with a total of 528 objectors and a remainder of 272 amount, purpose and
supporters. 5 beneficiary of the
deduction;
On account of the above-mentioned disauthorization, the
Company, being in a quandary as to whom to remit the payment As authority for their contention, petitioners cited Galvadores v.
of the questioned amount, filed an action for interpleader with the Trajano, 6 wherein it was ruled that no check-offs from any
Bureau of Labor Relations in order to resolve the conflicting amount due employees may be effected without individual written
claims of the parties concerned. Petitioners, who are regular rank- authorizations duly signed by the employees specifically stating
and-file employees of the Company and bona fide members of the the amount, purpose, and beneficiary of the deduction.
Union, filed a motion/complaint for intervention therein in two
groups of 161 and 94, respectively. They claimed to be among In its answer, the Union countered that the deductions not only
those union members who either did not sign any individual have the popular indorsement and approval of the general
written authorization, or having signed one, subsequently membership, but likewise complied with the legal requirements of
withdrew or retracted their signatures therefrom. Article 241 (n) and (o) of the Labor Code in that the board
resolution of the Union imposing the questioned special
Petitioners assailed the 10% special assessment as a violation of assessment had been duly approved in a general membership
Article 241(o) in relation to Article 222(b) of the Labor Code. meeting and that the collection of a special fund for labor
Article 222(b) provides as follows: education and research is mandated.

ART. 222. Appearances and Fees. — Article 241(n) of the Labor Code states that —

xxx xxx xxx ART. 241. Rights and conditions of


membership in a labor organization. —
(b) No attorney's fees,
negotiation fees or similar xxx xxx xxx
charges of any kind arising
from any collective (n) No special assessment or other
bargaining negotiations or extraordinary fees may be levied upon the
conclusion of the collective members of a labor organization unless
agreement shall be imposed authorized by a written resolution of a majority
on any individual member of all the members at a general membership
of the contracting union; meeting duly called for the purpose. The
Provided, however, that secretary of the organization shall record the
attorney's fees may be minutes of the meeting including the list of all
charged against union funds members present, the votes cast, the purpose of
in an amount to be agreed the special assessment or fees and the recipient
upon by the parties. Any of such assessments or fees. The record shall
contract, agreement or be attested to by the president;
arrangement of any sort to
the contrary shall be null
Med-Arbiter Manases T. Cruz ruled in favor of petitioners in an
and void.
order dated February 15, 1988 whereby he directed the Company
to remit the amount it had kept in trust directly to the rank-and-
On the other hand, Article 241(o) mandates that: file personnel without delay.

ART. 241. Rights and conditions of On appeal to the Bureau of Labor Relations, however, the order
membership in a labor organization. — of the Med-Arbiter was reversed and set aside by the respondent-
Director in a resolution dated August 19, 1988 upholding the
xxx xxx xxx claim of the Union that the special assessment is authorized under
Article 241 (n) of the Labor Code, and that the Union has
(o) Other than for complied with the requirements therein.
mandatory activities under
the Code, no special Hence, the instant petition.
assessments, attorney's fees,
negotiation fees or any
other extraordinary fees
Petitioners allege that the respondent-Director committed a grave there must be a general membership meeting. The contention of
abuse of discretion amounting to lack or excess of jurisdiction the Union that "the local membership meetings are precisely the
when she held Article 241 (n) of the Labor Code to be the very general meetings required by law" 10 is untenable because the
applicable provision instead of Article 222(b) in relation to law would not have specified a general membership meeting had
Article 241(o) of the same law. the legislative intent been to allow local meetings in lieu of the
latter.
According to petitioners, a cursory examination and comparison
of the two provisions of Article 241 reveals that paragraph (n) It submitted only minutes of the local membership meetings when
cannot prevail over paragraph (o). The reason advanced is that a what is required is a written resolution adopted at the general
special assessment is not a matter of major policy affecting the meeting. Worse still, the minutes of three of those local meetings
entire union membership but is one which concerns the individual held were recorded by a union director and not by the union
rights of union members. secretary. The minutes submitted to the Company contained no
list of the members present and no record of the votes cast. Since
Petitioners further assert that assuming arguendo that Article it is quite evident that the Union did not comply with the law at
241(n) should prevail over paragraph (o), the Union has every turn, the only conclusion that may be made therefrom is
nevertheless failed to comply with the procedure to legitimize the that there was no valid levy of the special assessment pursuant to
questioned special assessment by: (1) presenting mere minutes of paragraph (n) of Article 241 of the Labor Code.
local membership meetings instead of a written resolution; (2)
failing to call a general membership meeting; (3) having the Paragraph (o) on the other hand requires an individual written
minutes of three (3) local membership meetings recorded by a authorization duly signed by every employee in order that a
union director, and not by the union secretary as required; (4) special assessment may be validly checked-off. Even assuming
failing to have the list of members present included in the minutes that the special assessment was validly levied pursuant to
of the meetings; and (5) failing to present a record of the votes paragraph (n), and granting that individual written authorizations
cast. 7 Petitioners concluded their argument by citing Galvadores. were obtained by the Union, nevertheless there can be no valid
check-off considering that the majority of the union members had
After a careful review of the records of this case, We are already withdrawn their individual authorizations. A withdrawal
convinced that the deduction of the 10% special assessment by of individual authorizations is equivalent to no authorization at
the Union was not made in accordance with the requirements all. Hence, the ruling in Galvadores that "no check-offs from any
provided by law. amounts due employees may be effected without an individual
written authorization signed by the employees ... " is applicable.
Petitioners are correct in citing the ruling of this Court
in Galvadores which is applicable to the instant case. The The Union points out, however, that said disauthorizations are not
principle "that employees are protected by law from unwarranted valid for being collective in form, as they are "mere bunches of
practices that diminish their compensation without their known randomly procured signatures, under loose sheets of
edge and consent" 8 is in accord with the constitutional principle paper." 11 The contention deserves no merit for the simple reason
of the State affording full protection to labor. 9 that the documents containing the disauthorizations have the
signatures of the union members. The Court finds these
retractions to be valid. There is nothing in the law which requires
The respondent-Union brushed aside the defects pointed out by
that the disauthorization must be in individual form.
petitioners in the manner of compliance with the legal
requirements as "insignificant technicalities." On the contrary, the
failure of the Union to comply strictly with the requirements set Moreover, it is well-settled that "all doubts in the implementation
out by the law invalidates the questioned special assessment. and interpretation of the provisions of the Labor Code ... shall be
Substantial compliance is not enough in view of the fact that the resolved in favor of labor."12 And as previously stated, labor in
special assessment will diminish the compensation of the union this case refers to the union members, as employees of the
members. Their express consent is required, and this consent must Company. Their mere desire to establish a separate bargaining
be obtained in accordance with the steps outlined by law, which unit, albeit unproven, cannot be construed against them in relation
must be followed to the letter. No shortcuts are allowed. to the legality of the questioned special assessment. On the
contrary, the same may even be taken to reflect their
dissatisfaction with their bargaining representative, the
The applicable provisions are clear. The Union itself admits that
respondent-Union, as shown by the circumstances of the instant
both paragraphs (n) and (o) of Article 241 apply. Paragraph (n)
petition, and with good reason.
refers to "levy" while paragraph (o) refers to "check-off" of a
special assessment. Both provisions must be complied with.
Under paragraph (n), the Union must submit to the Company a The Med-Arbiter correctly ruled in his Order that:
written resolution of a majority of all the members at a general
membership meeting duly called for the purpose. In addition, the The mandate of the majority rank and file have
secretary of the organization must record the minutes of the (sic) to be respected considering they are the
meeting which, in turn, must include, among others, the list of all ones directly affected and the realities of the
the members present as well as the votes cast. high standards of survival nowadays. To
ignore the mandate of the rank and file would
As earlier outlined by petitioners, the Union obviously failed to enure to destabilizing industrial peace and
comply with the requirements of paragraph (n). It held local harmony within the rank and file and the
membership meetings on separate occasions, on different dates employer's fold, which we cannot
and at various venues, contrary to the express requirement that countenance.
Moreover, it will be recalled that precisely
union dues are collected from the union
members to be spent for the purposes alluded
to by respondent. There is no reason shown
that the regular union dues being now
G.R. No. 115949             March 16, 2000
implemented is not sufficient for the alleged
expenses. Furthermore, the rank and file have
spoken in withdrawing their consent to the EVANGELINE J. GABRIEL, TERESITA C. LUALHATI,
special assessment, believing that their regular EVELYN SIA, RODOLFO EUGENIO, ISAGANI MAKISIG,
union dues are adequate for the purposes stated and DEMETRIO SALAS, petitioners,
by the respondent. Thus, the rank and file vs.
having spoken and, as we have earlier THE HONORABLE SECRETARY OF LABOR AND
mentioned, their sentiments should be EMPLOYMENT and SIMEON SARMIENTO, JESUS
respected. CARLOS MARTINEZ III, ALBERT NAPIAL, MARVIN
ALMACIN, ROGELIO MATEO, GLENN SIAPNO,
EMILIANO CUETO, SALOME ATIENZA, NORMA V. GO,
Of the stated purposes of the special assessment, as embodied in
JUDITH DUDANG, MONINA DIZON, EUSEBIO
the board resolution of the Union, only the collection of a special
ROMERO, ISAGANI MORALES, ELISEO
fund for labor and education research is mandated, as correctly
BUENAVENTURA, CLEMENTE AGCAMARAN,
pointed out by the Union. The two other purposes, namely, the
CARMELITA NOLASCO, JOVITA FERI, LULU ACOSTA,
purchase of vehicles and other items for the benefit of the union
CAROL LAZARO, NIDA ARRIZA, ROMAN BERNARDO,
officers and the general membership, and the payment of services
DOMINGO B. MACALDO, EUGENE PIDLAOAN, MA.
rendered by union officers, consultants and others, should be
SOCORRO T. ANGOB, JOSEPHINE ALVAREZ,
supported by the regular union dues, there being no showing that
LOURDES FERRER, JACQUILINE BAQUIRAN, GRACIA
the latter are not sufficient to cover the same.
R. ESCUADRO, KRISTINA HERNANDEZ, LOURDES
IBEAS, MACARIO GARCIA, BILLY TECSON, ALEX
The last stated purpose is contended by petitioners to fall under RECTO III, LEBRUDO, JOSE RICAFORTE, RODOLFO
the coverage of Article 222 (b) of the Labor Code. The contention MORADA, TERESA AMADO, ROSITA TRINIDAD,
is impressed with merit. Article 222 (b) prohibits attorney's fees, JEANETTE ONG, VICTORINO LAS-AY, RANIEL DAYAO
negotiations fees and similar charges arising out of the conclusion OSCAR SANTOS, CRISTINA SALAVER, VICTORIA
of a collective bargaining agreement from being imposed on any ARINO, A.H. SAJO, MICHAEL BIETE, RED RP, GLORIA
individual union member. The collection of the special JUAT, ETHELINDA CASILAN, FAMER DIPASUPIL, MA.
assessment partly for the payment for services rendered by union HIDELISA POMER, MA. CHARLOTTE TAWATAO,
officers, consultants and others may not be in the category of GRACE REYES, ERNIE COLINA, ZENAIDA MENDOZA,
"attorney's fees or negotiations fees." But there is no question that PAULITA ADORABLE, BERNARDO MADUMBA,
it is an exaction which falls within the category of a "similar NESTOR NAVARRO, EASTER YAP, ALMA LIM, FELISA
charge," and, therefore, within the coverage of the prohibition in YU, TIMOTEO GANASTRA, REVELITA CARTAJENAS,
the aforementioned article. There is an additional proviso giving ANGELITO CABUAL, ROBERTA TAN, DOMINADOR
the Union President unlimited discretion to allocate the proceeds TAPO, GRACE LIM GADIANE JEMIE, CHRISTHDY
of the special assessment. Such a proviso may open the door to DAUD, BENEDICTO ACOSTA, JESUSA ACOSTA, MA.
abuse by the officers of the Union considering that the total AVELINA ARYAP, EVELYN BENITEZ, ESTERITA CHU,
amount of the special assessment is quite considerable — EVANGELINE CHU, BETTY CINCO, RICARDO
P1,027,694.33 collected from those union members who CONNEJO, MANULITO EVALO, FRANCIS LEONIDA,
originally authorized the deduction, and P1,267,863.39 from GREGORIO NOBLEZA, RODOLFO RIVERAL, ELSA SIA,
those who did not authorize the same, or subsequently retracted CLARA SUGBO, EDGARDO TABAO, MANUEL VELOSO,
their authorizations. 13 The former amount had already been MARLYN YU, ABSALON BUENA, WILFREDO PUERTO,
remitted to the Union, while the latter is being held in trust by the FLORENTINA PINGOL, MARILOU DAR, FE MORALES,
Company. MALEN BELLO, LORENA TAMAYO, CESAR LIM, PAUL
BALTAZAR, ALFREDO GAYAGAS, DUMAGUETE
The Court, therefore, stakes down the questioned special EMPLOYEES, CEBU EMPLOYEES, OZAMIZ
assessment for being a violation of Article 241, paragraphs (n) EMPLOYEES, TACLOBAN EMPLOYEES AND ALL
and (o), and Article 222 (b) of the Labor Code. OTHER SOLID BANK UNION MEMBERS, respondents.

WHEREFORE, the instant petition is hereby GRANTED. The QUISUMBING, J.:


Order of the Director of the Bureau of Labor Relations dated
August 19, 1988 is hereby REVERSED and SET ASIDE, while Before us is a special civil action for certiorari seeking to reverse
the order of the Med-Arbiter dated February 17, 1988 is partially the Order1 of public respondent dated June 3, 1994, in
reinstated, and the respondent Coca-Cola Bottlers (Philippines), Case No. OS-MA-A-8-170-92, which ruled that the workers
Inc. is hereby ordered to immediately remit the amount of through their union should be made to shoulder the expenses
P1,267,863.39 to the respective union members from whom the incurred for the professional services of a lawyer in connection
said amount was withheld. No pronouncement as to costs. This with the collective bargaining negotiations and that the
decision is immediately executory. reimbursement for the deductions from the workers should be
charged to the union's general fund or account.
SO ORDERED.
The records show the following factual antecedents:
Petitioners comprise the Executive Board of the SolidBank their conformity to the check-off of attorney's fees; and
Union, the duly recognized collective bargaining agent for the (2) the directive on the payment of 5% attorney's fees
rank and file employees of Solid Bank Corporation. Private should be deleted for lack of basis.
respondents are members of said union.
SO ORDERED.7
Sometime in October 1991, the union's Executive Board decided
to retain anew the service of Atty. Ignacio P. Lacsina (now On Motion for Reconsideration, public respondent affirmed the
deceased) as union counsel in connection with the negotiations said Order with modification that the union's counsel be dropped
for a new Collective Bargaining Agreement (CBA). Accordingly, as a party litigant and that the workers through their union should
on October 19, 1991, the board called a general membership be made to shoulder the expenses incurred for the attorney's
meeting for the purpose. At the said meeting, the majority of all services. Accordingly, the reimbursement should be charged to
union members approved and signed a resolution confirming the the union's general fund/account.8
decision of the executive board to engage the services of Atty.
Lacsina as union counsel.
Hence, the present petition seeking to partially annul the above-
cited order of the public respondent for being allegedly tainted
As approved, the resolution provided that ten percent (10%) of with grave abuse of discretion amounting to lack of jurisdiction.
the total economic benefits that may be secured through the
negotiations be given to Atty. Lacsina as attorney's fees. It also
The sole issue for consideration is, did the public respondent act
contained an authorization for SolidBank Corporation to check-
with grave abuse of discretion in issuing the challenged order?
off said attorney's fees from the first lump sum payment of
benefits to the employees under the new CBA and to turn over
said amount to Atty. Lacsina and/or his duly authorized Petitioners argue that the General Membership Resolution
representative.2 authorizing the bank to check-off attorney's fee from the first
lump sum payment of the legal benefits to the employees under
the new CBA satisfies the legal requirements for such
The new CBA was signed on February 21, 1992. The bank then,
assessment.9 Private respondents, on the other hand, claim that the
on request of the union, made payroll deductions for attorney's
check-off provision in question is illegal because it was never
fees from the CBA benefits paid to the union members in
submitted for approval at a general membership meeting called
accordance with the abovementioned resolution.
for the purpose and that it failed to meet the formalities mandated
by the Labor Code. 10
On October 2, 1992, private respondents instituted a complaint
against the petitioners and the union counsel before the
In check-off, the employer, on agreement with the Union, or on
Department of Labor and Employment (DOLE) for illegal
prior authorization from employees, deducts union dues or agency
deduction of attorney's fees as well as for quantification of the
fees from the latter's wages and remits them directly to the
benefits in the 1992 CBA.3 Petitioners, in response, moved for the
union. 11 It assures continuous funding; for the labor organization.
dismissal of the complaint citing litis pendentia, forum shopping
As this Court has acknowledged, the system of check-off is
and failure to state a cause of action as their grounds.4
primarily for the benefit of the union and only indirectly for the
individual employees. 12
On April 22, 1993, Med-Arbiter Paterno Adap of the DOLE-NCR
issued the following Order:
The pertinent legal provisions on check-offs are found in Article
222 (b) and Article 241 (o) of the Labor Code.
WHEREFORE, premises considered, the Respondents
Union Officers and Counsel are hereby directed to
Art. 222 (b) states:
immediately return or refund to the Complainants the
illegally deducted amount of attorney's fees from the
package of benefits due herein complainants under the No attorney's fees, negotiation fees or similar charges of
aforesaid new CBA. any kind arising from any collective bargaining
negotiations or conclusions of the collective agreement
shall be imposed on any individual member of the
Furthermore, Complainants are directed to pay five
contracting union: Provided, however, that attorney's
percent (5%) of the total amount to be refunded or
fees may be charged against unions funds in an amount
returned by the Respondent Union Officers and Counsel
to be agreed upon by the parties. Any contract,
to them in favor of Atty. Armando D. Morales, as
agreement or arrangement of any sort to the contrary
attorney's fees, in accordance with Section II, Rule VIII
shall be null and void. (Emphasis ours)
of Book II (sic) of the Omnibus Rules Implementing the
Labor Code.5
Art. 241 (o) provides:

On appeal, the Secretary of Labor rendered a Resolution dated
December 27, 1993, stating: Other than for mandatory activities under the Code, no
special assessment, attorney's fees, negotiation fees or
any other extraordinary fees may be checked off from
WHEREFORE, the appeal of respondents Evangeline
any amount due to an employee without an individual
Gabriel, et. al., is hereby partially granted and the Order
written authorization duly signed by the employee. The
of the Med-Arbiter dated 22 April 1993 is hereby
authorization should specifically state the amount,
modified as follows: (1) that the ordered refund shall be
limited to those union members who have not signified
purpose and beneficiary of the deduction. (Emphasis itself may require the individual worker to assume the
ours). obligation to pay attorney's fees from their own pockets.
So categorical is this intent that the law makes it clear
Art. 241 has three (3) requisites for the validity of the special that any agreement to the contrary shall be null and
assessment for union's incidental expenses, attorney's fees and void ab initio. (Emphasis ours.)1âwphi1
representation expenses. These are: 1) authorization by a written
resolution of the majority of all the members at the general From all the foregoing, we are of the considered view that public
membership meeting called for the purpose; (2) secretary's record respondent did not act with grave abuse of discretion in ruling
of the minutes of the meeting; and (3) individual written that the workers through their union should be made to shoulder
authorization for check off duly signed by the employees the expenses incurred for the services of a lawyer. And
concerned. accordingly the reimbursement should be charged to the union's
general fund or account. No deduction can be made from the
Clearly, attorney's fees may not be deducted or checked off from salaries of the concerned employees other than those mandated by
any amount due to an employee without his written consent. law.

After a thorough review of the records, we find that the General WHEREFORE, the petition is DENIED. The assailed Order dated
Membership Resolution of October 19, 1991 of the SolidBank June 3, 1994, of respondent Secretary of Labor signed by
Union did not satisfy the requirements laid down by law and Undersecretary Bienvenido E. Laguesma is AFFIRMED. No
jurisprudence for the validity of the ten percent (10%) special pronouncement as to costs.1âwphi1.nêt
assessment for union's incidental expenses, attorney's fees and
representation expenses. There were no individual written check SO ORDERED.
off authorizations by the employees concerned and so the
assessment cannot be legally deducted by their employer.

Even as early as February 1990, in the case of Palacol vs. Ferrer-


Calleja 13 we said that the express consent of employees is
required, and this consent must be obtained in accordance with
G.R. No. L-45824 June 19, 1985
the steps outlined by law, which must be followed to the letter.
No shortcuts are allowed. In Stellar Industrial Services, Inc. vs.
NLRC 14 we reiterated that a written individual authorization duly VOLKSCHEL LABOR UNION, petitioner,
signed by the employee concerned is a condition sine qua non for vs.
such deduction. BUREAU OF LABOR RELATIONS, ASSOCIATED
LABOR UNION FOR METAL, WORKERS, DMG, INC.,
PEOPLE'S CAR, INC., KARBAYAN INC., and RTC
These pronouncements are also in accord with the recent ruling of
TRADING, INC., respondents.
this Court in the case of ABS-CBN Supervisors Employees Union
Members vs. ABS-CBN Broadcasting
Corporation, et. al., 15 which provides: Ignacio P. Lacsina for petitioner.

Premises studiedly considered, we are of the irresistible William D. Dichoso for respondent DMG, Inc.
conclusion and, so find that the ruling in BPIEU-ALU
vs. NLRC that (1) the prohibition against attorney's fees Abraham B. Drapiza for private respondent.
in Article 222, paragraph (b) of the Labor Code applies
only when the payment of attorney's fees is effected
through forced contributions from the workers; and (2)
that no deduction must be take from the workers who
CUEVAS, J.:
did not sign the check-off authorization, applies to the
case under consideration. (Emphasis ours.)
Petition for certiorari to review the Resolutions dated January 25,
1977 and March 14, 1977 of the Bureau of Labor Relations.
We likewise ruled in Bank of the Philippine Islands Employees
Union-Association Labor Union (BPIEU-ALU) vs. NLRC, 16
On April 25. 1977, however, a Supplemental Petition was filed
seeking the issuance of —
. . . the afore-cited provision (Article 222 (b) of the
Labor Code) as prohibiting the payment of attorney's
fees only when it is effected through forced (1) A preliminary mandatory injunction
contributions from workers from their own funds as commanding respondents to return to
distinguished from the union funds. The purpose of the petitioner the union dues amounting to about
provision is to prevent imposition on the workers of the P55,000.00 lawfully pertaining to it but
duty to individually contribute their respective shares in illegally levied upon, collected and handed
the fee to be paid the attorney for his services on behalf over by respondent Bureau, acting through the
of the union in its negotiations with management. The NLRC sheriff, to respondent Associated Labor
obligation to pay the attorney's fees belongs to the Union for Metal workers, with the collusion of
union and cannot be shunted to the workers as their respondents DMG, Inc., Karbayan, Inc. and
direct responsibility. Neither the lawyer nor the union RTC Machineries, Inc.;
(2) A preliminary restraining order prohibiting gave the opinion that, petitioner's members should continue
respondents from making further delivery to paying their dues to ALUMETAL in the concept of agency fees. 1
respondent Associated Labor Union for Metal
workers of Union dues collected or to be From the said Resolution, of the Med-Arbiter both petitioner and
collected through check-off from the wages of respondent ALUMETAL appealed to the Director of respondent
petitioner's members by respondents, DMG, Bureau. Petitioner' contended that the Med-Arbiter's opinion to
Inc., Karbayan, Inc., RTC Machineries, Inc., the effect that petitioner's members remained obligated to pay
and People's Car, Inc., under or by virtue of the dues to respondent ALUMETAL was inconsistent with the
questioned writ of execution issued by dispositive finding that petitioner's disaffiliation from
respondent Bureau, dated April 4, 1977. ALUMETAL was valid. ALUMETAL, on the other hand,
assailed the Resolution in question asserting that the disaffiliation
Petitioner was once affiliated with the Associated Labor Union should have been declared contrary to law.
for Metal Workers (ALUMETAL for short). On August 1, 1975,
both unions, using the name Volkschel Labor Union Associated On January 25, 1977, respondent Bureau, through its Acting
Labor Union for Metal Workers, jointly entered into a collective Director, Francisco L. Estrella, REVERSED the Med-Arbiter's
bargaining agreement with respondent companies. One of the Resolution., and declared that the Bureau recognized "the
subjects dealt with is the payment of union dues which is continued affiliation of Volkschel Labor Union with the
provided for in Section 3, Article 1, of the CBA, which reads: Associated Labor Union for Metal Workers." 2

Section 3. CHECK-OFF. — The COMPANY Petitioner appealed the Acting Director's Resolution to the
agrees to make payroll deductions not softener Secretary of Labor know Minister of Labor and Employment)
than twice a month of UNION membership who, treating the appeal as a Motion for Reconsideration referred
dues and such special assessments fees or fines the same back to respondent Bureau On March 14, 1977, the
as may be duly authorized by the UNION, Bureau denied the appeal for lack of merit.
provided that the same is covered by the
individual check-off authorization of the
Hence, the instant petition.
UNION members. All said deductions shall be
promptly transmitted within five (5) days by
the COMPANY to the UNION Treasurer. The Meanwhile, on April 4, 1977, on motion of ALUMETAL, the
COMPANY shall prepare two (2) checks. One then Acting Secretary of Labor, Amado Gat Inciong, issued a of
(1) check will be under the name of the local execution commanding the Sheriff of the National Labor
union as their local fund including local special Relations Commission to enforce and execute the order of
assessment funds and the other check will be January 25, 1977, which has become final and
for the ALU Regional Office regarding the executory. 3 Pursuant thereto, the NLRC Sheriff enforced and
remittance of the UNION dues deduction. implemented the Order of January 25, 1977, as a result of which
respondent companies turned over and handed to respondent
federation the union dues and other assessments in accordance
On March 10, 1976, a majority of petitioner's members decided to
with the check-off provision of the CBA,
disaffiliate from respondent federation in order to operate on its
own as an independent labor group pursuant to Article 241
(formerly Article 240) of the Labor Code of the Philippines, the From the pleadings filed and arguments of counsel, the following
pertinent portion of which reads: issues present themselves for this Court's resolution.

Incumbent affiliates of existing federations or I


national unions may disaffiliate only for the
purpose of joining a federation or national Is petitioner union's disaffiliation from
union in the industry or region in which it respondent federation valid?
properly belongs or for the purpose of
operating as an independent labor group. II

Accordingly, a resolution was adopted and signed by petitioner's Do respondent companies have the right to
members revoking their check-off authorization in favor of effect union dues collections despite
ALUMETAL and notices thereof were served on ALUMETAL revocation by the employees of the check-off
and respondent companies. authorization? and

Confronted with the predicament of whether or not to continue III


deducting from employees' wages and remitting union dues to
respondent, ALUMETAL which wrote respondent companies
advising them to continue deducting union dues and remitting Is respondent federation entitled to union dues
them to said federation, respondent companies sought the legal payments from petitioner union's members
opinion of the respondent Bureau as regards the controversy notwithstanding their disaffiliation from said
between the two unions. On November 11, 1976, Med-Arbiter federation?
George A. Eduvalla of respondent Bureau rendered a Resolution
which in effect found the disaffiliation legal but at the same time We resolve the first issue in the affirmative.
The right of a local union to disaffiliate from its mother union is continues to represent the employees of an employer is entitled to
well-settled. In previous cases, it has been repeatedly held that a the check-off dues under a collective bargaining contract." 9
local union, being a separate and voluntary association, is free to
serve the interest of all its members including the freedom to WHEREFORE, the Resolutions of the Bureau of Labor Relations
disaffiliate when circumstances warrant. 4 This right is consistent of January 25, 1977 and March 14, 1977 are REVERSED and
with the Constitutional guarantee of freedom of association SET ASIDE. Respondent ALUMETAL is ordered to return to
(Article IV, Section 7, Philippine Constitution). petitioner union all the union dues enforced and collected through
the NLRC Sheriff by virtue of the writ of execution dated April 4,
Petitioner contends that the disaffiliation was not due to any 1977 issued by respondent Bureau.
opportunists motives on its part. Rather it was prompted by the
federation's deliberate and habitual dereliction of duties as mother No costs.
federation towards petitioner union. Employees' grievances were
allegedly left unattended to by respondent federation to the
SO ORDERED.
detriment of the employees' rights and interests.

In reversing the Med-Arbiter's resolution, respondent Bureau


declared: the Department of Labor is set on a task to restructure
the labor movement to the end that the workers will unite
themselves along industry lines. Carried to its complete fruition, This is a petition for certiorari[1] seeking to set aside the 31 July
only one union for every industry will remain to bargain 1996 Decision[2] of the National Labor Relations Commission
collectively for the workers. The clear policy therefore even now affirming the 30 June 1995 Decision of the Labor Arbiter holding
is to conjoin workers and worker groups, not to dismember petitioners Philippine Skylanders, Inc., Mariles C. Romulo[3] and
them. 5 This policy is commendable. However, we must not lose Francisco Dakila as well as the elected officers of the Philippine
sight of the constitutional mandate of protecting labor and the Skylanders Employees and Workers Association-PAFLU[4] guilty
workers' right to self-organization. In the implementation and of unfair labor practice and ordering them to pay private
interpretation of the provisions of the Labor Code and its respondent Philippine Association of Free Labor Union  (PAFLU)
implementing regulations, the workingman's welfare should be September[5] P150,000.00 as damages.  Petitioners likewise seek
the primordial and paramount consideration. In the case at bar, it the reversal of the 31 October 1996 Resolution of the NLRC
would go against the spirit of the labor law to restrict petitioner's denying their Motion for Reconsideration.
right to self-organization due to the existence of the CBA. We
agree with the Med-Arbiter's opinion that "A disaffiliation does In November 1993 the Philippine Skylanders Employees
not disturb the enforceability and administration of a collective Association (PSEA), a local labor union affiliated with the
agreement; it does not occasion a change of administrators of the Philippine Association of Free Labor Unions (PAFLU)
contract nor even an amendment of the provisions thereof." 6 But September (PAFLU),  won in the certification election conducted
nowhere in the record does it appear that the contract entered into among the rank and file employees of Philippine Skylanders, Inc.
by the petitioner and ALUMETAL prohibits the withdrawal of (PSI).  Its rival union, Philippine Skylanders Employees
the former from the latter. Association-WATU (PSEA-WATU) immediately protested the
result of the election before the Secretary of Labor.
This now brings us to the second issue. Under Section 3, Article I,
Several months later, pending settlement of the controversy,
of the CBA, the obligation of the respondent companies to deduct
PSEA sent PAFLU a notice of disaffiliation citing as reason
and remit dues to ALUMETAL is conditioned on the individual
PAFLU's supposed deliberate and habitual dereliction of duty
check-off authorization of petitioner's members, In other words,
toward its members.  Attached to the notice was a copy of the
ALUMETAL is entitled to receive the dues from respondent
resolution adopted and signed by the officers and members of
companies as long as petitioner union is affiliated with it and
PSEA authorizing their local union to disaffiliate from its mother
respondent companies are authorized by their employees
federation.
(members of petitioner union) to deduct union dues. Without said
affiliation, the employer has no link to the mother union. The
PSEA subsequently affiliated itself with the National Congress of
obligation of an employee to pay union dues is coterminous with
Workers (NCW), changed its name to Philippine Skylanders
his affiliation or membership. "The employees' check-off
Employees Association - National Congress of Workers (PSEA-
authorization, even if declared irrevocable, is good only as long as
NCW), and to maintain continuity within the organization,
they remain members of the union concerned." 7 A contract
allowed the former officers of PSEA-PAFLU to continue
between an employer and the parent organization as bargaining
occupying their positions as elected officers in the newly-forged
agent for the employees is terminated by the disaffiliation of the
PSEA-NCW.
local of which the employees are members. 8 Respondent
companies therefore were wrong in continuing the check-off in
On 17 March 1994 PSEA-NCW entered into a collective
favor of respondent federation since they were duly notified of the
bargaining agreement with PSI which was immediately registered
disaffiliation and of petitioner's members having already
with the Department of Labor and Employment.
rescinded their check-off authorization.
Meanwhile, apparently oblivious to PSEA's shift of allegiance,
With the view we take on those two issues, we find no necessity PAFLU Secretary General Serafin Ayroso wrote Mariles C.
in dwelling further on the last issue. Suffice it to state that Romulo requesting a copy of PSI's audited financial statement. 
respondent federation is not entitled to union dues payments from Ayroso explained that with the dismissal of PSEA-WATU's
petitioner's members. "A local union which has validly withdrawn election protest the time was ripe for the parties to enter into a
from its affiliation with the parent association and which collective bargaining agreement.
PAFLU and their officers to pay PAFLU P150,000.00 in
On 30 July 1994 PSI through its personnel manager Francisco damages.[10]
Dakila denied the request citing as reason PSEA's disaffiliation
from PAFLU and its subsequent affiliation with NCW. PSI, PSEA and their respective officers appealed to the National
Labor Relations Commission (NLRC).  But the NLRC upheld the
Agitated by PSI's recognition of PSEA-NCW, PAFLU through Decision of the Labor Arbiter and conjectured that since an
Serafin Ayroso filed a complaint for unfair labor practice against election protest questioning PSEA-PAFLU's certification as the
PSI, its president Mariles Romulo and personnel manager sole and exclusive bargaining agent was pending resolution
Francisco Dakila.   PAFLU alleged that aside from PSI's refusal before the Secretary of Labor, PSEA could not validly separate
to bargain collectively with its workers, the company through its from PAFLU, join another national federation and subsequently
president and personnel manager, was also liable for interfering enter into a collective bargaining agreement with its employer-
with its employees' union activities.[6] company.[11]

Two (2) days later or on 6 October 1994 Ayroso filed another Petitioners separately moved for reconsideration but both motions
complaint in behalf of PAFLU for unfair labor practice against were denied.   Hence, these petitions for certiorari filed by PSI
Francisco Dakila.  Through Ayroso PAFLU claimed that Dakila and PSEA-NCW together with their respective officers pleading
was present in PSEA's organizational meeting thereby confirming for a reversal of the NLRC's Decision which they claimed to have
his illicit participation in union activities.  Ayroso added that the been rendered in excess of jurisdiction.  In due time, both
members of the local union had unwittingly fallen into the petitions were consolidated.
manipulative machinations of PSI and were lured into endorsing a
collective bargaining agreement which was detrimental to their In these petitions, petitioner PSEA together with its officers
interests.[7] The two (2) complaints were thereafter consolidated. argued that by virtue of their disaffiliation PAFLU as a mere
agent had no authority to represent them before any proceedings.  
On 1 February 1995 PAFLU amended its complaint by including They further asserted that being an independent labor union
the elected officers of PSEA-PAFLU as additional party PSEA may freely serve the interest of all its members and readily
respondents.   PAFLU averred that the local officers of PSEA- disaffiliate from its mother federation when circumstances so
PAFLU, namely Macario Cabanias, Pepito Rodillas, Sharon warrant.   This right, they averred, was consistent with the
Castillo, Danilo Carbonel, Manuel Eda, Rolando Felix, Jocelyn constitutional guarantee of freedom of association.[12]
Fronda, Ricardo Lumba, Joseph Mirasol, Nerisa Mortel, Teofilo
Quirong, Leonardo Reyes, Manuel Cadiente, and Herminia Riosa, For their part, petitioners PSI, Romulo and Dakila alleged that
were equally guilty of unfair labor practice since they brazenly their decision to bargain collectively with PSEA-NCW was
allowed themselves to be manipulated and influenced by actuated, to a large extent, by PAFLU's behavior.   Having heard
petitioner Francisco Dakila.[8] no objections or protestations from PAFLU relative to PSEA's
disaffiliation, they reckoned that PSEA's subsequent association
PSI, its president Mariles C. Romulo, and its personnel manager with NSW was done bona fide.[13]
Dakila moved for the dismissal of the complaint on the ground
that the issue of disaffiliation was an inter-union conflict which The Solicitor General filed a Manifestation in Lieu of
lay beyond the jurisdiction of the Labor Arbiter.  On the other Comment recommending that both petitions be granted.  In
hand, PSEA-NCW took the cudgels for its officers who were his Manifestation, the Solicitor General argued against the Labor
being sued in their capacities as former officers of PSEA-PAFLU Arbiter's assumption of jurisdiction citing the following as
and asserted that since PSEA was no longer affiliated with reasons:  first, there was no employer-employee relationship
PAFLU, Ayroso or PAFLU for that matter had no personality to between complainant Ayroso and PSI over which the Labor
file the instant complaint.  In support of this assertion, PSEA- Arbiter could rightfully assert his jurisdiction; second, since the
NCW submitted in evidence a Katunayan signed by 111 out of case involved a dispute between PAFLU as mother federation and
120 rank and file employees of PSI disauthorizing Ayroso or PSEA as local union, the controversy fell within the jurisdiction
PAFLU from instituting any action in their behalf.[9] of the Bureau of Labor Relations; and lastly, the relationship of
principal-agent between PAFLU and PSEA had been severed by
In a Decision rendered on 30 June 1995 the Labor Arbiter the local union through the lawful exercise of its right of
declared PSEA's disaffiliation from PAFLU invalid and held PSI, disaffiliation.[14]
PSEA-PAFLU and their respective officers guilty of unfair labor
practice.  The Decision explained that despite PSEA-PAFLU's Stripped of non-essentials, the fundamental issue tapers down to
status as the sole and exclusive bargaining agent of PSI's rank and the legitimacy of PSEA's disaffiliation.  To be more precise, may
file employees, the company knowingly sanctioned and PSEA, which is an independent and separate local union, validly
confederated with Dakila in actively assisting a rival union.  This, disaffiliate from PAFLU pending the settlement of an election
according to the Labor Arbiter, was a classic case of interference protest questioning its status as the sole and exclusive bargaining
for which PSI could be held responsible.  As PSEA-NCW's agent of PSI's rank and file employees?
personality was not accorded recognition, its collective bargaining
agreement with PSI was struck down for being invalid.  Ayroso's At the outset, let it be noted that the issue of disaffiliation is an
legal personality to file the complaint was sustained on the inter-union conflict the jurisdiction of which properly lies with
ratiocination that under the Labor Code no petition questioning the Bureau of Labor Relations (BLR) and not with the Labor
the majority status of the incumbent bargaining agent shall be Arbiter.[15] Nonetheless, with due recognition of this fact, we
entertained outside of the sixty (60)-day period immediately deem it proper to settle the controversy at this instance since to
before the expiry date of such five (5)-year term of the collective remand the case to the BLR would only mean intolerable delay
bargaining agreement that the parties may enter into.  for the parties.
Accordingly, judgment was rendered ordering PSI, PSEA-
The right of a local union to disaffiliate from its mother federation mother federation to protect its locals are not altogether to be
is not a novel thesis unillumined by case law.  In the landmark shunned.  It will however be to err greatly against the
case of Liberty Cotton Mills Workers Union vs. Liberty Cotton Constitution if the desires of the federation would be favored over
Mills, Inc.[16] we upheld the right of local unions to separate from those of its members.  That, at any rate, is the policy of the law. 
their mother federation on the ground that as separate and For if it were otherwise, instead of protection, there would be
voluntary associations, local unions do not owe their creation and disregard and neglect of the lowly workingmen.
existence to the national federation to which they are affiliated
but, instead, to the will of their members.  The sole essence of WHEREFORE, the petitions of Philippine Skylanders, Inc. and of
affiliation is to increase, by collective action, the common Philippine Skylanders and Workers Association-NCW, together
bargaining power of local unions for the effective enhancement with their respective officers, are GRANTED.    The  Decision of
and protection of their interests.   Admittedly, there are times the National Labor Relations Commission of  31 July 1996
when without succor and support local unions may find it hard, affirming the Decision of the Labor Arbiter of 30 June 1995
unaided by other support groups, to secure justice for themselves. holding petitioners Philippine Skylanders and Workers
Association-NCW, Philippine Skylanders, Inc. and their
Yet the local unions remain the basic units of association, free to respective officers, guilty of unfair labor practice and ordering
serve their own interests subject to the restraints imposed by the them to pay damages to private respondent Philippine Association
constitution and by-laws of the national federation, and free also of Free Labor Unions (PAFLU) September (now UNIFIED
to renounce the affiliation upon the terms laid down in the PAFLU) as well as the Resolution of 31 October 1996 denying
agreement which brought such affiliation into existence. reconsideration is REVERSED and SET ASIDE.  No costs.

Such dictum has been punctiliously followed since then.[17] SO ORDERED.

Upon an application of the aforecited principle to the issue at


hand, the impropriety of the questioned Decisions becomes
clearly apparent.  There is nothing shown in the records nor is it
claimed by PAFLU that the local union was expressly forbidden [G.R. No. 190515, June 06 : 2011]
to disaffiliate from the federation nor were there any conditions
imposed for a valid breakaway.  As such, the pendency of an CIRTEK EMPLOYEES LABOR UNION-FEDERATION OF
election protest involving both the mother federation and the local FREE WORKERS PETITIONER, VS. CIRTEK
union did not constitute a bar to a valid disaffiliation.  Neither ELECTRONICS, INC., RESPONDENT.
was it disputed by PAFLU that 111 signatories out of the 120
members of the local union, or an equivalent of 92.5% of the total RESOLUTION
union membership supported the claim of disaffiliation and had in
fact disauthorized PAFLU from instituting any complaint in their CARPIO MORALES, J.:
behalf.  Surely, this is not a case where one (1) or two (2)
members of the local union decided to disaffiliate from the This resolves the motion for reconsideration and supplemental
mother federation, but it is a case where almost all local union motion for reconsideration filed by respondent, Cirtek
members decided to disaffiliate. Electronics, Inc., of the Court's Decision dated November 15,
2010.
It was entirely reasonable then for PSI to enter into a collective
bargaining agreement with PSEA-NCW.  As PSEA had validly Respondent-movant avers that petitioner, in filing the petition for
severed itself from PAFLU, there would be no restrictions which certiorari under Rule 65, availed of the wrong remedy, hence, the
could validly hinder it from subsequently affiliating with NCW Court should have dismissed the petition outright.  It goes on to
and entering into a collective bargaining agreement in behalf of aver that the Court erred in resolving a factual issue — whether
its members. the August 24, 2005
Memorandum of Agreement (MOA) was validly entered into —,
There is a further consideration that likewise argues for the which is not the office of a petition for certiorari.
granting of the petitions.   It stands unchallenged that PAFLU
instituted the complaint for unfair labor practice against the Respondent-movant further avers that the MOA[1] signed by the
wishes of workers whose interests it was supposedly protecting.  remaining officers of petitioner Union and allegedly gratified by
The mere act of disaffiliation did not divest PSEA of its own its members should have been given credence by the Court.
personality; neither did it give PAFLU the license to act
independently of the local union.  Recreant to its mission, PAFLU Furthermore, respondent-movant maintains that the Secretary of
cannot simply ignore the demands of the local chapter and decide Labor cannot insist on a ruling beyond the compromise agreement
for its welfare.  PAFLU might have forgotten that as an agent it entered into by the parties; and that, as early as February 5, 2010,
could only act in representation of and in accordance with the petitioner Union had already filed with the Department of Labor
interests of the local union.  The complaint then for unfair labor and Employment (DOLE) a resolution of disaffiliation from the
practice lodged by PAFLU against PSI, PSEA and their Federation of Free Workers resulting in the latter's lack of
respective officers, having been filed by a party which has no personality to represent the workers in the present case.
legal personality to institute the complaint, should have been
dismissed at the first instance for failure to state a cause of action. The motion is bereft of merit.

Policy considerations dictate that in weighing the claims of a local Respondent indeed availed of the wrong remedy of certiorari
union as against those of a national federation, those of the former under Rule 65. Due, however, to the nature of the case, ape
must be preferred.  Parenthetically though, the desires of the involving workers' wages and benefits, and the fact that whether
the petition was filed under Rule 65 or appeal by certiorari under (8) When the findings of fact art without citation of specific
Rule 45 it was filed within 15 days (the reglementary period evidence on which the conclusions are based;
under Rule 45) from petitioner's receipt of the resolution of the
Court of Appeals' Resolution denying its motion for (9) When the facts set forth in the petition as well as in the
reconsideration, the Court resolved to give it due course. petitioners1 main and reply briefs are not disputed by the
As Almelor v. RTC of Las Piñas, et al.[2] restates: respondents; and

Generally, on appeal taken either to the Supreme Court or (10) When the findings of  fact of the  Court  of Appeals are
the CA by the wrong or inappropriate mode shall be premised on the supposed absence of evidence and contradicted
dismissed. This is to prevent the party from benefiting from one's by the evidence on record, (emphasis and underscoring supplied)
neglect and mistakes. However, like most rules, it carries
certain exceptions. After all, the ultimate purpose of all rules In the present case, the findings of the Secretary of Labor and the
of procedures is to achieve substantial justice as expeditously appellate court on whether the MOA is valid and binding are
as possible. (emphasis and underscoring supplied) conflicting, the former giving scant consideration thereon, latter
affording it more and the weight.
Respecting the attribution of error to the Court in ruling on a
question of fact, it bears recalling that a QUESTION OF FACET As found by the Secretary of Labor, the MOA came about as a
arises when the doubt or difference arises as to the truth or result of the constitution, at respondent's behest, of the Labor-
falsehood of alleged facts,[3] while a QUESTION OF LAW exists Management Council (LMC) which, he reminded the parties,
when the doubt or difference arises as to what the law is on a should not be used as an avenue for bargaining but for the
certain set of facts. purpose of affording workers to participate in policy and
derision-making.  Hence, the agreements embodied in the MOA
The present case presents the primordial issue of whether the were not the proper subject of the LMC deliberation or procedure
Secretary of Labor is empowered to give arbitral awards in the but of CBA negotiations and, therefore, deserving little weight.
exercise of his authority to assume jurisdiction over labor
disputes. The appellate court, held, however, that the Secretary did not
have the authority to give an arbitral award higher than what was
Ineluctably, the issue involves a determination and application of stated in the MOA. The conflicting views drew the Court to re-
existing law, the provisions of the Labor Code, and prevailing evaluate the facts as borne by the records, an exception to the rule
jurisprudence. that only questions of law may be dealt with in an appeal by
Intertwined with the issue, however, is the question of validity of certiorari under Rule 45.
the MOA and its ratification which, as movant correctly points
out, is a question of fact and one which is not appropriate for a As discussed in the Decision under reconsideration, the then
petition for review on certiorari under Rule 45. The rule, Acting Secretary of Labor Manuel G. Imson acted well within his
however, is not without exceptions, viz: jurisdiction in ruling that the wage increases to be given are P10
per day effective January 1, 2004 and P15 per day effective
This rule provides that Hie parlies may raise only iquestions of January 1, 2005, pursuant to his power to assume jurisdiction
law, because the Supreme Court is dot a trier of facts. Generally, under Art. 263 (g)[4] of the Labor Code.
we are not duty-bound to analyze again and weigh the
evidence'introduced in and considered by the tribunals While an arbitral award cannot per se be categorized as an
below.  When supported by substantial evidence, the findings agreement voluntarily entered into by the parties because it
of fact of the CA are conclusive and binding on the parties requires the interference and imposing power of the State thru the
and are not renewable by this Court, unless the case falls Secretary of Labor when he assumes jurisdiction, the award can
under any of the following recognized exceptions: be considered, as an approximation of a collective bargaining
agreement which would otherwise have been entered into by
(1) When the conclusion is a finding grounded entirely on the parties. Hence, it has the force and effect of a valid contract
speculation, surmises and conjectures: obligation between the parties.[5]

(2) When the inference made is manifestly mistaken, absurd or In determining arbitral awards then, aside from the MOA, courts
impossible; considered  other factors and documents including, as in this case,
the financial documents[6] submitted by  respondent as well as  its
(3) Where there is a grave abuse of discretion: previous bargaining history and financial outlook and
improvements as stated in its own website.[7]
(4) When the  judgment is based on a misapprehension of
facts; The appellate court's ruling that giving credence to the "Pahavag"
and the minutes of the meeting which were not verified and
(5) When the findings of fact are conflating: notarized would violate the rule on parol evidence is erroneous. 
The parol evidence rule, like other rules on  evidence, should not
(6) When the Court of Appeals, in making its findings, went be strictly applied in labor cases. Interphil Laboratories
beyond the issues of the case and the same is contrary to the Employees Union-FFW v.  Interphil Laboratories Inc.[8] teaches:
admissions of both appellant and appellee;
[R]eliance on the parol evidence rule is misplaced.  In labor
(7) When the findings arc contrary  to those of , the trial cases pending before the Commission or the Labor Arbiter, the
court: rules of evidence prevailing in courts of law or equity are not
controlling.  Rules of procedure and evidence are not applied in a
very rigid and technical sense in labor cases. Hence, the Labor (g) validity/invalidity of impeachment/expulsion;- of union and
Arbiter is not precluded from accepting and evaluating evidence workers' association officers and members:
other than. and even contrary to, what is stated in the CBA. (h) validity/invalidity of voluntary recognition;
(emphasis and underscoring [supplied) (i) opposition to application for union and CBA registration:
(j) violations of or disagreements over any provision in a union or
On the contention that the MOA should have been given credence workers' association constitution and by-laws:
because it was validly entered into by the parties, the Court notes (k) disagreements over chartering or registration of labor
that even those who signed it expressed reservations, thereto.  A organizations and collective bargaining agreements:
CBA (assuming in this case that the MOA can be treated as one) (l) violations of the rights and conditions of union or workers'
is a contract imbued with public interest.  It must thus be given a association membership;
liberal, practical and realistic, rather than a narrow and technical (m violations of the rights of legitimate labor organizations,
construction, with due consideration to the context in which it is ) except interpretation of collective bargaining agreements;
negotiated and the purpose for which it is intended.[9] (n) such other disputes or conflicts involving the rights to self-
organization, union membership and collective bargaining -
As for the contention that the alleged disaffiliation of the Union
from the FFW during the pendency of the case resulted in the
FFW losing its personality to represent the Union, the same does (1) between and among legitimate labor organizations;
not affect the Court's upholding of the authority of the Secretary (2) between and  among members of a union or workers'
of Labor to impose arbitral awards higher than what was association.
supposedly agreed upon in the MOA. Contrary to respondent's SECTION 2. Coverage. - Other related labor relations disputes
assertion, the "unavoidable issue of disaffiliation" bears no shall include any conflict between a labor union and the employer
significant legal repercussions to warrant the reversal of the or any individual, entity or group that is not a labor organization
Court's Decision. or workers' association. This
includes: (1) cancellation of registration of unions and workers'
En passant, whether there was a valid disaffiliation is a factual associations; and (2) a petition for interpleader.[10] (emphasis
issue.  Besides, the alleged disaffiliation of the Union from the supplied)
FFW was by virtue of a Resolution signed on February 23, 2010
and submitted to the DOLE Laguna Field Office on March 5, Indeed, as respondent-movant itself argues, a local union may
2010 two months after the present petition was filed on December disaffiliate at any time from its mother federation, absent any
22, 2009, - hence, it did not affect FFW and its Legal Center's showing that the same is  prohibited under its constitution or rule. 
standing to file the petition nor this Court's jurisdiction to resolve Such, however, does not result in it losing its legal personality
the same. altogether.  Verily, Anglo-KMU v. Samahan Ng Mga
Manggagawang Nagkakaisa Sa Manila Bar Spinning Mills At
At all events, the issue of disaffiliation is an infra-union dispute J.P. Coats[11] enlightens
which must be resolved in a different forum in an action at the
instance of either or both the FFW and the Union or a rival labor A local labor union is a separate and distinct unit primarily
organization, not the employer. designed to secure and maintain an equality of bargaining power
between the employer and their employee-members. A local
An infra-union dispute refers to any conflict between; and union does not one its existence to the federation with which it
among union members, including grievances arising from any is affiliated.  It is a separate and distinct voluntary association
violation of the rights and conditions of membership, violation owing its creation to the will of its members.  The mere act of
of or disagreement over any provision of (he union's affiliation does not divest the local union of its own
constitution and by-laws, or disputes arising from chartering personality, neither does it give the mother federation the
or disaffiliation of the union. Sections 1 and 2. Rule XI of license to act independently of the local union.  It only gives
Department Order No. 40-03, Series of 2003 of the DOLE rise to a contract of agency where the former acts in
enumerate the following circumstances as inter/intra-union representation of the latter, (emphasis and underscoring
disputes, viz: supplied)

RULE XI Whether then, as respondent claims, FFW "went against the will
INTER/INTRA-UNION DISPUTES AND and wishes of its principal" (the member-employees) by pursuing
OTHER RELATED LABOR RELATIONS DISPUTES the case despite the signing of the MOA, is not for the Court, nor
for respondent to determine, but for the Union and FFW to
SECTION I. Coverage. - Inter/intra-union disputes shall include: resolve on their own pursuant to their principal-agent relationship.

(a) cancellation of registration of a labor organization filed by its WHEREFORE, the motion for reconsiders ion of this Court's
members or by another labor organization: Decision of November 15, 2010 is DENIED.
(b) conduct of election of union and workers' association
officers/nullification of election of union and workers' SO ORDERED
association officers;
(c) audit/accounts examination of union or workers association
funds:
(d) deregistration of collective bargaining agreements:
(e) validity/invalidity of union affiliation or disaffiliation:
(f) validity/invalidity of acceptance/non-acceptance for union
membership;

Das könnte Ihnen auch gefallen